Suicidal, violent, and treatment-resistant

Article Type
Changed
Thu, 08/01/2019 - 00:01
Display Headline
Suicidal, violent, and treatment-resistant

CASE Violent, then catatonic

Mr. T, age 52, has a long history of schizo­affective disorder, depressed type; several suicide attempts; and violent episodes. He is admitted to a mental health rehabilitation center under a forensic commitment.

Several years earlier, Mr. T had been charged with first-degree attempted murder, assault with a deadly weapon, and abuse of a dependent/geriatric adult after allegedly stabbing his mother in the upper chest and neck. At that time, Mr. T was not in psychiatric treatment and was drinking heavily. He had become obsessed with John F. Kennedy’s assassination and believed the Central Intelligence Agency (CIA), not Lee Harvey Oswald, was responsible. He feared the CIA wanted to kill him because of his knowledge, and he heard voices from his television he believed were threatening him. He acquired knives for self-protection. When his mother arrived at his apartment to take him to a psychiatric appointment, he believed she was conspiring with the CIA and attacked her. Mr. T’s mother survived her injuries. He was taken to the county jail, where psychiatric staff noted that Mr. T was psychotic.

The court found Mr. T incompetent to stand trial and sent him to a state hospital for psychiatric treatment and competency restoration. After 3 years, he was declared unable to be restored because of repeated decompensations, placed on a conservatorship, and sent back to county jail.

In the jail, Mr. T began to show signs of catatonia. He refused medications, food, and water, and became mute. He was admitted to a medical center after a 45-minute episode that appeared similar to a seizure; however, all laboratory evaluations were within normal limits, head CT was negative, and an EEG was unremarkable.

Mr. T’s catatonic state gradually resolved with increasing dosages of lorazepam, as well as clozapine. He showed improved mobility and oral intake. A month later, his train of thought was rambling and difficult to follow, circumstantial, and perseverating. However, at times he could be directed and respond to questions in a linear and logical fashion. Lorazepam was tapered, discontinued, and replaced with gabapentin because Mr. T viewed taking lorazepam as a threat to his sobriety.

Recently, Mr. T was transferred to our mental health rehabilitation center, where he expresses that he is grateful to be in a therapeutic environment. Upon admission, his medication regimen consists of clozapine, 300 mg by mouth at bedtime, duloxetine, 60 mg/d by mouth, gabapentin 600 mg by mouth 3 times a day, and docusate sodium, 250 mg/d by mouth. Our team has a discussion about the growing recognition of the pro-inflammatory state present in many patients who experience serious mental illness and the importance of augmenting standard evidence-based psychopharmacotherapy with agents that have neuroprotective properties.1,2 We offer Mr. T minocycline, 100 mg by mouth twice daily, a potent anti-inflammatory agent that has been shown to improve symptoms of schizophrenia.2 Mr. T is reluctant to take minocycline, saying he is happy with his current medication regimen.

[polldaddy:10375843]

The authors’ observations

Several studies have found that acute psychosis is associated with an inflammatory state, and interleukin-6 (IL-6) is a crucial biomarker. A recent meta-analysis of serum cytokines in patients with schizophrenia found that IL-6 levels were significantly increased among acutely ill patients compared with controls.3 IL-6 levels significantly decreased after treating acute episodes of schizophrenia.3 Further, levels of peripheral IL-6 mRNA levels in individuals with schizophrenia are directly correlated with severity of positive symptoms.4

Continue to: A meta-analyis reported...

 

 

A meta-analysis reported that tumor necrosis factor-alpha and IL-6 are elevated during acute psychosis3; however, IL-6 normalized with treatment, whereas tumor necrosis factor-alpha did not. This means that IL-6 is a more clinically meaningful biomarker to help gauge treatment response.

EVALUATION Elevated markers of inflammation

Laboratory testing reveals that Mr. T’s IL-6 level is 56.64 pg/mL, which is significantly elevated (reference range: 0.31 to 5.00 pg/mL). After reviewing the IL-6 results with Mr. T and explaining that there is “too much inflammation” in his brain, he agrees to take minocycline and complete follow-up IL-6 level tests to monitor his progress during treatment.

HISTORY Alcohol abuse, treatment resistance

According to Mr. T’s mother, he had met all developmental milestones and graduated from high school with plans to enter culinary school. At age 20, Mr. T began to experience psychotic symptoms, telling family members that he was being followed by FBI agents and was receiving messages from televisions. He began drinking heavily and was arrested twice for driving under the influence. In his mid-20s, he attempted suicide by overdose after his father died. Mr. T required inpatient hospitalization nearly every year thereafter. His mother, a registered nurse, was significantly involved in his care and carefully documented his treatment history.

Mr. T has had numerous medication trials, including oral and long-acting injectable risperidone, olanzapine, aripiprazole, ziprasidone, lithium, gabapentin, buspirone, quetiapine, trazodone, bupropion, and paroxetine. None of these medications were effective.

In his mid-40s, Mr. T attempted suicide by wandering into traffic and being struck by a motor vehicle. A year later, he attempted suicide by driving his car at high speed into a concrete highway median. Mr. T told first responders that he was “possessed,” and a demonic entity “forced” him to crash his car. He begged law enforcement officers at the scene to give him a gun so he could shoot himself.

Continue to: Mr. T entered an intensive outpatient treatment program...

 

 

Mr. T entered an intensive outpatient treatment program and was switched from long-acting injectable risperidone to oral aripiprazole. After taking aripiprazole for several weeks, he began to gamble compulsively at a nearby casino. Frustrated by the lack of response to psychotropic medications and his idiosyncratic response to aripiprazole, he stopped psychiatric treatment, relapsed to alcohol use, and isolated himself in his apartment shortly before stabbing his mother.

EVALUATION Pharmacogenomics testing

At the mental health rehabilitation center, Mr. T agrees to undergo pharmacogenomics testing, which suggests that he will have a normal response to selective serotonin reuptake inhibitors and is unlikely to experience adverse reactions. He does not carry the 2 alleles that place him at higher risk of serious dermatologic reactions when taking certain mood stabilizers. He is heterozygous for the C677T allele polymorphism in the MTHFR gene that is associated with reduced folic acid metabolism, moderately decreased serum folate levels, and moderately increased homocysteine levels. On the pharmacokinetic genes tested, Mr. T has the normal metabolism genotype on 5 of 6 cytochrome P450 (CYP) enzymes; he has the ultrarapid metabolizer genotype on CYP1A2. He also has normal activity and intermediate metabolizer phenotype on the 2 UGT enzymes tested, which are responsible for the glucuronidation process, a major part of phase II metabolism.

Based on these results, Mr. T’s clozapine dosage is decreased by 50% (from 300 to 150 mg/d) and he is started on fluvoxamine, 50 mg/d, because it is a strong inhibitor of CYP1A2. The reduced conversion of clozapine to norclozapine results in an average serum clozapine level of 527 ng/mL (a level of 350 ng/mL is usually therapeutic in patients with schizophrenia) and norclozapine level of 140 ng/mL (clozapine:norclozapine ratio = 3.8), which is to be expected because fluvoxamine can increase serum clozapine levels.

Due to accumulating evidence in the literature suggesting that latent infections in the CNS play a role in serious mental illnesses such as schizophrenia, Mr. T undergoes further laboratory testing.

[polldaddy:10375845]

The authors’ observations

Mr. T tested positive for TG and CMV and negative for HSV-1. We were aware of accumulating evidence that latent infections in the CNS play a role in serious mental illnesses such as schizophrenia, specifically TG5—a parasite transmitted by cats—and CMV and HSV-1,6 which are transmitted by humans. The theory that TG infection could be a factor in schizophrenia emerged in the 1990s but only in recent years received mainstream scientific attention. Toxoplasma gondii, the infectious parasite that causes toxoplasmosis, infects more than 30 million people in the United States; however, most individuals are asymptomatic because of the body’s immune response to the parasite.7

Continue to: A study of 162 individuals...

 

 

A study of 162 individuals with schizophrenia, bipolar disorder, or major depressive disorder found that this immunologic profile is associated with suicide attempts,8 which is consistent with Mr. T’s history. Research suggests that individuals with schizophrenia who have latent TG infection have a more severe form of the illness compared with patients without the infection.9-12 Many of these factors were present in Mr. T’s case (Table 18-12).

Features of schizophrenia in patients with Toxoplasma gondii infection

TREATMENT Improvement, then setback

Mr. T’s medication regimen at the rehabilitation center includes clozapine, 100 mg/d; minocycline, 200 mg/d; fluvoxamine, 200 mg/d; and N-acetylcysteine, 1,200 mg/d. N-acetylcysteine is an antioxidant that could ease negative symptoms of schizophrenia by reducing oxidative stress caused by free radicals.13 Mr. T makes slow but steady improvement, and his IL-6 levels drop steadily (Figure 1).

Interleukin-6 (IL-6) levels over the course of Mr. T’s hospitalization

After 6 months in the rehabilitation center, Mr. T no longer experiences catatonic symptoms and is able to participate in the therapeutic program. He is permitted to leave the facility on day passes with family members. However, approximately every 8 weeks, he continues to cycle through periods of intense anxiety, perseverates on topics, and exhibits fragmented thinking and speech. During these episodes, he has difficulty receiving and processing information.

During one of these periods, Mr. T eats 4 oleander leaves he gathered while on day pass outside of the facility. After he experiences stomach pain, nausea, and vomiting, he informs nursing staff that he ate oleander. He is brought to the emergency department, receives activated charcoal and a digoxin antidote, and is placed on continuous electrocardiogram monitoring. When asked why he made the suicide attempt, he said “I realized things will never be the same because of what happened. I felt trapped.” He later expresses regret and wants to return to the mental health rehabilitation center.

At the facility, Mr. T agrees to take 2 more agents—valproic acid and ginger root extract—that specifically target latent toxoplasmosis infection before pursuing electroconvulsive therapy. We offer valproic acid because it inhibits replication of TG in an in vitro model.14 Mr. T is started on extended-release valproic acid, 1,500 mg/d, which results in a therapeutic serum level of 74.8 µg/mL.

Continue to: Additionally, Mr. T expresses interest...

 

 

Additionally, Mr. T expresses interest in taking “natural” agents in addition to psychotropics. After reviewing the quality of available ginger root extract products, Mr. T is started on a supplement that contains 22.4 mg of gingerols and 6.7 mg of shogaols, titrated to 4 capsules twice daily.

The authors’ observations

Psychotropic medications with anti-toxoplasmic activity

A retrospective cross-sectional analysis reported that patients with bipolar disorder who received medications with anti-toxoplasmic activity (Table 215), specifically valproic acid, had significantly fewer lifetime depressive episodes compared with patients who received medications without anti-toxoplasmic activity.15

 

Alternative medicine options

Research has demonstrated the beneficial effects of Chinese herbal plants for toxoplasmosis16,17 and ginger root extract has potent anti-toxoplasmic activity. A mouse model found that ginger root extract (Zingiber officinale) reduced the number of TG-infected cells by suppressing activation of apoptotic proteins the parasite induces, which prevents programmed cell death.18

Table 3 presents a stepwise approach to identifying and treating inflammation in patients with treatment-resistant psychosis.

A stepwise approach to identifying and treating inflammation in patients with treatment-resistant psychosis

OUTCOME Immune response, improvement

One month after the valproic acid and ginger root extract therapy is initiated, Mr. T’s toxoplasma antibody immunoglobulin G increases by 15.2 IU/mL, indicating that his immune system is mounting an enhanced response against the parasite (Figure 2). Mr. T continues to make progress while receiving the new regimen of clozapine, minocycline, valproic acid, and ginger root extract. He no longer cycles into periods of intense anxiety, perseverative thought, and fragmented thought and speech. He participates meaningfully in weekly psychotherapy and hopes to live independently and obtain gainful employment.

Toxoplasma antibody IgG titers before and after the addition of valproic acid and ginger root extract

The District Attorney’s office dismisses his criminal charges, and Mr. T is discharged to a less restrictive level of care.

Continue to: Bottom Line

 

 

Bottom Line

Several studies have shown that neuroinflammation increases the severity of mental illness. Consider adjunct anti-inflammatory agents for patients who have elevated levels of inflammatory biomarkers and for whom standard treatment approaches do not adequately control psychiatric symptoms. Also consider testing for the presence of latent infections in the CNS, which could reveal the underlying cause of treatment resistance or the genesis of disabling psychiatric symptoms.

Related Resources

  • Fond G, Macgregor A, Tamouza R, et al. Comparative analysis of anti-toxoplasmic activity of antipsychotic drugs and valproate. Eur Arch Psychiatry Clin Neurosci. 2014;264(2):179-183.
  • Hamdani N, Daban-Huard C, Lajnef M, et al. Cognitive deterioration among bipolar disorder patients infected by Toxoplasma gondii is correlated to interleukin 6 levels. J Affect Disord. 2015;179:161-166.
  • Monroe JM, Buckley PF, Miller BJ. Meta-analysis of antitoxoplasma gondii IgM antibodies in acute psychosis. Schizophr Bull. 2015;41(4):989-998.

Drug Brand Names

Acyclovir • Zovirax
Aripiprazole • Abilify
Bupropion • Wellbutrin
Buspirone • Buspar
Clozapine • Clozaril
Duloxetine • Cymbalta
Fluphenazine • Prolixin
Fluvoxamine • Luvox
Gabapentin • Neurontin
Haloperidol • Haldol
Lithium • Eskalith, Lithobid
Lorazepam • Ativan
Loxapine • Loxitane
Minocycline • Minocin
Olanzapine • Zyprexa
Paliperidone • Invega
Paroxetine • Paxil
Quetiapine • Seroquel
Risperidone • Risperdal, Risperdal Consta
Thioridazine • Mellaril
Trifluoperazine • Stelazine
Trazodone • Desyrel
Valproic acid • Depakote
Ziprasidone • Geodon

References

1. Koola MM, Raines JK, Hamilton RG, et al. Can anti-inflammatory medications improve symptoms and reduce mortality in schizophrenia? Current Psychiatry. 2016;15(5):52-57.
2. Nasrallah HA. Are you neuroprotecting your patients? 10 Adjunctive therapies to consider. Current Psychiatry. 2016;15(12):12-14.
3. Goldsmith DR, Rapaport MH, Miller BJ. A meta-analysis of blood cytokine network alterations in psychiatric patients: comparisons between schizophrenia, bipolar disorder and depression. Mol Psychiatry. 2016;21(12):1696-1709.
4. Chase KA, Cone JJ, Rosen C, et al. The value of interleukin 6 as a peripheral diagnostic marker in schizophrenia. BMC Psychiatry. 2016;16:152.
5. Torrey EF, Bartko JJ, Lun ZR, et al. Antibodies to Toxoplasma gondii in patients with schizophrenia: a meta-analysis. Schizophr Bull. 2007;33(3):729-736.
6. Shirts BH, Prasad KM, Pogue-Geile MF, et al. Antibodies to cytomegalovirus and herpes simplex virus 1 associated with cognitive function in schizophrenia. Schizophr Res. 2008;106(2-3):268-274.
7. Centers for Disease Control and Prevention. Parasites - Toxoplasmosis (Toxoplasma infection). https://www.cdc.gov/parasites/toxoplasmosis/index.html. Accessed February 26, 2019.
8. Dickerson F, Wilcox HC, Adamos M, et al. Suicide attempts and markers of immune response in individuals with serious mental illness. J Psychiatr Res. 2017;87:37-43.
9. Celik T, Kartalci S, Aytas O, et al. Association between latent toxoplasmosis and clinical course of schizophrenia - continuous course of the disease is characteristic for Toxoplasma gondii-infected patients. Folia Parasitol (Praha). 2015;62. doi: 10.14411/fp.2015.015.
10. Dickerson F, Boronow J, Stallings C, et al. Toxoplasma gondii in individuals with schizophrenia: association with clinical and demographic factors and with mortality. Schizophr Bull. 2007;33(3):737-740.
11. Esshili A, Thabet S, Jemli A, et al. Toxoplasma gondii infection in schizophrenia and associated clinical features. Psychiatry Res. 2016;245:327-332.
12. Holub D, Flegr J, Dragomirecka E, et al. Differences in onset of disease and severity of psychopathology between toxoplasmosis-related and toxoplasmosis-unrelated schizophrenia. Acta Psychiatr Scand. 2013;127(3):227-238.
13. Chen AT, Chibnall JT, Nasrallah HA. Placebo-controlled augmentation trials of the antioxidant NAC in schizophrenia: a review. Ann Clin Psychiatry. 2016;28(3):190-196.

14. Jones-Brando L, Torrey EF, Yolken R. Drugs used in the treatment of schizophrenia and bipolar disorder inhibit the replication of Toxoplasma gondii. Schizophr Res. 2003;62(3):237-244.
15. Fond G, Boyer L, Gaman A, et al. Treatment with anti-toxoplasmic activity (TATA) for toxoplasma positive patients with bipolar disorders or schizophrenia: a cross-sectional study. J Psychiatr Res. 2015;63:58-64.
16. Wei HX, Wei SS, Lindsay DS, et al. A systematic review and meta-analysis of the efficacy of anti-Toxoplasma gondii medicines in humans. PLoS One. 2015;10(9):e0138204.
17. Zhuo XH, Sun HC, Huang B, et al. Evaluation of potential anti-toxoplasmosis efficiency of combined traditional herbs in a mouse model. J Zhejiang Univ Sci B. 2017;18(6):453-461.
18. Choi WH, Jiang MH, Chu JP. Antiparasitic effects of Zingiber officinale (Ginger) extract against Toxoplasma gondii. Journal of Applied Biomedicine. 2013;11:15-26.

Article PDF
Author and Disclosure Information

Ms. Adabie is Psychiatric Mental Health Nurse Practitioner, Doctor of Nursing Practice Candidate, University of California, San Francisco School of Nursing, San Francisco, California. Dr. Hassid is Infection Control Officer, San Mateo Medical Center and Clinics, San Mateo, California. Dr. Hastik is a psychiatrist in private practice, San Francisco, California. Ms. Sinclair is Director of Research Treatment Advocacy Center, Arlington, Virginia. Dr. Quanbeck is Associate Medical Director, Department of Psychiatry, San Mateo County Health, San Mateo, California.

Disclosures
The authors report no financial relationships with any companies whose products are mentioned in this article, or with manufacturers of competing products.

Issue
Current Psychiatry - 18(8)
Publications
Topics
Page Number
e3-e9
Sections
Author and Disclosure Information

Ms. Adabie is Psychiatric Mental Health Nurse Practitioner, Doctor of Nursing Practice Candidate, University of California, San Francisco School of Nursing, San Francisco, California. Dr. Hassid is Infection Control Officer, San Mateo Medical Center and Clinics, San Mateo, California. Dr. Hastik is a psychiatrist in private practice, San Francisco, California. Ms. Sinclair is Director of Research Treatment Advocacy Center, Arlington, Virginia. Dr. Quanbeck is Associate Medical Director, Department of Psychiatry, San Mateo County Health, San Mateo, California.

Disclosures
The authors report no financial relationships with any companies whose products are mentioned in this article, or with manufacturers of competing products.

Author and Disclosure Information

Ms. Adabie is Psychiatric Mental Health Nurse Practitioner, Doctor of Nursing Practice Candidate, University of California, San Francisco School of Nursing, San Francisco, California. Dr. Hassid is Infection Control Officer, San Mateo Medical Center and Clinics, San Mateo, California. Dr. Hastik is a psychiatrist in private practice, San Francisco, California. Ms. Sinclair is Director of Research Treatment Advocacy Center, Arlington, Virginia. Dr. Quanbeck is Associate Medical Director, Department of Psychiatry, San Mateo County Health, San Mateo, California.

Disclosures
The authors report no financial relationships with any companies whose products are mentioned in this article, or with manufacturers of competing products.

Article PDF
Article PDF

CASE Violent, then catatonic

Mr. T, age 52, has a long history of schizo­affective disorder, depressed type; several suicide attempts; and violent episodes. He is admitted to a mental health rehabilitation center under a forensic commitment.

Several years earlier, Mr. T had been charged with first-degree attempted murder, assault with a deadly weapon, and abuse of a dependent/geriatric adult after allegedly stabbing his mother in the upper chest and neck. At that time, Mr. T was not in psychiatric treatment and was drinking heavily. He had become obsessed with John F. Kennedy’s assassination and believed the Central Intelligence Agency (CIA), not Lee Harvey Oswald, was responsible. He feared the CIA wanted to kill him because of his knowledge, and he heard voices from his television he believed were threatening him. He acquired knives for self-protection. When his mother arrived at his apartment to take him to a psychiatric appointment, he believed she was conspiring with the CIA and attacked her. Mr. T’s mother survived her injuries. He was taken to the county jail, where psychiatric staff noted that Mr. T was psychotic.

The court found Mr. T incompetent to stand trial and sent him to a state hospital for psychiatric treatment and competency restoration. After 3 years, he was declared unable to be restored because of repeated decompensations, placed on a conservatorship, and sent back to county jail.

In the jail, Mr. T began to show signs of catatonia. He refused medications, food, and water, and became mute. He was admitted to a medical center after a 45-minute episode that appeared similar to a seizure; however, all laboratory evaluations were within normal limits, head CT was negative, and an EEG was unremarkable.

Mr. T’s catatonic state gradually resolved with increasing dosages of lorazepam, as well as clozapine. He showed improved mobility and oral intake. A month later, his train of thought was rambling and difficult to follow, circumstantial, and perseverating. However, at times he could be directed and respond to questions in a linear and logical fashion. Lorazepam was tapered, discontinued, and replaced with gabapentin because Mr. T viewed taking lorazepam as a threat to his sobriety.

Recently, Mr. T was transferred to our mental health rehabilitation center, where he expresses that he is grateful to be in a therapeutic environment. Upon admission, his medication regimen consists of clozapine, 300 mg by mouth at bedtime, duloxetine, 60 mg/d by mouth, gabapentin 600 mg by mouth 3 times a day, and docusate sodium, 250 mg/d by mouth. Our team has a discussion about the growing recognition of the pro-inflammatory state present in many patients who experience serious mental illness and the importance of augmenting standard evidence-based psychopharmacotherapy with agents that have neuroprotective properties.1,2 We offer Mr. T minocycline, 100 mg by mouth twice daily, a potent anti-inflammatory agent that has been shown to improve symptoms of schizophrenia.2 Mr. T is reluctant to take minocycline, saying he is happy with his current medication regimen.

[polldaddy:10375843]

The authors’ observations

Several studies have found that acute psychosis is associated with an inflammatory state, and interleukin-6 (IL-6) is a crucial biomarker. A recent meta-analysis of serum cytokines in patients with schizophrenia found that IL-6 levels were significantly increased among acutely ill patients compared with controls.3 IL-6 levels significantly decreased after treating acute episodes of schizophrenia.3 Further, levels of peripheral IL-6 mRNA levels in individuals with schizophrenia are directly correlated with severity of positive symptoms.4

Continue to: A meta-analyis reported...

 

 

A meta-analysis reported that tumor necrosis factor-alpha and IL-6 are elevated during acute psychosis3; however, IL-6 normalized with treatment, whereas tumor necrosis factor-alpha did not. This means that IL-6 is a more clinically meaningful biomarker to help gauge treatment response.

EVALUATION Elevated markers of inflammation

Laboratory testing reveals that Mr. T’s IL-6 level is 56.64 pg/mL, which is significantly elevated (reference range: 0.31 to 5.00 pg/mL). After reviewing the IL-6 results with Mr. T and explaining that there is “too much inflammation” in his brain, he agrees to take minocycline and complete follow-up IL-6 level tests to monitor his progress during treatment.

HISTORY Alcohol abuse, treatment resistance

According to Mr. T’s mother, he had met all developmental milestones and graduated from high school with plans to enter culinary school. At age 20, Mr. T began to experience psychotic symptoms, telling family members that he was being followed by FBI agents and was receiving messages from televisions. He began drinking heavily and was arrested twice for driving under the influence. In his mid-20s, he attempted suicide by overdose after his father died. Mr. T required inpatient hospitalization nearly every year thereafter. His mother, a registered nurse, was significantly involved in his care and carefully documented his treatment history.

Mr. T has had numerous medication trials, including oral and long-acting injectable risperidone, olanzapine, aripiprazole, ziprasidone, lithium, gabapentin, buspirone, quetiapine, trazodone, bupropion, and paroxetine. None of these medications were effective.

In his mid-40s, Mr. T attempted suicide by wandering into traffic and being struck by a motor vehicle. A year later, he attempted suicide by driving his car at high speed into a concrete highway median. Mr. T told first responders that he was “possessed,” and a demonic entity “forced” him to crash his car. He begged law enforcement officers at the scene to give him a gun so he could shoot himself.

Continue to: Mr. T entered an intensive outpatient treatment program...

 

 

Mr. T entered an intensive outpatient treatment program and was switched from long-acting injectable risperidone to oral aripiprazole. After taking aripiprazole for several weeks, he began to gamble compulsively at a nearby casino. Frustrated by the lack of response to psychotropic medications and his idiosyncratic response to aripiprazole, he stopped psychiatric treatment, relapsed to alcohol use, and isolated himself in his apartment shortly before stabbing his mother.

EVALUATION Pharmacogenomics testing

At the mental health rehabilitation center, Mr. T agrees to undergo pharmacogenomics testing, which suggests that he will have a normal response to selective serotonin reuptake inhibitors and is unlikely to experience adverse reactions. He does not carry the 2 alleles that place him at higher risk of serious dermatologic reactions when taking certain mood stabilizers. He is heterozygous for the C677T allele polymorphism in the MTHFR gene that is associated with reduced folic acid metabolism, moderately decreased serum folate levels, and moderately increased homocysteine levels. On the pharmacokinetic genes tested, Mr. T has the normal metabolism genotype on 5 of 6 cytochrome P450 (CYP) enzymes; he has the ultrarapid metabolizer genotype on CYP1A2. He also has normal activity and intermediate metabolizer phenotype on the 2 UGT enzymes tested, which are responsible for the glucuronidation process, a major part of phase II metabolism.

Based on these results, Mr. T’s clozapine dosage is decreased by 50% (from 300 to 150 mg/d) and he is started on fluvoxamine, 50 mg/d, because it is a strong inhibitor of CYP1A2. The reduced conversion of clozapine to norclozapine results in an average serum clozapine level of 527 ng/mL (a level of 350 ng/mL is usually therapeutic in patients with schizophrenia) and norclozapine level of 140 ng/mL (clozapine:norclozapine ratio = 3.8), which is to be expected because fluvoxamine can increase serum clozapine levels.

Due to accumulating evidence in the literature suggesting that latent infections in the CNS play a role in serious mental illnesses such as schizophrenia, Mr. T undergoes further laboratory testing.

[polldaddy:10375845]

The authors’ observations

Mr. T tested positive for TG and CMV and negative for HSV-1. We were aware of accumulating evidence that latent infections in the CNS play a role in serious mental illnesses such as schizophrenia, specifically TG5—a parasite transmitted by cats—and CMV and HSV-1,6 which are transmitted by humans. The theory that TG infection could be a factor in schizophrenia emerged in the 1990s but only in recent years received mainstream scientific attention. Toxoplasma gondii, the infectious parasite that causes toxoplasmosis, infects more than 30 million people in the United States; however, most individuals are asymptomatic because of the body’s immune response to the parasite.7

Continue to: A study of 162 individuals...

 

 

A study of 162 individuals with schizophrenia, bipolar disorder, or major depressive disorder found that this immunologic profile is associated with suicide attempts,8 which is consistent with Mr. T’s history. Research suggests that individuals with schizophrenia who have latent TG infection have a more severe form of the illness compared with patients without the infection.9-12 Many of these factors were present in Mr. T’s case (Table 18-12).

Features of schizophrenia in patients with Toxoplasma gondii infection

TREATMENT Improvement, then setback

Mr. T’s medication regimen at the rehabilitation center includes clozapine, 100 mg/d; minocycline, 200 mg/d; fluvoxamine, 200 mg/d; and N-acetylcysteine, 1,200 mg/d. N-acetylcysteine is an antioxidant that could ease negative symptoms of schizophrenia by reducing oxidative stress caused by free radicals.13 Mr. T makes slow but steady improvement, and his IL-6 levels drop steadily (Figure 1).

Interleukin-6 (IL-6) levels over the course of Mr. T’s hospitalization

After 6 months in the rehabilitation center, Mr. T no longer experiences catatonic symptoms and is able to participate in the therapeutic program. He is permitted to leave the facility on day passes with family members. However, approximately every 8 weeks, he continues to cycle through periods of intense anxiety, perseverates on topics, and exhibits fragmented thinking and speech. During these episodes, he has difficulty receiving and processing information.

During one of these periods, Mr. T eats 4 oleander leaves he gathered while on day pass outside of the facility. After he experiences stomach pain, nausea, and vomiting, he informs nursing staff that he ate oleander. He is brought to the emergency department, receives activated charcoal and a digoxin antidote, and is placed on continuous electrocardiogram monitoring. When asked why he made the suicide attempt, he said “I realized things will never be the same because of what happened. I felt trapped.” He later expresses regret and wants to return to the mental health rehabilitation center.

At the facility, Mr. T agrees to take 2 more agents—valproic acid and ginger root extract—that specifically target latent toxoplasmosis infection before pursuing electroconvulsive therapy. We offer valproic acid because it inhibits replication of TG in an in vitro model.14 Mr. T is started on extended-release valproic acid, 1,500 mg/d, which results in a therapeutic serum level of 74.8 µg/mL.

Continue to: Additionally, Mr. T expresses interest...

 

 

Additionally, Mr. T expresses interest in taking “natural” agents in addition to psychotropics. After reviewing the quality of available ginger root extract products, Mr. T is started on a supplement that contains 22.4 mg of gingerols and 6.7 mg of shogaols, titrated to 4 capsules twice daily.

The authors’ observations

Psychotropic medications with anti-toxoplasmic activity

A retrospective cross-sectional analysis reported that patients with bipolar disorder who received medications with anti-toxoplasmic activity (Table 215), specifically valproic acid, had significantly fewer lifetime depressive episodes compared with patients who received medications without anti-toxoplasmic activity.15

 

Alternative medicine options

Research has demonstrated the beneficial effects of Chinese herbal plants for toxoplasmosis16,17 and ginger root extract has potent anti-toxoplasmic activity. A mouse model found that ginger root extract (Zingiber officinale) reduced the number of TG-infected cells by suppressing activation of apoptotic proteins the parasite induces, which prevents programmed cell death.18

Table 3 presents a stepwise approach to identifying and treating inflammation in patients with treatment-resistant psychosis.

A stepwise approach to identifying and treating inflammation in patients with treatment-resistant psychosis

OUTCOME Immune response, improvement

One month after the valproic acid and ginger root extract therapy is initiated, Mr. T’s toxoplasma antibody immunoglobulin G increases by 15.2 IU/mL, indicating that his immune system is mounting an enhanced response against the parasite (Figure 2). Mr. T continues to make progress while receiving the new regimen of clozapine, minocycline, valproic acid, and ginger root extract. He no longer cycles into periods of intense anxiety, perseverative thought, and fragmented thought and speech. He participates meaningfully in weekly psychotherapy and hopes to live independently and obtain gainful employment.

Toxoplasma antibody IgG titers before and after the addition of valproic acid and ginger root extract

The District Attorney’s office dismisses his criminal charges, and Mr. T is discharged to a less restrictive level of care.

Continue to: Bottom Line

 

 

Bottom Line

Several studies have shown that neuroinflammation increases the severity of mental illness. Consider adjunct anti-inflammatory agents for patients who have elevated levels of inflammatory biomarkers and for whom standard treatment approaches do not adequately control psychiatric symptoms. Also consider testing for the presence of latent infections in the CNS, which could reveal the underlying cause of treatment resistance or the genesis of disabling psychiatric symptoms.

Related Resources

  • Fond G, Macgregor A, Tamouza R, et al. Comparative analysis of anti-toxoplasmic activity of antipsychotic drugs and valproate. Eur Arch Psychiatry Clin Neurosci. 2014;264(2):179-183.
  • Hamdani N, Daban-Huard C, Lajnef M, et al. Cognitive deterioration among bipolar disorder patients infected by Toxoplasma gondii is correlated to interleukin 6 levels. J Affect Disord. 2015;179:161-166.
  • Monroe JM, Buckley PF, Miller BJ. Meta-analysis of antitoxoplasma gondii IgM antibodies in acute psychosis. Schizophr Bull. 2015;41(4):989-998.

Drug Brand Names

Acyclovir • Zovirax
Aripiprazole • Abilify
Bupropion • Wellbutrin
Buspirone • Buspar
Clozapine • Clozaril
Duloxetine • Cymbalta
Fluphenazine • Prolixin
Fluvoxamine • Luvox
Gabapentin • Neurontin
Haloperidol • Haldol
Lithium • Eskalith, Lithobid
Lorazepam • Ativan
Loxapine • Loxitane
Minocycline • Minocin
Olanzapine • Zyprexa
Paliperidone • Invega
Paroxetine • Paxil
Quetiapine • Seroquel
Risperidone • Risperdal, Risperdal Consta
Thioridazine • Mellaril
Trifluoperazine • Stelazine
Trazodone • Desyrel
Valproic acid • Depakote
Ziprasidone • Geodon

CASE Violent, then catatonic

Mr. T, age 52, has a long history of schizo­affective disorder, depressed type; several suicide attempts; and violent episodes. He is admitted to a mental health rehabilitation center under a forensic commitment.

Several years earlier, Mr. T had been charged with first-degree attempted murder, assault with a deadly weapon, and abuse of a dependent/geriatric adult after allegedly stabbing his mother in the upper chest and neck. At that time, Mr. T was not in psychiatric treatment and was drinking heavily. He had become obsessed with John F. Kennedy’s assassination and believed the Central Intelligence Agency (CIA), not Lee Harvey Oswald, was responsible. He feared the CIA wanted to kill him because of his knowledge, and he heard voices from his television he believed were threatening him. He acquired knives for self-protection. When his mother arrived at his apartment to take him to a psychiatric appointment, he believed she was conspiring with the CIA and attacked her. Mr. T’s mother survived her injuries. He was taken to the county jail, where psychiatric staff noted that Mr. T was psychotic.

The court found Mr. T incompetent to stand trial and sent him to a state hospital for psychiatric treatment and competency restoration. After 3 years, he was declared unable to be restored because of repeated decompensations, placed on a conservatorship, and sent back to county jail.

In the jail, Mr. T began to show signs of catatonia. He refused medications, food, and water, and became mute. He was admitted to a medical center after a 45-minute episode that appeared similar to a seizure; however, all laboratory evaluations were within normal limits, head CT was negative, and an EEG was unremarkable.

Mr. T’s catatonic state gradually resolved with increasing dosages of lorazepam, as well as clozapine. He showed improved mobility and oral intake. A month later, his train of thought was rambling and difficult to follow, circumstantial, and perseverating. However, at times he could be directed and respond to questions in a linear and logical fashion. Lorazepam was tapered, discontinued, and replaced with gabapentin because Mr. T viewed taking lorazepam as a threat to his sobriety.

Recently, Mr. T was transferred to our mental health rehabilitation center, where he expresses that he is grateful to be in a therapeutic environment. Upon admission, his medication regimen consists of clozapine, 300 mg by mouth at bedtime, duloxetine, 60 mg/d by mouth, gabapentin 600 mg by mouth 3 times a day, and docusate sodium, 250 mg/d by mouth. Our team has a discussion about the growing recognition of the pro-inflammatory state present in many patients who experience serious mental illness and the importance of augmenting standard evidence-based psychopharmacotherapy with agents that have neuroprotective properties.1,2 We offer Mr. T minocycline, 100 mg by mouth twice daily, a potent anti-inflammatory agent that has been shown to improve symptoms of schizophrenia.2 Mr. T is reluctant to take minocycline, saying he is happy with his current medication regimen.

[polldaddy:10375843]

The authors’ observations

Several studies have found that acute psychosis is associated with an inflammatory state, and interleukin-6 (IL-6) is a crucial biomarker. A recent meta-analysis of serum cytokines in patients with schizophrenia found that IL-6 levels were significantly increased among acutely ill patients compared with controls.3 IL-6 levels significantly decreased after treating acute episodes of schizophrenia.3 Further, levels of peripheral IL-6 mRNA levels in individuals with schizophrenia are directly correlated with severity of positive symptoms.4

Continue to: A meta-analyis reported...

 

 

A meta-analysis reported that tumor necrosis factor-alpha and IL-6 are elevated during acute psychosis3; however, IL-6 normalized with treatment, whereas tumor necrosis factor-alpha did not. This means that IL-6 is a more clinically meaningful biomarker to help gauge treatment response.

EVALUATION Elevated markers of inflammation

Laboratory testing reveals that Mr. T’s IL-6 level is 56.64 pg/mL, which is significantly elevated (reference range: 0.31 to 5.00 pg/mL). After reviewing the IL-6 results with Mr. T and explaining that there is “too much inflammation” in his brain, he agrees to take minocycline and complete follow-up IL-6 level tests to monitor his progress during treatment.

HISTORY Alcohol abuse, treatment resistance

According to Mr. T’s mother, he had met all developmental milestones and graduated from high school with plans to enter culinary school. At age 20, Mr. T began to experience psychotic symptoms, telling family members that he was being followed by FBI agents and was receiving messages from televisions. He began drinking heavily and was arrested twice for driving under the influence. In his mid-20s, he attempted suicide by overdose after his father died. Mr. T required inpatient hospitalization nearly every year thereafter. His mother, a registered nurse, was significantly involved in his care and carefully documented his treatment history.

Mr. T has had numerous medication trials, including oral and long-acting injectable risperidone, olanzapine, aripiprazole, ziprasidone, lithium, gabapentin, buspirone, quetiapine, trazodone, bupropion, and paroxetine. None of these medications were effective.

In his mid-40s, Mr. T attempted suicide by wandering into traffic and being struck by a motor vehicle. A year later, he attempted suicide by driving his car at high speed into a concrete highway median. Mr. T told first responders that he was “possessed,” and a demonic entity “forced” him to crash his car. He begged law enforcement officers at the scene to give him a gun so he could shoot himself.

Continue to: Mr. T entered an intensive outpatient treatment program...

 

 

Mr. T entered an intensive outpatient treatment program and was switched from long-acting injectable risperidone to oral aripiprazole. After taking aripiprazole for several weeks, he began to gamble compulsively at a nearby casino. Frustrated by the lack of response to psychotropic medications and his idiosyncratic response to aripiprazole, he stopped psychiatric treatment, relapsed to alcohol use, and isolated himself in his apartment shortly before stabbing his mother.

EVALUATION Pharmacogenomics testing

At the mental health rehabilitation center, Mr. T agrees to undergo pharmacogenomics testing, which suggests that he will have a normal response to selective serotonin reuptake inhibitors and is unlikely to experience adverse reactions. He does not carry the 2 alleles that place him at higher risk of serious dermatologic reactions when taking certain mood stabilizers. He is heterozygous for the C677T allele polymorphism in the MTHFR gene that is associated with reduced folic acid metabolism, moderately decreased serum folate levels, and moderately increased homocysteine levels. On the pharmacokinetic genes tested, Mr. T has the normal metabolism genotype on 5 of 6 cytochrome P450 (CYP) enzymes; he has the ultrarapid metabolizer genotype on CYP1A2. He also has normal activity and intermediate metabolizer phenotype on the 2 UGT enzymes tested, which are responsible for the glucuronidation process, a major part of phase II metabolism.

Based on these results, Mr. T’s clozapine dosage is decreased by 50% (from 300 to 150 mg/d) and he is started on fluvoxamine, 50 mg/d, because it is a strong inhibitor of CYP1A2. The reduced conversion of clozapine to norclozapine results in an average serum clozapine level of 527 ng/mL (a level of 350 ng/mL is usually therapeutic in patients with schizophrenia) and norclozapine level of 140 ng/mL (clozapine:norclozapine ratio = 3.8), which is to be expected because fluvoxamine can increase serum clozapine levels.

Due to accumulating evidence in the literature suggesting that latent infections in the CNS play a role in serious mental illnesses such as schizophrenia, Mr. T undergoes further laboratory testing.

[polldaddy:10375845]

The authors’ observations

Mr. T tested positive for TG and CMV and negative for HSV-1. We were aware of accumulating evidence that latent infections in the CNS play a role in serious mental illnesses such as schizophrenia, specifically TG5—a parasite transmitted by cats—and CMV and HSV-1,6 which are transmitted by humans. The theory that TG infection could be a factor in schizophrenia emerged in the 1990s but only in recent years received mainstream scientific attention. Toxoplasma gondii, the infectious parasite that causes toxoplasmosis, infects more than 30 million people in the United States; however, most individuals are asymptomatic because of the body’s immune response to the parasite.7

Continue to: A study of 162 individuals...

 

 

A study of 162 individuals with schizophrenia, bipolar disorder, or major depressive disorder found that this immunologic profile is associated with suicide attempts,8 which is consistent with Mr. T’s history. Research suggests that individuals with schizophrenia who have latent TG infection have a more severe form of the illness compared with patients without the infection.9-12 Many of these factors were present in Mr. T’s case (Table 18-12).

Features of schizophrenia in patients with Toxoplasma gondii infection

TREATMENT Improvement, then setback

Mr. T’s medication regimen at the rehabilitation center includes clozapine, 100 mg/d; minocycline, 200 mg/d; fluvoxamine, 200 mg/d; and N-acetylcysteine, 1,200 mg/d. N-acetylcysteine is an antioxidant that could ease negative symptoms of schizophrenia by reducing oxidative stress caused by free radicals.13 Mr. T makes slow but steady improvement, and his IL-6 levels drop steadily (Figure 1).

Interleukin-6 (IL-6) levels over the course of Mr. T’s hospitalization

After 6 months in the rehabilitation center, Mr. T no longer experiences catatonic symptoms and is able to participate in the therapeutic program. He is permitted to leave the facility on day passes with family members. However, approximately every 8 weeks, he continues to cycle through periods of intense anxiety, perseverates on topics, and exhibits fragmented thinking and speech. During these episodes, he has difficulty receiving and processing information.

During one of these periods, Mr. T eats 4 oleander leaves he gathered while on day pass outside of the facility. After he experiences stomach pain, nausea, and vomiting, he informs nursing staff that he ate oleander. He is brought to the emergency department, receives activated charcoal and a digoxin antidote, and is placed on continuous electrocardiogram monitoring. When asked why he made the suicide attempt, he said “I realized things will never be the same because of what happened. I felt trapped.” He later expresses regret and wants to return to the mental health rehabilitation center.

At the facility, Mr. T agrees to take 2 more agents—valproic acid and ginger root extract—that specifically target latent toxoplasmosis infection before pursuing electroconvulsive therapy. We offer valproic acid because it inhibits replication of TG in an in vitro model.14 Mr. T is started on extended-release valproic acid, 1,500 mg/d, which results in a therapeutic serum level of 74.8 µg/mL.

Continue to: Additionally, Mr. T expresses interest...

 

 

Additionally, Mr. T expresses interest in taking “natural” agents in addition to psychotropics. After reviewing the quality of available ginger root extract products, Mr. T is started on a supplement that contains 22.4 mg of gingerols and 6.7 mg of shogaols, titrated to 4 capsules twice daily.

The authors’ observations

Psychotropic medications with anti-toxoplasmic activity

A retrospective cross-sectional analysis reported that patients with bipolar disorder who received medications with anti-toxoplasmic activity (Table 215), specifically valproic acid, had significantly fewer lifetime depressive episodes compared with patients who received medications without anti-toxoplasmic activity.15

 

Alternative medicine options

Research has demonstrated the beneficial effects of Chinese herbal plants for toxoplasmosis16,17 and ginger root extract has potent anti-toxoplasmic activity. A mouse model found that ginger root extract (Zingiber officinale) reduced the number of TG-infected cells by suppressing activation of apoptotic proteins the parasite induces, which prevents programmed cell death.18

Table 3 presents a stepwise approach to identifying and treating inflammation in patients with treatment-resistant psychosis.

A stepwise approach to identifying and treating inflammation in patients with treatment-resistant psychosis

OUTCOME Immune response, improvement

One month after the valproic acid and ginger root extract therapy is initiated, Mr. T’s toxoplasma antibody immunoglobulin G increases by 15.2 IU/mL, indicating that his immune system is mounting an enhanced response against the parasite (Figure 2). Mr. T continues to make progress while receiving the new regimen of clozapine, minocycline, valproic acid, and ginger root extract. He no longer cycles into periods of intense anxiety, perseverative thought, and fragmented thought and speech. He participates meaningfully in weekly psychotherapy and hopes to live independently and obtain gainful employment.

Toxoplasma antibody IgG titers before and after the addition of valproic acid and ginger root extract

The District Attorney’s office dismisses his criminal charges, and Mr. T is discharged to a less restrictive level of care.

Continue to: Bottom Line

 

 

Bottom Line

Several studies have shown that neuroinflammation increases the severity of mental illness. Consider adjunct anti-inflammatory agents for patients who have elevated levels of inflammatory biomarkers and for whom standard treatment approaches do not adequately control psychiatric symptoms. Also consider testing for the presence of latent infections in the CNS, which could reveal the underlying cause of treatment resistance or the genesis of disabling psychiatric symptoms.

Related Resources

  • Fond G, Macgregor A, Tamouza R, et al. Comparative analysis of anti-toxoplasmic activity of antipsychotic drugs and valproate. Eur Arch Psychiatry Clin Neurosci. 2014;264(2):179-183.
  • Hamdani N, Daban-Huard C, Lajnef M, et al. Cognitive deterioration among bipolar disorder patients infected by Toxoplasma gondii is correlated to interleukin 6 levels. J Affect Disord. 2015;179:161-166.
  • Monroe JM, Buckley PF, Miller BJ. Meta-analysis of antitoxoplasma gondii IgM antibodies in acute psychosis. Schizophr Bull. 2015;41(4):989-998.

Drug Brand Names

Acyclovir • Zovirax
Aripiprazole • Abilify
Bupropion • Wellbutrin
Buspirone • Buspar
Clozapine • Clozaril
Duloxetine • Cymbalta
Fluphenazine • Prolixin
Fluvoxamine • Luvox
Gabapentin • Neurontin
Haloperidol • Haldol
Lithium • Eskalith, Lithobid
Lorazepam • Ativan
Loxapine • Loxitane
Minocycline • Minocin
Olanzapine • Zyprexa
Paliperidone • Invega
Paroxetine • Paxil
Quetiapine • Seroquel
Risperidone • Risperdal, Risperdal Consta
Thioridazine • Mellaril
Trifluoperazine • Stelazine
Trazodone • Desyrel
Valproic acid • Depakote
Ziprasidone • Geodon

References

1. Koola MM, Raines JK, Hamilton RG, et al. Can anti-inflammatory medications improve symptoms and reduce mortality in schizophrenia? Current Psychiatry. 2016;15(5):52-57.
2. Nasrallah HA. Are you neuroprotecting your patients? 10 Adjunctive therapies to consider. Current Psychiatry. 2016;15(12):12-14.
3. Goldsmith DR, Rapaport MH, Miller BJ. A meta-analysis of blood cytokine network alterations in psychiatric patients: comparisons between schizophrenia, bipolar disorder and depression. Mol Psychiatry. 2016;21(12):1696-1709.
4. Chase KA, Cone JJ, Rosen C, et al. The value of interleukin 6 as a peripheral diagnostic marker in schizophrenia. BMC Psychiatry. 2016;16:152.
5. Torrey EF, Bartko JJ, Lun ZR, et al. Antibodies to Toxoplasma gondii in patients with schizophrenia: a meta-analysis. Schizophr Bull. 2007;33(3):729-736.
6. Shirts BH, Prasad KM, Pogue-Geile MF, et al. Antibodies to cytomegalovirus and herpes simplex virus 1 associated with cognitive function in schizophrenia. Schizophr Res. 2008;106(2-3):268-274.
7. Centers for Disease Control and Prevention. Parasites - Toxoplasmosis (Toxoplasma infection). https://www.cdc.gov/parasites/toxoplasmosis/index.html. Accessed February 26, 2019.
8. Dickerson F, Wilcox HC, Adamos M, et al. Suicide attempts and markers of immune response in individuals with serious mental illness. J Psychiatr Res. 2017;87:37-43.
9. Celik T, Kartalci S, Aytas O, et al. Association between latent toxoplasmosis and clinical course of schizophrenia - continuous course of the disease is characteristic for Toxoplasma gondii-infected patients. Folia Parasitol (Praha). 2015;62. doi: 10.14411/fp.2015.015.
10. Dickerson F, Boronow J, Stallings C, et al. Toxoplasma gondii in individuals with schizophrenia: association with clinical and demographic factors and with mortality. Schizophr Bull. 2007;33(3):737-740.
11. Esshili A, Thabet S, Jemli A, et al. Toxoplasma gondii infection in schizophrenia and associated clinical features. Psychiatry Res. 2016;245:327-332.
12. Holub D, Flegr J, Dragomirecka E, et al. Differences in onset of disease and severity of psychopathology between toxoplasmosis-related and toxoplasmosis-unrelated schizophrenia. Acta Psychiatr Scand. 2013;127(3):227-238.
13. Chen AT, Chibnall JT, Nasrallah HA. Placebo-controlled augmentation trials of the antioxidant NAC in schizophrenia: a review. Ann Clin Psychiatry. 2016;28(3):190-196.

14. Jones-Brando L, Torrey EF, Yolken R. Drugs used in the treatment of schizophrenia and bipolar disorder inhibit the replication of Toxoplasma gondii. Schizophr Res. 2003;62(3):237-244.
15. Fond G, Boyer L, Gaman A, et al. Treatment with anti-toxoplasmic activity (TATA) for toxoplasma positive patients with bipolar disorders or schizophrenia: a cross-sectional study. J Psychiatr Res. 2015;63:58-64.
16. Wei HX, Wei SS, Lindsay DS, et al. A systematic review and meta-analysis of the efficacy of anti-Toxoplasma gondii medicines in humans. PLoS One. 2015;10(9):e0138204.
17. Zhuo XH, Sun HC, Huang B, et al. Evaluation of potential anti-toxoplasmosis efficiency of combined traditional herbs in a mouse model. J Zhejiang Univ Sci B. 2017;18(6):453-461.
18. Choi WH, Jiang MH, Chu JP. Antiparasitic effects of Zingiber officinale (Ginger) extract against Toxoplasma gondii. Journal of Applied Biomedicine. 2013;11:15-26.

References

1. Koola MM, Raines JK, Hamilton RG, et al. Can anti-inflammatory medications improve symptoms and reduce mortality in schizophrenia? Current Psychiatry. 2016;15(5):52-57.
2. Nasrallah HA. Are you neuroprotecting your patients? 10 Adjunctive therapies to consider. Current Psychiatry. 2016;15(12):12-14.
3. Goldsmith DR, Rapaport MH, Miller BJ. A meta-analysis of blood cytokine network alterations in psychiatric patients: comparisons between schizophrenia, bipolar disorder and depression. Mol Psychiatry. 2016;21(12):1696-1709.
4. Chase KA, Cone JJ, Rosen C, et al. The value of interleukin 6 as a peripheral diagnostic marker in schizophrenia. BMC Psychiatry. 2016;16:152.
5. Torrey EF, Bartko JJ, Lun ZR, et al. Antibodies to Toxoplasma gondii in patients with schizophrenia: a meta-analysis. Schizophr Bull. 2007;33(3):729-736.
6. Shirts BH, Prasad KM, Pogue-Geile MF, et al. Antibodies to cytomegalovirus and herpes simplex virus 1 associated with cognitive function in schizophrenia. Schizophr Res. 2008;106(2-3):268-274.
7. Centers for Disease Control and Prevention. Parasites - Toxoplasmosis (Toxoplasma infection). https://www.cdc.gov/parasites/toxoplasmosis/index.html. Accessed February 26, 2019.
8. Dickerson F, Wilcox HC, Adamos M, et al. Suicide attempts and markers of immune response in individuals with serious mental illness. J Psychiatr Res. 2017;87:37-43.
9. Celik T, Kartalci S, Aytas O, et al. Association between latent toxoplasmosis and clinical course of schizophrenia - continuous course of the disease is characteristic for Toxoplasma gondii-infected patients. Folia Parasitol (Praha). 2015;62. doi: 10.14411/fp.2015.015.
10. Dickerson F, Boronow J, Stallings C, et al. Toxoplasma gondii in individuals with schizophrenia: association with clinical and demographic factors and with mortality. Schizophr Bull. 2007;33(3):737-740.
11. Esshili A, Thabet S, Jemli A, et al. Toxoplasma gondii infection in schizophrenia and associated clinical features. Psychiatry Res. 2016;245:327-332.
12. Holub D, Flegr J, Dragomirecka E, et al. Differences in onset of disease and severity of psychopathology between toxoplasmosis-related and toxoplasmosis-unrelated schizophrenia. Acta Psychiatr Scand. 2013;127(3):227-238.
13. Chen AT, Chibnall JT, Nasrallah HA. Placebo-controlled augmentation trials of the antioxidant NAC in schizophrenia: a review. Ann Clin Psychiatry. 2016;28(3):190-196.

14. Jones-Brando L, Torrey EF, Yolken R. Drugs used in the treatment of schizophrenia and bipolar disorder inhibit the replication of Toxoplasma gondii. Schizophr Res. 2003;62(3):237-244.
15. Fond G, Boyer L, Gaman A, et al. Treatment with anti-toxoplasmic activity (TATA) for toxoplasma positive patients with bipolar disorders or schizophrenia: a cross-sectional study. J Psychiatr Res. 2015;63:58-64.
16. Wei HX, Wei SS, Lindsay DS, et al. A systematic review and meta-analysis of the efficacy of anti-Toxoplasma gondii medicines in humans. PLoS One. 2015;10(9):e0138204.
17. Zhuo XH, Sun HC, Huang B, et al. Evaluation of potential anti-toxoplasmosis efficiency of combined traditional herbs in a mouse model. J Zhejiang Univ Sci B. 2017;18(6):453-461.
18. Choi WH, Jiang MH, Chu JP. Antiparasitic effects of Zingiber officinale (Ginger) extract against Toxoplasma gondii. Journal of Applied Biomedicine. 2013;11:15-26.

Issue
Current Psychiatry - 18(8)
Issue
Current Psychiatry - 18(8)
Page Number
e3-e9
Page Number
e3-e9
Publications
Publications
Topics
Article Type
Display Headline
Suicidal, violent, and treatment-resistant
Display Headline
Suicidal, violent, and treatment-resistant
Sections
Disallow All Ads
Content Gating
No Gating (article Unlocked/Free)
Alternative CME
Disqus Comments
Default
Use ProPublica
Hide sidebar & use full width
render the right sidebar.
Article PDF Media

Beyond ‘selfies’: An epidemic of acquired narcissism

Article Type
Changed
Thu, 08/01/2019 - 13:56
Display Headline
Beyond ‘selfies’: An epidemic of acquired narcissism

Narcissism has an evil reputation. But is it justified? A modicum of narcissism is actually healthy. It can bolster self-confidence, assertiveness, and success in business and in the sociobiology of mating. Perhaps that’s why narcissism as a trait has a survival value from an evolutionary perspective.

Taking an excessive number of “selfies” with a smartphone is probably the most common and relatively benign form of mild narcissism (and not in DSM-5, yet). Narcissistic personality disorder (NPD), with a prevalence of 1%, is on the extreme end of the narcissism continuum. It has become tainted with such an intensely negative halo that it has become a despised trait, an insult, and even a vile epithet, like a 4-letter word. But as psychiatrists and other mental health professionals, we clinically relate to patients with NPD as being afflicted with a serious neuropsychiatric disorder, not as despicable individuals. Many people outside the mental health profession abhor persons with NPD because of their gargantuan hubris, insufferable selfishness, self-aggrandizement, emotional abuse of others, and irremediable vanity. Narcissistic personality disorder deprives its sufferers of the prosocial capacity for empathy, which leads them to belittle others or treat competent individuals with disdain, never as equals. They also seem to be incapable of experiencing shame as they inflate their self-importance and megalomania at the expense of those they degrade. They cannot tolerate any success by others because it threatens to overshadow their own exaggerated achievements. They can be mercilessly harsh towards their underlings. They are incapable of fostering warm, long-term loving relationships, where bidirectional respect is essential. Their lives often are replete with brief, broken-up relationships because they emotionally, physically, or sexually abuse their intimate partners.

Primary NPD has been shown in twin studies to be highly genetic, and more strongly heritable than 17 other personality dimensions.1 It is also resistant to any effective psychotherapeutic, pharmacologic, or somatic treatments. This is particularly relevant given the proclivity of individuals with NPD to experience a crushing disappointment, commonly known as “narcissistic injury,” following a real or imagined failure. This could lead to a painful depression or an outburst of “narcissistic rage” directed at anyone perceived as undermining them, and may even lead to violent behavior.2

Apart from heritable narcissism, there is also another form of narcissism that can develop in some individuals following life events. That hazardous condition, known as “acquired narcissism,” is most often associated with achieving the coveted status of an exalted celebrity. At risk for this acquired personality affliction are famous actors, singers, movie directors, TV anchors, or politicians (although some politicians are natural-born narcissists, driven to seek the powers of public office), and less frequently physicians (perhaps because the practice of medicine is not done in front of spectators) or scientists (because research, no matter how momentous, rarely procures the glamour or public adulation of the entertainment industry). The ardent fans of those “celebs” shower them with such intense attention and adulation that it malignantly transforms previously “normal” individuals into narcissists who start believing they are indeed “very special” and superior to the rest of us mortals (especially as their earning power balloons into the millions after growing up with humble social or economic roots).

Social media has become a catalyst for acquired narcissism, with millions of followers on Twitter, Facebook, or YouTube. Cable TV also caters to politicians, some of whom morph into narcissists, intoxicated with their newfound eminence and stature among their partisan followers, and become genuinely convinced that they have supreme power or influence over the masses. They get carried away with their own exaggerated self-importance as oracles of the “truth,” regardless of how extreme their views may be. Celebrity, politics, social media, and cable TV have converged into a combustible mix, a crucible for acquired narcissism.

An interesting feature of acquired narcissism is “collective narcissism,” in which celebrities coalesce to consolidate their imagined superhuman attributes that go beyond the technical skills of their professions such as acting, singing, sports, or politics. Thus, entertainers or star athletes believe they can enunciate radical statements about contemporary social, political, or environmental issues (at both ends of the debate) as though their artistic success renders them wise arbiters of the truth. What complicates matters is their delirious fans, who revere and mimic whatever their idols say (and their fashion or their tattoos), which further intensifies the grandiosity and megalomania of acquired narcissism. Celebrity triggers mindless idolatry, fueling the narcissism of individuals who are blessed (or cursed?) with runaway personal success. Neuroscientists should conduct research into how the brain is neurobiologically altered by fame, but there are many more urgent questions that demand their attention. It would be important to know if it is reversible or enduring, even as fame inevitably dims.

Continue to: The pursuit of wealth and fame...

 

 

The pursuit of wealth and fame is widely prevalent and can be healthy if it is not all-consuming. But if achieved beyond the aspirer’s wildest dreams, he/she may reach an inflection point conducive to a pathologic degree of acquired narcissism. That’s what the French refer to as “les risques du métier” (ie, occupational hazard). I recall reading about celebrities who became enraged when a policeman “dared” to stop their car for some driving violation, confronting the officer with “Do you know who I am?” That question may be a clinical biomarker of acquired narcissism.

Interestingly, several years ago, when the American Psychiatry Association last revised the DSM—sometimes referred to as the “bible” of psychiatric nosology—it came close to dropping NPD from its listed disorders, but then reverted and kept it as one of the 275 diagnostic categories included in DSM-5.3 Had the NPD diagnosis been discarded, one wonders if the mythical god of narcissism would have suffered a transcendental “narcissistic injury”…

References

1. Livesley WJ, Jang KL, Jackson DN, et al. Genetic and environmental contributions to dimensions of personality disorder. Am J Psychiatry. 1993;150(12):1826-1831
2. Malmquist CP. Homicide: a psychiatric perspective. Washington, DC: American Psychiatric Publishing, Inc.; 2006:181-182.
3. Diagnostic and statistical manual of mental disorders, 5th ed. Washington, DC: American Psychiatric Association; 2013.

Article PDF
Author and Disclosure Information

Henry A. Nasrallah, MD
Editor-in-Chief

Issue
Current Psychiatry - 18(8)
Publications
Topics
Page Number
7-8,41
Sections
Author and Disclosure Information

Henry A. Nasrallah, MD
Editor-in-Chief

Author and Disclosure Information

Henry A. Nasrallah, MD
Editor-in-Chief

Article PDF
Article PDF

Narcissism has an evil reputation. But is it justified? A modicum of narcissism is actually healthy. It can bolster self-confidence, assertiveness, and success in business and in the sociobiology of mating. Perhaps that’s why narcissism as a trait has a survival value from an evolutionary perspective.

Taking an excessive number of “selfies” with a smartphone is probably the most common and relatively benign form of mild narcissism (and not in DSM-5, yet). Narcissistic personality disorder (NPD), with a prevalence of 1%, is on the extreme end of the narcissism continuum. It has become tainted with such an intensely negative halo that it has become a despised trait, an insult, and even a vile epithet, like a 4-letter word. But as psychiatrists and other mental health professionals, we clinically relate to patients with NPD as being afflicted with a serious neuropsychiatric disorder, not as despicable individuals. Many people outside the mental health profession abhor persons with NPD because of their gargantuan hubris, insufferable selfishness, self-aggrandizement, emotional abuse of others, and irremediable vanity. Narcissistic personality disorder deprives its sufferers of the prosocial capacity for empathy, which leads them to belittle others or treat competent individuals with disdain, never as equals. They also seem to be incapable of experiencing shame as they inflate their self-importance and megalomania at the expense of those they degrade. They cannot tolerate any success by others because it threatens to overshadow their own exaggerated achievements. They can be mercilessly harsh towards their underlings. They are incapable of fostering warm, long-term loving relationships, where bidirectional respect is essential. Their lives often are replete with brief, broken-up relationships because they emotionally, physically, or sexually abuse their intimate partners.

Primary NPD has been shown in twin studies to be highly genetic, and more strongly heritable than 17 other personality dimensions.1 It is also resistant to any effective psychotherapeutic, pharmacologic, or somatic treatments. This is particularly relevant given the proclivity of individuals with NPD to experience a crushing disappointment, commonly known as “narcissistic injury,” following a real or imagined failure. This could lead to a painful depression or an outburst of “narcissistic rage” directed at anyone perceived as undermining them, and may even lead to violent behavior.2

Apart from heritable narcissism, there is also another form of narcissism that can develop in some individuals following life events. That hazardous condition, known as “acquired narcissism,” is most often associated with achieving the coveted status of an exalted celebrity. At risk for this acquired personality affliction are famous actors, singers, movie directors, TV anchors, or politicians (although some politicians are natural-born narcissists, driven to seek the powers of public office), and less frequently physicians (perhaps because the practice of medicine is not done in front of spectators) or scientists (because research, no matter how momentous, rarely procures the glamour or public adulation of the entertainment industry). The ardent fans of those “celebs” shower them with such intense attention and adulation that it malignantly transforms previously “normal” individuals into narcissists who start believing they are indeed “very special” and superior to the rest of us mortals (especially as their earning power balloons into the millions after growing up with humble social or economic roots).

Social media has become a catalyst for acquired narcissism, with millions of followers on Twitter, Facebook, or YouTube. Cable TV also caters to politicians, some of whom morph into narcissists, intoxicated with their newfound eminence and stature among their partisan followers, and become genuinely convinced that they have supreme power or influence over the masses. They get carried away with their own exaggerated self-importance as oracles of the “truth,” regardless of how extreme their views may be. Celebrity, politics, social media, and cable TV have converged into a combustible mix, a crucible for acquired narcissism.

An interesting feature of acquired narcissism is “collective narcissism,” in which celebrities coalesce to consolidate their imagined superhuman attributes that go beyond the technical skills of their professions such as acting, singing, sports, or politics. Thus, entertainers or star athletes believe they can enunciate radical statements about contemporary social, political, or environmental issues (at both ends of the debate) as though their artistic success renders them wise arbiters of the truth. What complicates matters is their delirious fans, who revere and mimic whatever their idols say (and their fashion or their tattoos), which further intensifies the grandiosity and megalomania of acquired narcissism. Celebrity triggers mindless idolatry, fueling the narcissism of individuals who are blessed (or cursed?) with runaway personal success. Neuroscientists should conduct research into how the brain is neurobiologically altered by fame, but there are many more urgent questions that demand their attention. It would be important to know if it is reversible or enduring, even as fame inevitably dims.

Continue to: The pursuit of wealth and fame...

 

 

The pursuit of wealth and fame is widely prevalent and can be healthy if it is not all-consuming. But if achieved beyond the aspirer’s wildest dreams, he/she may reach an inflection point conducive to a pathologic degree of acquired narcissism. That’s what the French refer to as “les risques du métier” (ie, occupational hazard). I recall reading about celebrities who became enraged when a policeman “dared” to stop their car for some driving violation, confronting the officer with “Do you know who I am?” That question may be a clinical biomarker of acquired narcissism.

Interestingly, several years ago, when the American Psychiatry Association last revised the DSM—sometimes referred to as the “bible” of psychiatric nosology—it came close to dropping NPD from its listed disorders, but then reverted and kept it as one of the 275 diagnostic categories included in DSM-5.3 Had the NPD diagnosis been discarded, one wonders if the mythical god of narcissism would have suffered a transcendental “narcissistic injury”…

Narcissism has an evil reputation. But is it justified? A modicum of narcissism is actually healthy. It can bolster self-confidence, assertiveness, and success in business and in the sociobiology of mating. Perhaps that’s why narcissism as a trait has a survival value from an evolutionary perspective.

Taking an excessive number of “selfies” with a smartphone is probably the most common and relatively benign form of mild narcissism (and not in DSM-5, yet). Narcissistic personality disorder (NPD), with a prevalence of 1%, is on the extreme end of the narcissism continuum. It has become tainted with such an intensely negative halo that it has become a despised trait, an insult, and even a vile epithet, like a 4-letter word. But as psychiatrists and other mental health professionals, we clinically relate to patients with NPD as being afflicted with a serious neuropsychiatric disorder, not as despicable individuals. Many people outside the mental health profession abhor persons with NPD because of their gargantuan hubris, insufferable selfishness, self-aggrandizement, emotional abuse of others, and irremediable vanity. Narcissistic personality disorder deprives its sufferers of the prosocial capacity for empathy, which leads them to belittle others or treat competent individuals with disdain, never as equals. They also seem to be incapable of experiencing shame as they inflate their self-importance and megalomania at the expense of those they degrade. They cannot tolerate any success by others because it threatens to overshadow their own exaggerated achievements. They can be mercilessly harsh towards their underlings. They are incapable of fostering warm, long-term loving relationships, where bidirectional respect is essential. Their lives often are replete with brief, broken-up relationships because they emotionally, physically, or sexually abuse their intimate partners.

Primary NPD has been shown in twin studies to be highly genetic, and more strongly heritable than 17 other personality dimensions.1 It is also resistant to any effective psychotherapeutic, pharmacologic, or somatic treatments. This is particularly relevant given the proclivity of individuals with NPD to experience a crushing disappointment, commonly known as “narcissistic injury,” following a real or imagined failure. This could lead to a painful depression or an outburst of “narcissistic rage” directed at anyone perceived as undermining them, and may even lead to violent behavior.2

Apart from heritable narcissism, there is also another form of narcissism that can develop in some individuals following life events. That hazardous condition, known as “acquired narcissism,” is most often associated with achieving the coveted status of an exalted celebrity. At risk for this acquired personality affliction are famous actors, singers, movie directors, TV anchors, or politicians (although some politicians are natural-born narcissists, driven to seek the powers of public office), and less frequently physicians (perhaps because the practice of medicine is not done in front of spectators) or scientists (because research, no matter how momentous, rarely procures the glamour or public adulation of the entertainment industry). The ardent fans of those “celebs” shower them with such intense attention and adulation that it malignantly transforms previously “normal” individuals into narcissists who start believing they are indeed “very special” and superior to the rest of us mortals (especially as their earning power balloons into the millions after growing up with humble social or economic roots).

Social media has become a catalyst for acquired narcissism, with millions of followers on Twitter, Facebook, or YouTube. Cable TV also caters to politicians, some of whom morph into narcissists, intoxicated with their newfound eminence and stature among their partisan followers, and become genuinely convinced that they have supreme power or influence over the masses. They get carried away with their own exaggerated self-importance as oracles of the “truth,” regardless of how extreme their views may be. Celebrity, politics, social media, and cable TV have converged into a combustible mix, a crucible for acquired narcissism.

An interesting feature of acquired narcissism is “collective narcissism,” in which celebrities coalesce to consolidate their imagined superhuman attributes that go beyond the technical skills of their professions such as acting, singing, sports, or politics. Thus, entertainers or star athletes believe they can enunciate radical statements about contemporary social, political, or environmental issues (at both ends of the debate) as though their artistic success renders them wise arbiters of the truth. What complicates matters is their delirious fans, who revere and mimic whatever their idols say (and their fashion or their tattoos), which further intensifies the grandiosity and megalomania of acquired narcissism. Celebrity triggers mindless idolatry, fueling the narcissism of individuals who are blessed (or cursed?) with runaway personal success. Neuroscientists should conduct research into how the brain is neurobiologically altered by fame, but there are many more urgent questions that demand their attention. It would be important to know if it is reversible or enduring, even as fame inevitably dims.

Continue to: The pursuit of wealth and fame...

 

 

The pursuit of wealth and fame is widely prevalent and can be healthy if it is not all-consuming. But if achieved beyond the aspirer’s wildest dreams, he/she may reach an inflection point conducive to a pathologic degree of acquired narcissism. That’s what the French refer to as “les risques du métier” (ie, occupational hazard). I recall reading about celebrities who became enraged when a policeman “dared” to stop their car for some driving violation, confronting the officer with “Do you know who I am?” That question may be a clinical biomarker of acquired narcissism.

Interestingly, several years ago, when the American Psychiatry Association last revised the DSM—sometimes referred to as the “bible” of psychiatric nosology—it came close to dropping NPD from its listed disorders, but then reverted and kept it as one of the 275 diagnostic categories included in DSM-5.3 Had the NPD diagnosis been discarded, one wonders if the mythical god of narcissism would have suffered a transcendental “narcissistic injury”…

References

1. Livesley WJ, Jang KL, Jackson DN, et al. Genetic and environmental contributions to dimensions of personality disorder. Am J Psychiatry. 1993;150(12):1826-1831
2. Malmquist CP. Homicide: a psychiatric perspective. Washington, DC: American Psychiatric Publishing, Inc.; 2006:181-182.
3. Diagnostic and statistical manual of mental disorders, 5th ed. Washington, DC: American Psychiatric Association; 2013.

References

1. Livesley WJ, Jang KL, Jackson DN, et al. Genetic and environmental contributions to dimensions of personality disorder. Am J Psychiatry. 1993;150(12):1826-1831
2. Malmquist CP. Homicide: a psychiatric perspective. Washington, DC: American Psychiatric Publishing, Inc.; 2006:181-182.
3. Diagnostic and statistical manual of mental disorders, 5th ed. Washington, DC: American Psychiatric Association; 2013.

Issue
Current Psychiatry - 18(8)
Issue
Current Psychiatry - 18(8)
Page Number
7-8,41
Page Number
7-8,41
Publications
Publications
Topics
Article Type
Display Headline
Beyond ‘selfies’: An epidemic of acquired narcissism
Display Headline
Beyond ‘selfies’: An epidemic of acquired narcissism
Sections
Disallow All Ads
Content Gating
No Gating (article Unlocked/Free)
Alternative CME
Disqus Comments
Default
Use ProPublica
Hide sidebar & use full width
render the right sidebar.
Article PDF Media

Backlash against using rating scales

Article Type
Changed
Thu, 08/01/2019 - 13:59
Display Headline
Backlash against using rating scales

I strongly disagree with the editorial by Ahmed A. Aboraya, MD, DrPH, and Henry A. Nasrallah, MD, (“It’s time to implement measurement-based care in psychiatric practice,” From the Editor, Current Psychiatry. June 2019, p. 6-8). I am 76 years old and recently retired. I have seen many attempts to “objectify” medicine. These have all failed, but each has taken a piece of medicine with it to the grave.

We do not have much more to lose before it’s a checklist, vital signs, and a script. I now refer to our profession as “McMedicine.” If you don’t have what is on the menu, you cannot get served. Diseases are rarely treated, symptoms are treated. This is not the profession of medicine. We are not fixing much; we are mostly providing consumers for pharmaceutical companies.

Few psychiatric disorders have been subjected to more measurement than depression. Quite a while ago, someone tried to compare depression scales. They correlated scale scores with the results of evaluations by board-certified psychiatrists. The best scale was a single question: “Are you depressed?” This had been included as a control. Can you do better?

Furthermore, the “paper and numbers” people can’t wait to get an “objective” wrench to tighten the screws and apply the principles of the industrial revolution to squeeze more money out of the system. They will find some way to turn patients into standardized products.

John L. Schenkel, MD
Retired psychiatrist
Peru, NY

I disagree with Drs. Aboraya and Nasrallah regarding implementing rating scales in psychiatry. Frankly, medicine has become awash in details—mounds and mounds of details.

With the use of an electronic medical record, what should be a simple 1-page note is transformed into a 5-page note of details. Doctors no longer attend to their patients but rather to their computers. Has this raised consciousness—the most important metric, according to Dr. David Hawkins? I doubt it.

In the words of my great professor, Dr. James Gustafson, I will continue to start my interview with what concerns the patient. Most of the time, they implicitly know.

Our focus should instead be on bringing down the cost of health care. This is what angers our patients most, and yet we do not make it a priority.

Mike Primc, MD
Psychiatrist
Glenbeigh Hospital
Rock Creek, Ohio
Signature Health
Ashtabula, Ohio
Behavioral Wellness Group
Mentor, Ohio

Continue to: The authors respond

 

 

The authors respond

We appreciate Drs. Schenkel’s and Primc’s comments on our editorial regarding measurement-based care (MBC). However, MBC will not increase the workload of psychiatrists; rather, it will streamline the evaluation of patients and measure the severity of their symptoms or adverse effects as well as the degree of their improvement. The proper use of scales with the appropriate patient populations may actually help clinicians to reduce the extensive amount of details that go into medical records.

The following quote, an excerpt from another article we wrote on MBC,1 speaks to Dr. Primc’s concerns:

“…measures in psychiatry could be considered the equivalent of a thermometer and a stethoscope to a physician. No measure, scale, or diagnostic interview will ever replace a seasoned, experienced clinician who has been evaluating and treating real patients for years. MBC is not intended to replace clinical judgment and cannot substitute for an observant and caring clinician. Just as thermometers, stethoscopes, and lab tests help other types of physicians reach accurate diagnoses and provide appropriate management, the use of MBC by psychiatrists has the potential to improve the accuracy of diagnoses and improve the outcomes of care.”

Ahmed A. Aboraya, MD, DrPH
Assistant Professor
Department of Behavioral Medicine and Psychiatry
Chief of Psychiatry
Sharpe Hospital West Virginia University
Weston, West Virginia

Henry A. Nasrallah, MD
Professor of Psychiatry, Neurology, and Neuroscience
Medical Director: Neuropsychiatry
Director, Schizophrenia and Neuropsychiatry Programs
University of Cincinnati College of Medicine
Cincinnati, Ohio
Professor Emeritus, Saint Louis University
St. Louis, Missouri

Reference
1. Aboraya A, Nasrallah HA, Elswick DE, et al. Measurement-based care in psychiatry-past, present, and future. Innov Clin Neurosci. 2018;15(11-12):13-26.

Article PDF
Issue
Current Psychiatry - 18(8)
Publications
Page Number
9
Sections
Article PDF
Article PDF

I strongly disagree with the editorial by Ahmed A. Aboraya, MD, DrPH, and Henry A. Nasrallah, MD, (“It’s time to implement measurement-based care in psychiatric practice,” From the Editor, Current Psychiatry. June 2019, p. 6-8). I am 76 years old and recently retired. I have seen many attempts to “objectify” medicine. These have all failed, but each has taken a piece of medicine with it to the grave.

We do not have much more to lose before it’s a checklist, vital signs, and a script. I now refer to our profession as “McMedicine.” If you don’t have what is on the menu, you cannot get served. Diseases are rarely treated, symptoms are treated. This is not the profession of medicine. We are not fixing much; we are mostly providing consumers for pharmaceutical companies.

Few psychiatric disorders have been subjected to more measurement than depression. Quite a while ago, someone tried to compare depression scales. They correlated scale scores with the results of evaluations by board-certified psychiatrists. The best scale was a single question: “Are you depressed?” This had been included as a control. Can you do better?

Furthermore, the “paper and numbers” people can’t wait to get an “objective” wrench to tighten the screws and apply the principles of the industrial revolution to squeeze more money out of the system. They will find some way to turn patients into standardized products.

John L. Schenkel, MD
Retired psychiatrist
Peru, NY

I disagree with Drs. Aboraya and Nasrallah regarding implementing rating scales in psychiatry. Frankly, medicine has become awash in details—mounds and mounds of details.

With the use of an electronic medical record, what should be a simple 1-page note is transformed into a 5-page note of details. Doctors no longer attend to their patients but rather to their computers. Has this raised consciousness—the most important metric, according to Dr. David Hawkins? I doubt it.

In the words of my great professor, Dr. James Gustafson, I will continue to start my interview with what concerns the patient. Most of the time, they implicitly know.

Our focus should instead be on bringing down the cost of health care. This is what angers our patients most, and yet we do not make it a priority.

Mike Primc, MD
Psychiatrist
Glenbeigh Hospital
Rock Creek, Ohio
Signature Health
Ashtabula, Ohio
Behavioral Wellness Group
Mentor, Ohio

Continue to: The authors respond

 

 

The authors respond

We appreciate Drs. Schenkel’s and Primc’s comments on our editorial regarding measurement-based care (MBC). However, MBC will not increase the workload of psychiatrists; rather, it will streamline the evaluation of patients and measure the severity of their symptoms or adverse effects as well as the degree of their improvement. The proper use of scales with the appropriate patient populations may actually help clinicians to reduce the extensive amount of details that go into medical records.

The following quote, an excerpt from another article we wrote on MBC,1 speaks to Dr. Primc’s concerns:

“…measures in psychiatry could be considered the equivalent of a thermometer and a stethoscope to a physician. No measure, scale, or diagnostic interview will ever replace a seasoned, experienced clinician who has been evaluating and treating real patients for years. MBC is not intended to replace clinical judgment and cannot substitute for an observant and caring clinician. Just as thermometers, stethoscopes, and lab tests help other types of physicians reach accurate diagnoses and provide appropriate management, the use of MBC by psychiatrists has the potential to improve the accuracy of diagnoses and improve the outcomes of care.”

Ahmed A. Aboraya, MD, DrPH
Assistant Professor
Department of Behavioral Medicine and Psychiatry
Chief of Psychiatry
Sharpe Hospital West Virginia University
Weston, West Virginia

Henry A. Nasrallah, MD
Professor of Psychiatry, Neurology, and Neuroscience
Medical Director: Neuropsychiatry
Director, Schizophrenia and Neuropsychiatry Programs
University of Cincinnati College of Medicine
Cincinnati, Ohio
Professor Emeritus, Saint Louis University
St. Louis, Missouri

Reference
1. Aboraya A, Nasrallah HA, Elswick DE, et al. Measurement-based care in psychiatry-past, present, and future. Innov Clin Neurosci. 2018;15(11-12):13-26.

I strongly disagree with the editorial by Ahmed A. Aboraya, MD, DrPH, and Henry A. Nasrallah, MD, (“It’s time to implement measurement-based care in psychiatric practice,” From the Editor, Current Psychiatry. June 2019, p. 6-8). I am 76 years old and recently retired. I have seen many attempts to “objectify” medicine. These have all failed, but each has taken a piece of medicine with it to the grave.

We do not have much more to lose before it’s a checklist, vital signs, and a script. I now refer to our profession as “McMedicine.” If you don’t have what is on the menu, you cannot get served. Diseases are rarely treated, symptoms are treated. This is not the profession of medicine. We are not fixing much; we are mostly providing consumers for pharmaceutical companies.

Few psychiatric disorders have been subjected to more measurement than depression. Quite a while ago, someone tried to compare depression scales. They correlated scale scores with the results of evaluations by board-certified psychiatrists. The best scale was a single question: “Are you depressed?” This had been included as a control. Can you do better?

Furthermore, the “paper and numbers” people can’t wait to get an “objective” wrench to tighten the screws and apply the principles of the industrial revolution to squeeze more money out of the system. They will find some way to turn patients into standardized products.

John L. Schenkel, MD
Retired psychiatrist
Peru, NY

I disagree with Drs. Aboraya and Nasrallah regarding implementing rating scales in psychiatry. Frankly, medicine has become awash in details—mounds and mounds of details.

With the use of an electronic medical record, what should be a simple 1-page note is transformed into a 5-page note of details. Doctors no longer attend to their patients but rather to their computers. Has this raised consciousness—the most important metric, according to Dr. David Hawkins? I doubt it.

In the words of my great professor, Dr. James Gustafson, I will continue to start my interview with what concerns the patient. Most of the time, they implicitly know.

Our focus should instead be on bringing down the cost of health care. This is what angers our patients most, and yet we do not make it a priority.

Mike Primc, MD
Psychiatrist
Glenbeigh Hospital
Rock Creek, Ohio
Signature Health
Ashtabula, Ohio
Behavioral Wellness Group
Mentor, Ohio

Continue to: The authors respond

 

 

The authors respond

We appreciate Drs. Schenkel’s and Primc’s comments on our editorial regarding measurement-based care (MBC). However, MBC will not increase the workload of psychiatrists; rather, it will streamline the evaluation of patients and measure the severity of their symptoms or adverse effects as well as the degree of their improvement. The proper use of scales with the appropriate patient populations may actually help clinicians to reduce the extensive amount of details that go into medical records.

The following quote, an excerpt from another article we wrote on MBC,1 speaks to Dr. Primc’s concerns:

“…measures in psychiatry could be considered the equivalent of a thermometer and a stethoscope to a physician. No measure, scale, or diagnostic interview will ever replace a seasoned, experienced clinician who has been evaluating and treating real patients for years. MBC is not intended to replace clinical judgment and cannot substitute for an observant and caring clinician. Just as thermometers, stethoscopes, and lab tests help other types of physicians reach accurate diagnoses and provide appropriate management, the use of MBC by psychiatrists has the potential to improve the accuracy of diagnoses and improve the outcomes of care.”

Ahmed A. Aboraya, MD, DrPH
Assistant Professor
Department of Behavioral Medicine and Psychiatry
Chief of Psychiatry
Sharpe Hospital West Virginia University
Weston, West Virginia

Henry A. Nasrallah, MD
Professor of Psychiatry, Neurology, and Neuroscience
Medical Director: Neuropsychiatry
Director, Schizophrenia and Neuropsychiatry Programs
University of Cincinnati College of Medicine
Cincinnati, Ohio
Professor Emeritus, Saint Louis University
St. Louis, Missouri

Reference
1. Aboraya A, Nasrallah HA, Elswick DE, et al. Measurement-based care in psychiatry-past, present, and future. Innov Clin Neurosci. 2018;15(11-12):13-26.

Issue
Current Psychiatry - 18(8)
Issue
Current Psychiatry - 18(8)
Page Number
9
Page Number
9
Publications
Publications
Article Type
Display Headline
Backlash against using rating scales
Display Headline
Backlash against using rating scales
Sections
Disallow All Ads
Content Gating
No Gating (article Unlocked/Free)
Alternative CME
Disqus Comments
Default
Use ProPublica
Hide sidebar & use full width
render the right sidebar.
Article PDF Media

Seizure-like episodes, but is it really epilepsy?

Article Type
Changed
Thu, 08/01/2019 - 00:01
Display Headline
Seizure-like episodes, but is it really epilepsy?

CASE Increasingly frequent paroxysmal episodes

Ms. N, age 12, comes to the hospital for evaluation of paroxysmal episodes of pain, weakness, and muscle spasms. A neurologist who evaluated her as an outpatient had recommended a routine electroencephalogram (EEG); after those results were inconclusive, Ms. N’s mother brought her to the hospital for a 24-hour video EEG.

Ms. N has a history of asthma. She has no history of seizures or headache, but her mother has an unspecified seizure disorder that has been stable with antiepileptic medication for many years. Ms. N has no other family history of autoimmune or neurologic disorders.

Ms. N’s episodes began 6 months ago and have progressively increased in frequency from 5 to 12 episodes a day. She says that before she has an episode, she “ feels tingling in her fingers and mouth, and butterflies in her belly,” and then her “whole body clenches up.” She denies experiencing tongue biting, facial or extremity weakness, incontinence, or loss of consciousness during these episodes.

Shortly before her hospitalization, Ms. N had won a scholarship to attend an overnight art camp. Because her episodes were becoming more frequent and their etiology remained unclear, Ms. N and her mother decided it would be unsafe for her to attend, and that she should go to the hospital for evaluation instead.

EVALUATION Tough questions reveal answers

The pediatric team evaluates Ms. N. Her physical exam, laboratory values, and imaging are all within normal limits. Her neurologic exam demonstrates full strength, tone, and sensation in all extremities. All cranial nerves and reflexes are intact. No dysmorphic features or gait abnormalities are noted. All laboratory and imaging tests are normal, including complete blood cell count, electrolytes, calcium, magnesium, phosphorus, glucose, creatine kinase, liver enzymes, urine drug screen, human chorionic gonadotropin (hCG) urine test, and head CT.

After the initial workup, the pediatric team consults the child and adolescent psychiatry team for a complete assessment of Ms. N due to concerns that a psychological component is contributing to her episodes. According to the psychosocial history obtained from Ms. N and her mother, Ms. N had experienced disrupted attachment, trauma, and loss. At age 5, Ms. N was temporarily removed from her mother’s custody after a fight between her mother and brother. At age 9, Ms. N’s stepfather, her primary father figure, died of a brain tumor.

Ms. N also has significant trauma stemming from her relationship with her biological father. Ms. N’s mother reports that her daughter was conceived during nonconsensual sexual intercourse. Ms. N did not have much contact with her biological father until 6 months ago, when he started picking her up at school and taking her to his home for several hours without permission or supervision. Afterwards, Ms. N confided to her mother and a teacher that her father sexually assaulted her during those visits.

Continue to: Ms. N and her mother...

 

 

Ms. N and her mother reported the assault to the police and were awaiting legal action.

During the interview with the psychiatry team, Ms. N denies that any thoughts or actions trigger the episodes and reports that she cannot control when they happen. Because she cannot anticipate the episodes, she says she is afraid to leave her house. She does not know why the episodes are happening and feels frustrated that they are getting worse. Ms. N says, “I have been feeling down lately,” but she denies hopelessness, worthlessness, suicidal ideation, homicidal ideation, delusions, or hallucinations.

In the hospital, when the psychiatry team asks Ms. N about her visits with her father, she says that they are “too painful to talk about,” and fears that discussing them will trigger an episode. However, her mother suggests that her daughter’s sexual trauma, as well as ongoing frustrations with the legal system, are influencing her mood; she has had low energy, poor appetite, and is spending more time in bed. Her mother also reports that Ms. N “avoids going out in the sun and spending time with her friends outside. She doesn’t seem to enjoy shopping and art like she used to.” Ms. N told her mother that she was having nightmares about the trauma and “could not stop thinking about some of the bad stuff that happened during the day.”

Ten minutes into the interview, while being questioned about her father, Ms. N experiences a spastic episode. She curls up in bed on her left side, clenches her entire body, and shuts her eyes. Her mother quickly runs to her bedside and counts the seconds until the end of the episode. After 25 seconds, Ms. N awakes with full recollection of the episode. On review of the video EEG during the episode, no ictal patterns are seen.

[polldaddy:10375873]

The authors’ observations

Paroxysmal episodes of weakness, numbness, and muscle spasms in a young female are suggestive of either epilepsy or nonepileptic seizure (NES).1,2 The negative EEG and physical features are inconsistent with epileptiform seizure, and Ms. N’s history and evaluation are suggestive of NES. Nonepileptic seizures are a type of a conversion disorder, or functional neurologic symptom disorder, in which a patient experiences weakness, abnormal movements, or seizure-like episodes that are inconsistent with organic neurologic disease.3 When a diagnosis of conversion disorder is suspected, a clinician must always consider other pathology that can explain the symptoms, such as migraine, vasovagal syncope, or intracranial mass. If a patient has focal neurologic deficits, head imaging should be pursued. Additionally, the clinician must screen for malingering and factitious disorder before establishing a definitive diagnosis. However, conversion disorder is not a diagnosis of exclusion. For example, a negative EEG does not rule out epilepsy, and patients can have both epilepsy and concomitant NES.

Continue to: Although NES is a common...

 

 

Although NES is a common type of conversion disorder, it is often difficult to diagnose, manage, and treat. Patients often receive antiepileptic medications but continue to have worsening events that are refractory to treatment. Various clinical features can suggest NES instead of epilepsy. Forced eye closure on video recording is a specific finding suggestive of NES, yet this feature is not sufficient to make the diagnosis.4 A video EEG must be performed to assess for epilepsy. The diagnosis of NES does not exclude the possibility that a patient has epilepsy, as NES can occur in up to 40% of patients with epilepsy.5 A video EEG without ictal patterns before, during, and after an observed episode is diagnostic of NES.6

[polldaddy:10375874]

The authors’ observations

Conversion disorders such as NES are a presentation of neurologic symptoms that cannot be readily accounted for by other conditions and are often associated with antecedent trauma. Multiple factors in Ms. N’s history increase her risk of NES, including loss of multiple loved ones, ongoing legal involvement, and alleged sexual abuse by her father.

Victims of sexual abuse are more likely than the general population to demonstrate symptoms of conversion disorder, especially NES.7,8 The onset of paroxysmal episodes after incestuous abuse in a teenage girl is characteristic of NES. Compared with patients with complex partial epilepsy (CPE), patients with NES are 3 times more likely to report sexual trauma.9,10 Children who report sexual abuse that precedes NES are more likely to have been victimized by a first-degree relative than patients with CPE who report sexual abuse.11 Risk factors for victims developing NES may be related to the severity of adversity, stress sensitivity, and decreased hippocampal volume.12,13

Ms. N endorsed many psychiatric symptoms that accompany her paroxysmal episodes; this is similar to findings in other patients with NES.14 One study found that depression is 3 times more prevalent and PTSD is 8 times more prevalent in patients with NES.12 During the evaluation, Ms. N’s mother said her daughter had low energy, poor appetite, lethargy, and anhedonia for the preceding 5 months, which is consistent with adjustment disorder.3 Her flashbacks, nightmares, difficulty sleeping, and agoraphobia, along with her trouble engaging with the people and activities that used to bring her joy, are symptoms of PTSD. Nonepileptic seizure is often associated with PTSD and can be viewed as an expression of a dissociated subtype.15

In a literature review, Durrant et al16 isolated prognostic indicators for NES (Table16). This study found that 70% of children and 40% of adults achieve remission from NES. Ms. N’s case has multiple concerning features, such as her comorbid psychiatric conditions, ongoing involvement in a legal case, and sexual trauma; this last factor is associated with the most severe symptoms and worse outcomes.16,17 Despite this somber reality, Ms. N has the support of her mother and is relatively young, which play a vital role in recovery.

Prognostic indicators for nonepileptic seizure

Continue to: TREATMENT A strategy for minimizing the episodes

 

 

TREATMENT A strategy for minimizing the episodes

Ms. N’s medical workup remains unremarkable throughout the rest of her hospital stay. The psychiatry and pediatric teams discuss their assessments and agree that NES is the most likely diagnosis. The psychiatry team counsels Ms. N and her mother on the diagnosis and etiology of NES.

[polldaddy:10375876]

The authors’ observations

Cognitive-behavioral therapy is currently the treatment of choice for reducing seizure frequency in patients with NES.18,19 The use of CBT was suggested due to the theory that NES represents a dissociative response to trauma. Therapy focuses on changing a patient’s beliefs and perceptions associated with attacks.5 A randomized study of 66 patients with NES compared the use of CBT plus standard medical care with standard medical care alone.18 The standard medical care consisted of supportive treatment, an explanation of NES from a neuropsychiatrist, and supervised withdrawal of antiepileptic drugs. The CBT treatment group was offered weekly hour-long sessions for 12 weeks, accompanied by CBT homework and journaling the frequency and nature of seizure episodes (the CBT techniques are outlined in the Figure18). After 4 months, the CBT treatment group had fewer seizures, and after a 6-month follow-up, they were more likely to be seizure-free. However, in this study, CBT treatment did not improve mood or employment status.

CBT techniques for nonepileptic seizure

A later investigation looked at using selective serotonin reuptake inhibitors to treat NES in adults.19 This study divided participants into 4 treatment groups: CBT with informed psychotherapy (CBT-ip), CBT-ip plus sertraline, sertraline alone, and treatment as usual. Sertraline was titrated up to a dose of 200 mg/d as tolerated. After 16 weeks of sertraline alone, seizure frequency did not decrease. Although both CBT groups showed a reduction in symptoms of up to 60%, the CBT-ip group reported fewer psychiatric symptoms with better social interactions, quality of life, and global functioning compared with patients treated with CBT-ip plus sertraline. The authors suggested that this may be due to the somatic adverse effects associated with sertraline. This study suggests that CBT without medication is the treatment of choice.

In addition to CBT, studies of psychodynamic psychotherapy for NES have had promising findings.20 Psychodynamic psychotherapy focuses on addressing conscious and unconscious anger, loss, feelings of isolation, and trauma. Through improving emotional processing, insight, coping skills and self-regulation, patients often benefit from an improvement in seizures, psychosocial functioning and health care utilization.

Metin et al21 found that group therapy alongside a family-centered approach elicited a strong and durable reduction in seizures in patients with NES. At enrollment, investigators distributed information on NES to patients and families. Psychoeducation and psychoanalysis with behavior modification techniques were provided in 90-minute weekly group sessions over 3 months. Participants also underwent monthly individualized sessions for standard psychiatric care for 9 months. During the group sessions, operant conditioning techniques were used to prevent secondary gain from seizure-like activity. Families met 4 times for 1 hour each to discuss seizures, receive psychoeducation on a subconscious etiology of NES, and learn behavior modification techniques. All 9 participants who completed group and individual therapy reported a significant and sustained reduction in seizure frequency by at least 50% at 12-month follow-up. Patients also demonstrated improvements in mood, anxiety, and quality of life.

Continue to: A meta-analysis...

 

 

A meta-analysis by Carlson and Perry22 that included 13 studies and 228 participants, examined different treatment modalities and their effectiveness for NES. They found that patients who received psychological intervention had a 47% remission rate and 82% improvement in seizure frequency compared with only 14% to 23% of those who did not receive therapy. They postulated that therapy for this illness must be flexible to properly address the socially, psychologically, and functionally heterogenous patient population. Although there are few randomized controlled trials for NES to determine the best evidence-based intervention, there is now consensus that NES has a favorable prognosis when barriers to psychological care are eliminated.

OUTCOME Referral for CBT

The treatment team advises Ms. N to engage in outpatient therapy after discharge from the hospital. Ms. N and her mother agree to the treatment plan, and leave the hospital with a referral for CBT the next day.

 

Bottom Line

Nonepileptic seizure (NES) is a type of conversion disorder characterized by seizure-like episodes without ictal qualities. Risk factors for NES include concomitant epilepsy, psychiatric disorders, unstable psychosocial situations, and antecedent trauma. Patients with a history of incestuous sexual abuse are most at risk for developing NES. A normal EEG that fully captures a seizure-like episode is diagnostic of NES. Cognitive-behavioral therapy can minimize seizure frequency and intensity.

Related Resources

Drug Brand Name

Sertraline • Zoloft

References

1. Lesser R. Psychogenic seizures. Neurology. 1996;46(6):1499-1507.
2. Stone J, LaFrance W, Brown R, et al. Conversion disorder: current problems and potential solutions for DSM-5. J Psychosom Res. 2011;71(6):369-376.
3. Diagnostic and statistical manual of mental disorders, 5th ed. Washington, DC: American Psychiatric Association; 2013.
4. Syed T, Arozullah A, Suciu G, et al. Do observer and self-reports of ictal eye closure predict psychogenic nonepileptic seizures? Epilepsia. 2008;49(5):898-904.
5. Vega-Zelaya L, Alvarez M, Ezquiaga E, et al. Psychogenic non-epileptic seizures in a surgical epilepsy unit: experience and a comprehensive review. Epilepsy Topics. 2014. doi: 10.5772/57439.
6. LaFrance W, Baker G, Duncan R, et al. Minimum requirements for the diagnosis of psychogenic nonepileptic seizures: a staged approach. Epilepsia. 2013;54(11):2005-2018.
7. Roeloes K, Pasman J. Stress, childhood trauma, and cognitive functions in functional neurologic disorders. In: Hallett M, Stone J, Carson A, eds. Handbook of clinical neurology: functional neurologic disorders. 3rd ed. New York, NY: Elsevier; 2017:139-155.
8. Paras M, Murad M, Chen L, et al. Sexual abuse and lifetime diagnosis of somatic disorders. JAMA. 2009;302(5):550.
9. Fiszman A, Alves-Leon SV, Nunes RG, et al. Traumatic events and posttraumatic stress disorder in patients with psychogenic nonepileptic seizures: a critical review. Epilepsy Behav. 2004;5(6):818-825.
10. Sharpe D, Faye C. Non-epileptic seizures and child sexual abuse: a critical review of the literature. Clin Psychol Rev. 2006;26(8):1020-1040.
11. Alper K, Devinsky O, Perrine K, et al. Nonepileptic seizures and childhood sexual and physical abuse. Neurology. 1993;43(10):1950-1953.
12. Plioplys S, Doss J, Siddarth P et al. A multisite controlled study of risk factors in pediatric psychogenic nonepileptic seizures. Epilepsia. 2014;55(11):1739-1747.
13. Andersen S, Tomada A, Vincow E, et al. Preliminary evidence for sensitive periods in the effect of childhood sexual abuse on regional brain development. J Neuropsychiatry Clin Neurosci. 2008;20(3):292-301.
14. Sar V. Childhood trauma, dissociation, and psychiatric comorbidity in patients with conversion disorder. Am J Psychiatry. 2004;161(12):2271-2276.
15. Rosenberg HJ, Rosenberg SD, Williamson PD, et al. A comparative study of trauma and posttraumatic stress disorder prevalence in epilepsy patients and psychogenic nonepileptic seizure patients. Epilepsia. 2000;41(4):447-452.
16. Durrant J, Rickards H, Cavanna A. Prognosis and outcome predictors in psychogenic nonepileptic seizures. Epilepsy Res Treat. 2011;2011:1-7.
17. Selkirk M, Duncan R, Oto M, et al. Clinical differences between patients with nonepileptic seizures who report antecedent sexual abuse and those who do not. Epilepsia. 2008;49(8):1446-1450.
18. Goldstein L, Chalder T, Chigwedere C, et al. Cognitive-behavioral therapy for psychogenic nonepileptic seizures: a pilot RCT. Neurology. 2010;74(24):1986-1994.
19. LaFrance W, Baird G, Barry J, et al. Multicenter pilot treatment trial for psychogenic nonepileptic seizures. JAMA Psychiatry. 2014;71(9):997.
20. Howlett S, Reuber M. An augmented model of brief psychodynamic interpersonal therapy for patients with nonepileptic seizures. Psychotherapy (Chic). 2009;46(1):125-138.
21. Metin SZ, Ozmen M, Metin B, et al. Treatment with group psychotherapy for chronic psychogenic nonepileptic seizures. Epilepsy Behav. 2013;28(1):91-94.
22. Carlson P, Perry KN. Psychological interventions for psychogenic non-epileptic seizures: a meta-analysis. Seizure. 2017;45:142-150.

Article PDF
Author and Disclosure Information

Dr. Madora is a PGY-2 Psychiatry Resident, Department of Psychiatry, Montefiore Medical Center, Bronx, New York. Dr. Janssen is Associate Professor of Psychiatry and Behavioral Sciences, Northwestern University Feinberg School of Medicine, Chicago, Illinois. Dr. Junewicz is a Forensic Psychiatry Fellow, Department of Psychiatry, New York University, New York, New York.

Disclosures
The authors report no financial relationships with any company whose products are mentioned in this article, or with manufacturers of competing products.

Issue
Current Psychiatry - 18(8)
Publications
Topics
Page Number
42-47
Sections
Author and Disclosure Information

Dr. Madora is a PGY-2 Psychiatry Resident, Department of Psychiatry, Montefiore Medical Center, Bronx, New York. Dr. Janssen is Associate Professor of Psychiatry and Behavioral Sciences, Northwestern University Feinberg School of Medicine, Chicago, Illinois. Dr. Junewicz is a Forensic Psychiatry Fellow, Department of Psychiatry, New York University, New York, New York.

Disclosures
The authors report no financial relationships with any company whose products are mentioned in this article, or with manufacturers of competing products.

Author and Disclosure Information

Dr. Madora is a PGY-2 Psychiatry Resident, Department of Psychiatry, Montefiore Medical Center, Bronx, New York. Dr. Janssen is Associate Professor of Psychiatry and Behavioral Sciences, Northwestern University Feinberg School of Medicine, Chicago, Illinois. Dr. Junewicz is a Forensic Psychiatry Fellow, Department of Psychiatry, New York University, New York, New York.

Disclosures
The authors report no financial relationships with any company whose products are mentioned in this article, or with manufacturers of competing products.

Article PDF
Article PDF

CASE Increasingly frequent paroxysmal episodes

Ms. N, age 12, comes to the hospital for evaluation of paroxysmal episodes of pain, weakness, and muscle spasms. A neurologist who evaluated her as an outpatient had recommended a routine electroencephalogram (EEG); after those results were inconclusive, Ms. N’s mother brought her to the hospital for a 24-hour video EEG.

Ms. N has a history of asthma. She has no history of seizures or headache, but her mother has an unspecified seizure disorder that has been stable with antiepileptic medication for many years. Ms. N has no other family history of autoimmune or neurologic disorders.

Ms. N’s episodes began 6 months ago and have progressively increased in frequency from 5 to 12 episodes a day. She says that before she has an episode, she “ feels tingling in her fingers and mouth, and butterflies in her belly,” and then her “whole body clenches up.” She denies experiencing tongue biting, facial or extremity weakness, incontinence, or loss of consciousness during these episodes.

Shortly before her hospitalization, Ms. N had won a scholarship to attend an overnight art camp. Because her episodes were becoming more frequent and their etiology remained unclear, Ms. N and her mother decided it would be unsafe for her to attend, and that she should go to the hospital for evaluation instead.

EVALUATION Tough questions reveal answers

The pediatric team evaluates Ms. N. Her physical exam, laboratory values, and imaging are all within normal limits. Her neurologic exam demonstrates full strength, tone, and sensation in all extremities. All cranial nerves and reflexes are intact. No dysmorphic features or gait abnormalities are noted. All laboratory and imaging tests are normal, including complete blood cell count, electrolytes, calcium, magnesium, phosphorus, glucose, creatine kinase, liver enzymes, urine drug screen, human chorionic gonadotropin (hCG) urine test, and head CT.

After the initial workup, the pediatric team consults the child and adolescent psychiatry team for a complete assessment of Ms. N due to concerns that a psychological component is contributing to her episodes. According to the psychosocial history obtained from Ms. N and her mother, Ms. N had experienced disrupted attachment, trauma, and loss. At age 5, Ms. N was temporarily removed from her mother’s custody after a fight between her mother and brother. At age 9, Ms. N’s stepfather, her primary father figure, died of a brain tumor.

Ms. N also has significant trauma stemming from her relationship with her biological father. Ms. N’s mother reports that her daughter was conceived during nonconsensual sexual intercourse. Ms. N did not have much contact with her biological father until 6 months ago, when he started picking her up at school and taking her to his home for several hours without permission or supervision. Afterwards, Ms. N confided to her mother and a teacher that her father sexually assaulted her during those visits.

Continue to: Ms. N and her mother...

 

 

Ms. N and her mother reported the assault to the police and were awaiting legal action.

During the interview with the psychiatry team, Ms. N denies that any thoughts or actions trigger the episodes and reports that she cannot control when they happen. Because she cannot anticipate the episodes, she says she is afraid to leave her house. She does not know why the episodes are happening and feels frustrated that they are getting worse. Ms. N says, “I have been feeling down lately,” but she denies hopelessness, worthlessness, suicidal ideation, homicidal ideation, delusions, or hallucinations.

In the hospital, when the psychiatry team asks Ms. N about her visits with her father, she says that they are “too painful to talk about,” and fears that discussing them will trigger an episode. However, her mother suggests that her daughter’s sexual trauma, as well as ongoing frustrations with the legal system, are influencing her mood; she has had low energy, poor appetite, and is spending more time in bed. Her mother also reports that Ms. N “avoids going out in the sun and spending time with her friends outside. She doesn’t seem to enjoy shopping and art like she used to.” Ms. N told her mother that she was having nightmares about the trauma and “could not stop thinking about some of the bad stuff that happened during the day.”

Ten minutes into the interview, while being questioned about her father, Ms. N experiences a spastic episode. She curls up in bed on her left side, clenches her entire body, and shuts her eyes. Her mother quickly runs to her bedside and counts the seconds until the end of the episode. After 25 seconds, Ms. N awakes with full recollection of the episode. On review of the video EEG during the episode, no ictal patterns are seen.

[polldaddy:10375873]

The authors’ observations

Paroxysmal episodes of weakness, numbness, and muscle spasms in a young female are suggestive of either epilepsy or nonepileptic seizure (NES).1,2 The negative EEG and physical features are inconsistent with epileptiform seizure, and Ms. N’s history and evaluation are suggestive of NES. Nonepileptic seizures are a type of a conversion disorder, or functional neurologic symptom disorder, in which a patient experiences weakness, abnormal movements, or seizure-like episodes that are inconsistent with organic neurologic disease.3 When a diagnosis of conversion disorder is suspected, a clinician must always consider other pathology that can explain the symptoms, such as migraine, vasovagal syncope, or intracranial mass. If a patient has focal neurologic deficits, head imaging should be pursued. Additionally, the clinician must screen for malingering and factitious disorder before establishing a definitive diagnosis. However, conversion disorder is not a diagnosis of exclusion. For example, a negative EEG does not rule out epilepsy, and patients can have both epilepsy and concomitant NES.

Continue to: Although NES is a common...

 

 

Although NES is a common type of conversion disorder, it is often difficult to diagnose, manage, and treat. Patients often receive antiepileptic medications but continue to have worsening events that are refractory to treatment. Various clinical features can suggest NES instead of epilepsy. Forced eye closure on video recording is a specific finding suggestive of NES, yet this feature is not sufficient to make the diagnosis.4 A video EEG must be performed to assess for epilepsy. The diagnosis of NES does not exclude the possibility that a patient has epilepsy, as NES can occur in up to 40% of patients with epilepsy.5 A video EEG without ictal patterns before, during, and after an observed episode is diagnostic of NES.6

[polldaddy:10375874]

The authors’ observations

Conversion disorders such as NES are a presentation of neurologic symptoms that cannot be readily accounted for by other conditions and are often associated with antecedent trauma. Multiple factors in Ms. N’s history increase her risk of NES, including loss of multiple loved ones, ongoing legal involvement, and alleged sexual abuse by her father.

Victims of sexual abuse are more likely than the general population to demonstrate symptoms of conversion disorder, especially NES.7,8 The onset of paroxysmal episodes after incestuous abuse in a teenage girl is characteristic of NES. Compared with patients with complex partial epilepsy (CPE), patients with NES are 3 times more likely to report sexual trauma.9,10 Children who report sexual abuse that precedes NES are more likely to have been victimized by a first-degree relative than patients with CPE who report sexual abuse.11 Risk factors for victims developing NES may be related to the severity of adversity, stress sensitivity, and decreased hippocampal volume.12,13

Ms. N endorsed many psychiatric symptoms that accompany her paroxysmal episodes; this is similar to findings in other patients with NES.14 One study found that depression is 3 times more prevalent and PTSD is 8 times more prevalent in patients with NES.12 During the evaluation, Ms. N’s mother said her daughter had low energy, poor appetite, lethargy, and anhedonia for the preceding 5 months, which is consistent with adjustment disorder.3 Her flashbacks, nightmares, difficulty sleeping, and agoraphobia, along with her trouble engaging with the people and activities that used to bring her joy, are symptoms of PTSD. Nonepileptic seizure is often associated with PTSD and can be viewed as an expression of a dissociated subtype.15

In a literature review, Durrant et al16 isolated prognostic indicators for NES (Table16). This study found that 70% of children and 40% of adults achieve remission from NES. Ms. N’s case has multiple concerning features, such as her comorbid psychiatric conditions, ongoing involvement in a legal case, and sexual trauma; this last factor is associated with the most severe symptoms and worse outcomes.16,17 Despite this somber reality, Ms. N has the support of her mother and is relatively young, which play a vital role in recovery.

Prognostic indicators for nonepileptic seizure

Continue to: TREATMENT A strategy for minimizing the episodes

 

 

TREATMENT A strategy for minimizing the episodes

Ms. N’s medical workup remains unremarkable throughout the rest of her hospital stay. The psychiatry and pediatric teams discuss their assessments and agree that NES is the most likely diagnosis. The psychiatry team counsels Ms. N and her mother on the diagnosis and etiology of NES.

[polldaddy:10375876]

The authors’ observations

Cognitive-behavioral therapy is currently the treatment of choice for reducing seizure frequency in patients with NES.18,19 The use of CBT was suggested due to the theory that NES represents a dissociative response to trauma. Therapy focuses on changing a patient’s beliefs and perceptions associated with attacks.5 A randomized study of 66 patients with NES compared the use of CBT plus standard medical care with standard medical care alone.18 The standard medical care consisted of supportive treatment, an explanation of NES from a neuropsychiatrist, and supervised withdrawal of antiepileptic drugs. The CBT treatment group was offered weekly hour-long sessions for 12 weeks, accompanied by CBT homework and journaling the frequency and nature of seizure episodes (the CBT techniques are outlined in the Figure18). After 4 months, the CBT treatment group had fewer seizures, and after a 6-month follow-up, they were more likely to be seizure-free. However, in this study, CBT treatment did not improve mood or employment status.

CBT techniques for nonepileptic seizure

A later investigation looked at using selective serotonin reuptake inhibitors to treat NES in adults.19 This study divided participants into 4 treatment groups: CBT with informed psychotherapy (CBT-ip), CBT-ip plus sertraline, sertraline alone, and treatment as usual. Sertraline was titrated up to a dose of 200 mg/d as tolerated. After 16 weeks of sertraline alone, seizure frequency did not decrease. Although both CBT groups showed a reduction in symptoms of up to 60%, the CBT-ip group reported fewer psychiatric symptoms with better social interactions, quality of life, and global functioning compared with patients treated with CBT-ip plus sertraline. The authors suggested that this may be due to the somatic adverse effects associated with sertraline. This study suggests that CBT without medication is the treatment of choice.

In addition to CBT, studies of psychodynamic psychotherapy for NES have had promising findings.20 Psychodynamic psychotherapy focuses on addressing conscious and unconscious anger, loss, feelings of isolation, and trauma. Through improving emotional processing, insight, coping skills and self-regulation, patients often benefit from an improvement in seizures, psychosocial functioning and health care utilization.

Metin et al21 found that group therapy alongside a family-centered approach elicited a strong and durable reduction in seizures in patients with NES. At enrollment, investigators distributed information on NES to patients and families. Psychoeducation and psychoanalysis with behavior modification techniques were provided in 90-minute weekly group sessions over 3 months. Participants also underwent monthly individualized sessions for standard psychiatric care for 9 months. During the group sessions, operant conditioning techniques were used to prevent secondary gain from seizure-like activity. Families met 4 times for 1 hour each to discuss seizures, receive psychoeducation on a subconscious etiology of NES, and learn behavior modification techniques. All 9 participants who completed group and individual therapy reported a significant and sustained reduction in seizure frequency by at least 50% at 12-month follow-up. Patients also demonstrated improvements in mood, anxiety, and quality of life.

Continue to: A meta-analysis...

 

 

A meta-analysis by Carlson and Perry22 that included 13 studies and 228 participants, examined different treatment modalities and their effectiveness for NES. They found that patients who received psychological intervention had a 47% remission rate and 82% improvement in seizure frequency compared with only 14% to 23% of those who did not receive therapy. They postulated that therapy for this illness must be flexible to properly address the socially, psychologically, and functionally heterogenous patient population. Although there are few randomized controlled trials for NES to determine the best evidence-based intervention, there is now consensus that NES has a favorable prognosis when barriers to psychological care are eliminated.

OUTCOME Referral for CBT

The treatment team advises Ms. N to engage in outpatient therapy after discharge from the hospital. Ms. N and her mother agree to the treatment plan, and leave the hospital with a referral for CBT the next day.

 

Bottom Line

Nonepileptic seizure (NES) is a type of conversion disorder characterized by seizure-like episodes without ictal qualities. Risk factors for NES include concomitant epilepsy, psychiatric disorders, unstable psychosocial situations, and antecedent trauma. Patients with a history of incestuous sexual abuse are most at risk for developing NES. A normal EEG that fully captures a seizure-like episode is diagnostic of NES. Cognitive-behavioral therapy can minimize seizure frequency and intensity.

Related Resources

Drug Brand Name

Sertraline • Zoloft

CASE Increasingly frequent paroxysmal episodes

Ms. N, age 12, comes to the hospital for evaluation of paroxysmal episodes of pain, weakness, and muscle spasms. A neurologist who evaluated her as an outpatient had recommended a routine electroencephalogram (EEG); after those results were inconclusive, Ms. N’s mother brought her to the hospital for a 24-hour video EEG.

Ms. N has a history of asthma. She has no history of seizures or headache, but her mother has an unspecified seizure disorder that has been stable with antiepileptic medication for many years. Ms. N has no other family history of autoimmune or neurologic disorders.

Ms. N’s episodes began 6 months ago and have progressively increased in frequency from 5 to 12 episodes a day. She says that before she has an episode, she “ feels tingling in her fingers and mouth, and butterflies in her belly,” and then her “whole body clenches up.” She denies experiencing tongue biting, facial or extremity weakness, incontinence, or loss of consciousness during these episodes.

Shortly before her hospitalization, Ms. N had won a scholarship to attend an overnight art camp. Because her episodes were becoming more frequent and their etiology remained unclear, Ms. N and her mother decided it would be unsafe for her to attend, and that she should go to the hospital for evaluation instead.

EVALUATION Tough questions reveal answers

The pediatric team evaluates Ms. N. Her physical exam, laboratory values, and imaging are all within normal limits. Her neurologic exam demonstrates full strength, tone, and sensation in all extremities. All cranial nerves and reflexes are intact. No dysmorphic features or gait abnormalities are noted. All laboratory and imaging tests are normal, including complete blood cell count, electrolytes, calcium, magnesium, phosphorus, glucose, creatine kinase, liver enzymes, urine drug screen, human chorionic gonadotropin (hCG) urine test, and head CT.

After the initial workup, the pediatric team consults the child and adolescent psychiatry team for a complete assessment of Ms. N due to concerns that a psychological component is contributing to her episodes. According to the psychosocial history obtained from Ms. N and her mother, Ms. N had experienced disrupted attachment, trauma, and loss. At age 5, Ms. N was temporarily removed from her mother’s custody after a fight between her mother and brother. At age 9, Ms. N’s stepfather, her primary father figure, died of a brain tumor.

Ms. N also has significant trauma stemming from her relationship with her biological father. Ms. N’s mother reports that her daughter was conceived during nonconsensual sexual intercourse. Ms. N did not have much contact with her biological father until 6 months ago, when he started picking her up at school and taking her to his home for several hours without permission or supervision. Afterwards, Ms. N confided to her mother and a teacher that her father sexually assaulted her during those visits.

Continue to: Ms. N and her mother...

 

 

Ms. N and her mother reported the assault to the police and were awaiting legal action.

During the interview with the psychiatry team, Ms. N denies that any thoughts or actions trigger the episodes and reports that she cannot control when they happen. Because she cannot anticipate the episodes, she says she is afraid to leave her house. She does not know why the episodes are happening and feels frustrated that they are getting worse. Ms. N says, “I have been feeling down lately,” but she denies hopelessness, worthlessness, suicidal ideation, homicidal ideation, delusions, or hallucinations.

In the hospital, when the psychiatry team asks Ms. N about her visits with her father, she says that they are “too painful to talk about,” and fears that discussing them will trigger an episode. However, her mother suggests that her daughter’s sexual trauma, as well as ongoing frustrations with the legal system, are influencing her mood; she has had low energy, poor appetite, and is spending more time in bed. Her mother also reports that Ms. N “avoids going out in the sun and spending time with her friends outside. She doesn’t seem to enjoy shopping and art like she used to.” Ms. N told her mother that she was having nightmares about the trauma and “could not stop thinking about some of the bad stuff that happened during the day.”

Ten minutes into the interview, while being questioned about her father, Ms. N experiences a spastic episode. She curls up in bed on her left side, clenches her entire body, and shuts her eyes. Her mother quickly runs to her bedside and counts the seconds until the end of the episode. After 25 seconds, Ms. N awakes with full recollection of the episode. On review of the video EEG during the episode, no ictal patterns are seen.

[polldaddy:10375873]

The authors’ observations

Paroxysmal episodes of weakness, numbness, and muscle spasms in a young female are suggestive of either epilepsy or nonepileptic seizure (NES).1,2 The negative EEG and physical features are inconsistent with epileptiform seizure, and Ms. N’s history and evaluation are suggestive of NES. Nonepileptic seizures are a type of a conversion disorder, or functional neurologic symptom disorder, in which a patient experiences weakness, abnormal movements, or seizure-like episodes that are inconsistent with organic neurologic disease.3 When a diagnosis of conversion disorder is suspected, a clinician must always consider other pathology that can explain the symptoms, such as migraine, vasovagal syncope, or intracranial mass. If a patient has focal neurologic deficits, head imaging should be pursued. Additionally, the clinician must screen for malingering and factitious disorder before establishing a definitive diagnosis. However, conversion disorder is not a diagnosis of exclusion. For example, a negative EEG does not rule out epilepsy, and patients can have both epilepsy and concomitant NES.

Continue to: Although NES is a common...

 

 

Although NES is a common type of conversion disorder, it is often difficult to diagnose, manage, and treat. Patients often receive antiepileptic medications but continue to have worsening events that are refractory to treatment. Various clinical features can suggest NES instead of epilepsy. Forced eye closure on video recording is a specific finding suggestive of NES, yet this feature is not sufficient to make the diagnosis.4 A video EEG must be performed to assess for epilepsy. The diagnosis of NES does not exclude the possibility that a patient has epilepsy, as NES can occur in up to 40% of patients with epilepsy.5 A video EEG without ictal patterns before, during, and after an observed episode is diagnostic of NES.6

[polldaddy:10375874]

The authors’ observations

Conversion disorders such as NES are a presentation of neurologic symptoms that cannot be readily accounted for by other conditions and are often associated with antecedent trauma. Multiple factors in Ms. N’s history increase her risk of NES, including loss of multiple loved ones, ongoing legal involvement, and alleged sexual abuse by her father.

Victims of sexual abuse are more likely than the general population to demonstrate symptoms of conversion disorder, especially NES.7,8 The onset of paroxysmal episodes after incestuous abuse in a teenage girl is characteristic of NES. Compared with patients with complex partial epilepsy (CPE), patients with NES are 3 times more likely to report sexual trauma.9,10 Children who report sexual abuse that precedes NES are more likely to have been victimized by a first-degree relative than patients with CPE who report sexual abuse.11 Risk factors for victims developing NES may be related to the severity of adversity, stress sensitivity, and decreased hippocampal volume.12,13

Ms. N endorsed many psychiatric symptoms that accompany her paroxysmal episodes; this is similar to findings in other patients with NES.14 One study found that depression is 3 times more prevalent and PTSD is 8 times more prevalent in patients with NES.12 During the evaluation, Ms. N’s mother said her daughter had low energy, poor appetite, lethargy, and anhedonia for the preceding 5 months, which is consistent with adjustment disorder.3 Her flashbacks, nightmares, difficulty sleeping, and agoraphobia, along with her trouble engaging with the people and activities that used to bring her joy, are symptoms of PTSD. Nonepileptic seizure is often associated with PTSD and can be viewed as an expression of a dissociated subtype.15

In a literature review, Durrant et al16 isolated prognostic indicators for NES (Table16). This study found that 70% of children and 40% of adults achieve remission from NES. Ms. N’s case has multiple concerning features, such as her comorbid psychiatric conditions, ongoing involvement in a legal case, and sexual trauma; this last factor is associated with the most severe symptoms and worse outcomes.16,17 Despite this somber reality, Ms. N has the support of her mother and is relatively young, which play a vital role in recovery.

Prognostic indicators for nonepileptic seizure

Continue to: TREATMENT A strategy for minimizing the episodes

 

 

TREATMENT A strategy for minimizing the episodes

Ms. N’s medical workup remains unremarkable throughout the rest of her hospital stay. The psychiatry and pediatric teams discuss their assessments and agree that NES is the most likely diagnosis. The psychiatry team counsels Ms. N and her mother on the diagnosis and etiology of NES.

[polldaddy:10375876]

The authors’ observations

Cognitive-behavioral therapy is currently the treatment of choice for reducing seizure frequency in patients with NES.18,19 The use of CBT was suggested due to the theory that NES represents a dissociative response to trauma. Therapy focuses on changing a patient’s beliefs and perceptions associated with attacks.5 A randomized study of 66 patients with NES compared the use of CBT plus standard medical care with standard medical care alone.18 The standard medical care consisted of supportive treatment, an explanation of NES from a neuropsychiatrist, and supervised withdrawal of antiepileptic drugs. The CBT treatment group was offered weekly hour-long sessions for 12 weeks, accompanied by CBT homework and journaling the frequency and nature of seizure episodes (the CBT techniques are outlined in the Figure18). After 4 months, the CBT treatment group had fewer seizures, and after a 6-month follow-up, they were more likely to be seizure-free. However, in this study, CBT treatment did not improve mood or employment status.

CBT techniques for nonepileptic seizure

A later investigation looked at using selective serotonin reuptake inhibitors to treat NES in adults.19 This study divided participants into 4 treatment groups: CBT with informed psychotherapy (CBT-ip), CBT-ip plus sertraline, sertraline alone, and treatment as usual. Sertraline was titrated up to a dose of 200 mg/d as tolerated. After 16 weeks of sertraline alone, seizure frequency did not decrease. Although both CBT groups showed a reduction in symptoms of up to 60%, the CBT-ip group reported fewer psychiatric symptoms with better social interactions, quality of life, and global functioning compared with patients treated with CBT-ip plus sertraline. The authors suggested that this may be due to the somatic adverse effects associated with sertraline. This study suggests that CBT without medication is the treatment of choice.

In addition to CBT, studies of psychodynamic psychotherapy for NES have had promising findings.20 Psychodynamic psychotherapy focuses on addressing conscious and unconscious anger, loss, feelings of isolation, and trauma. Through improving emotional processing, insight, coping skills and self-regulation, patients often benefit from an improvement in seizures, psychosocial functioning and health care utilization.

Metin et al21 found that group therapy alongside a family-centered approach elicited a strong and durable reduction in seizures in patients with NES. At enrollment, investigators distributed information on NES to patients and families. Psychoeducation and psychoanalysis with behavior modification techniques were provided in 90-minute weekly group sessions over 3 months. Participants also underwent monthly individualized sessions for standard psychiatric care for 9 months. During the group sessions, operant conditioning techniques were used to prevent secondary gain from seizure-like activity. Families met 4 times for 1 hour each to discuss seizures, receive psychoeducation on a subconscious etiology of NES, and learn behavior modification techniques. All 9 participants who completed group and individual therapy reported a significant and sustained reduction in seizure frequency by at least 50% at 12-month follow-up. Patients also demonstrated improvements in mood, anxiety, and quality of life.

Continue to: A meta-analysis...

 

 

A meta-analysis by Carlson and Perry22 that included 13 studies and 228 participants, examined different treatment modalities and their effectiveness for NES. They found that patients who received psychological intervention had a 47% remission rate and 82% improvement in seizure frequency compared with only 14% to 23% of those who did not receive therapy. They postulated that therapy for this illness must be flexible to properly address the socially, psychologically, and functionally heterogenous patient population. Although there are few randomized controlled trials for NES to determine the best evidence-based intervention, there is now consensus that NES has a favorable prognosis when barriers to psychological care are eliminated.

OUTCOME Referral for CBT

The treatment team advises Ms. N to engage in outpatient therapy after discharge from the hospital. Ms. N and her mother agree to the treatment plan, and leave the hospital with a referral for CBT the next day.

 

Bottom Line

Nonepileptic seizure (NES) is a type of conversion disorder characterized by seizure-like episodes without ictal qualities. Risk factors for NES include concomitant epilepsy, psychiatric disorders, unstable psychosocial situations, and antecedent trauma. Patients with a history of incestuous sexual abuse are most at risk for developing NES. A normal EEG that fully captures a seizure-like episode is diagnostic of NES. Cognitive-behavioral therapy can minimize seizure frequency and intensity.

Related Resources

Drug Brand Name

Sertraline • Zoloft

References

1. Lesser R. Psychogenic seizures. Neurology. 1996;46(6):1499-1507.
2. Stone J, LaFrance W, Brown R, et al. Conversion disorder: current problems and potential solutions for DSM-5. J Psychosom Res. 2011;71(6):369-376.
3. Diagnostic and statistical manual of mental disorders, 5th ed. Washington, DC: American Psychiatric Association; 2013.
4. Syed T, Arozullah A, Suciu G, et al. Do observer and self-reports of ictal eye closure predict psychogenic nonepileptic seizures? Epilepsia. 2008;49(5):898-904.
5. Vega-Zelaya L, Alvarez M, Ezquiaga E, et al. Psychogenic non-epileptic seizures in a surgical epilepsy unit: experience and a comprehensive review. Epilepsy Topics. 2014. doi: 10.5772/57439.
6. LaFrance W, Baker G, Duncan R, et al. Minimum requirements for the diagnosis of psychogenic nonepileptic seizures: a staged approach. Epilepsia. 2013;54(11):2005-2018.
7. Roeloes K, Pasman J. Stress, childhood trauma, and cognitive functions in functional neurologic disorders. In: Hallett M, Stone J, Carson A, eds. Handbook of clinical neurology: functional neurologic disorders. 3rd ed. New York, NY: Elsevier; 2017:139-155.
8. Paras M, Murad M, Chen L, et al. Sexual abuse and lifetime diagnosis of somatic disorders. JAMA. 2009;302(5):550.
9. Fiszman A, Alves-Leon SV, Nunes RG, et al. Traumatic events and posttraumatic stress disorder in patients with psychogenic nonepileptic seizures: a critical review. Epilepsy Behav. 2004;5(6):818-825.
10. Sharpe D, Faye C. Non-epileptic seizures and child sexual abuse: a critical review of the literature. Clin Psychol Rev. 2006;26(8):1020-1040.
11. Alper K, Devinsky O, Perrine K, et al. Nonepileptic seizures and childhood sexual and physical abuse. Neurology. 1993;43(10):1950-1953.
12. Plioplys S, Doss J, Siddarth P et al. A multisite controlled study of risk factors in pediatric psychogenic nonepileptic seizures. Epilepsia. 2014;55(11):1739-1747.
13. Andersen S, Tomada A, Vincow E, et al. Preliminary evidence for sensitive periods in the effect of childhood sexual abuse on regional brain development. J Neuropsychiatry Clin Neurosci. 2008;20(3):292-301.
14. Sar V. Childhood trauma, dissociation, and psychiatric comorbidity in patients with conversion disorder. Am J Psychiatry. 2004;161(12):2271-2276.
15. Rosenberg HJ, Rosenberg SD, Williamson PD, et al. A comparative study of trauma and posttraumatic stress disorder prevalence in epilepsy patients and psychogenic nonepileptic seizure patients. Epilepsia. 2000;41(4):447-452.
16. Durrant J, Rickards H, Cavanna A. Prognosis and outcome predictors in psychogenic nonepileptic seizures. Epilepsy Res Treat. 2011;2011:1-7.
17. Selkirk M, Duncan R, Oto M, et al. Clinical differences between patients with nonepileptic seizures who report antecedent sexual abuse and those who do not. Epilepsia. 2008;49(8):1446-1450.
18. Goldstein L, Chalder T, Chigwedere C, et al. Cognitive-behavioral therapy for psychogenic nonepileptic seizures: a pilot RCT. Neurology. 2010;74(24):1986-1994.
19. LaFrance W, Baird G, Barry J, et al. Multicenter pilot treatment trial for psychogenic nonepileptic seizures. JAMA Psychiatry. 2014;71(9):997.
20. Howlett S, Reuber M. An augmented model of brief psychodynamic interpersonal therapy for patients with nonepileptic seizures. Psychotherapy (Chic). 2009;46(1):125-138.
21. Metin SZ, Ozmen M, Metin B, et al. Treatment with group psychotherapy for chronic psychogenic nonepileptic seizures. Epilepsy Behav. 2013;28(1):91-94.
22. Carlson P, Perry KN. Psychological interventions for psychogenic non-epileptic seizures: a meta-analysis. Seizure. 2017;45:142-150.

References

1. Lesser R. Psychogenic seizures. Neurology. 1996;46(6):1499-1507.
2. Stone J, LaFrance W, Brown R, et al. Conversion disorder: current problems and potential solutions for DSM-5. J Psychosom Res. 2011;71(6):369-376.
3. Diagnostic and statistical manual of mental disorders, 5th ed. Washington, DC: American Psychiatric Association; 2013.
4. Syed T, Arozullah A, Suciu G, et al. Do observer and self-reports of ictal eye closure predict psychogenic nonepileptic seizures? Epilepsia. 2008;49(5):898-904.
5. Vega-Zelaya L, Alvarez M, Ezquiaga E, et al. Psychogenic non-epileptic seizures in a surgical epilepsy unit: experience and a comprehensive review. Epilepsy Topics. 2014. doi: 10.5772/57439.
6. LaFrance W, Baker G, Duncan R, et al. Minimum requirements for the diagnosis of psychogenic nonepileptic seizures: a staged approach. Epilepsia. 2013;54(11):2005-2018.
7. Roeloes K, Pasman J. Stress, childhood trauma, and cognitive functions in functional neurologic disorders. In: Hallett M, Stone J, Carson A, eds. Handbook of clinical neurology: functional neurologic disorders. 3rd ed. New York, NY: Elsevier; 2017:139-155.
8. Paras M, Murad M, Chen L, et al. Sexual abuse and lifetime diagnosis of somatic disorders. JAMA. 2009;302(5):550.
9. Fiszman A, Alves-Leon SV, Nunes RG, et al. Traumatic events and posttraumatic stress disorder in patients with psychogenic nonepileptic seizures: a critical review. Epilepsy Behav. 2004;5(6):818-825.
10. Sharpe D, Faye C. Non-epileptic seizures and child sexual abuse: a critical review of the literature. Clin Psychol Rev. 2006;26(8):1020-1040.
11. Alper K, Devinsky O, Perrine K, et al. Nonepileptic seizures and childhood sexual and physical abuse. Neurology. 1993;43(10):1950-1953.
12. Plioplys S, Doss J, Siddarth P et al. A multisite controlled study of risk factors in pediatric psychogenic nonepileptic seizures. Epilepsia. 2014;55(11):1739-1747.
13. Andersen S, Tomada A, Vincow E, et al. Preliminary evidence for sensitive periods in the effect of childhood sexual abuse on regional brain development. J Neuropsychiatry Clin Neurosci. 2008;20(3):292-301.
14. Sar V. Childhood trauma, dissociation, and psychiatric comorbidity in patients with conversion disorder. Am J Psychiatry. 2004;161(12):2271-2276.
15. Rosenberg HJ, Rosenberg SD, Williamson PD, et al. A comparative study of trauma and posttraumatic stress disorder prevalence in epilepsy patients and psychogenic nonepileptic seizure patients. Epilepsia. 2000;41(4):447-452.
16. Durrant J, Rickards H, Cavanna A. Prognosis and outcome predictors in psychogenic nonepileptic seizures. Epilepsy Res Treat. 2011;2011:1-7.
17. Selkirk M, Duncan R, Oto M, et al. Clinical differences between patients with nonepileptic seizures who report antecedent sexual abuse and those who do not. Epilepsia. 2008;49(8):1446-1450.
18. Goldstein L, Chalder T, Chigwedere C, et al. Cognitive-behavioral therapy for psychogenic nonepileptic seizures: a pilot RCT. Neurology. 2010;74(24):1986-1994.
19. LaFrance W, Baird G, Barry J, et al. Multicenter pilot treatment trial for psychogenic nonepileptic seizures. JAMA Psychiatry. 2014;71(9):997.
20. Howlett S, Reuber M. An augmented model of brief psychodynamic interpersonal therapy for patients with nonepileptic seizures. Psychotherapy (Chic). 2009;46(1):125-138.
21. Metin SZ, Ozmen M, Metin B, et al. Treatment with group psychotherapy for chronic psychogenic nonepileptic seizures. Epilepsy Behav. 2013;28(1):91-94.
22. Carlson P, Perry KN. Psychological interventions for psychogenic non-epileptic seizures: a meta-analysis. Seizure. 2017;45:142-150.

Issue
Current Psychiatry - 18(8)
Issue
Current Psychiatry - 18(8)
Page Number
42-47
Page Number
42-47
Publications
Publications
Topics
Article Type
Display Headline
Seizure-like episodes, but is it really epilepsy?
Display Headline
Seizure-like episodes, but is it really epilepsy?
Sections
Disallow All Ads
Content Gating
No Gating (article Unlocked/Free)
Alternative CME
Disqus Comments
Default
Use ProPublica
Hide sidebar & use full width
render the right sidebar.
Article PDF Media

How to avoid ‘checklist’ psychiatry

Article Type
Changed
Thu, 08/22/2019 - 15:34
Display Headline
How to avoid ‘checklist’ psychiatry

To determine whether a patient meets the criteria for a DSM-5 diagnosis, we rely on objective data, direct observations, and individual biopsychosocial factors as well as our patient’s subjective report of symptoms. However, because the line differentiating normal from abnormal emotional responses can sometimes be blurred, we should be prudent when establishing a diagnosis. Specifically, we need to avoid falling into the trap of “checklist” psychiatry—relegating diagnostic assessments to robotic statements about whether patients meet DSM criteria—because this can lead to making diagnoses too quickly or inaccurately.1 Potential consequences of checklist psychiatry include1,2:

  • becoming so “married” to a particular diagnosis that you don’t consider alternative diagnoses
  • labeling patients with a diagnosis that many clinicians may view as pejorative (eg, antisocial personality disorder), which might affect their ability to receive future treatment
  • developing ineffective treatment plans based on an incorrect diagnosis, including exposing patients to medications that could have serious adverse effects
  • performing suicide or violence risk assessments based on inaccurate diagnoses, thereby over- or underestimating the possible risk for an adverse outcome
  • leading patients to assume the identity of the inaccurate diagnosis and possibly viewing themselves as dysfunctional or impaired.

When you are uncertain whether your patient has a diagnosable condition, it can be useful to use the terms “no diagnosis” or “diagnosis deferred.” However, many insurance companies will not reimburse without an actual diagnosis. Therefore, the following tips may be helpful in establishing an accurate diagnosis while avoiding checklist psychiatry.1,2

Ask patients about the degree and duration of impairment in functioning. Although impairment in functioning is a criterion of almost all DSM-5 diagnoses, not all endorsed symptoms warrant a diagnosis. Mild symptoms often resolve spontaneously over time without the need for diagnostic labels or interventions.

Make longitudinal observations. Inter­viewing patients over a long period of time and on multiple occasions can provide data on the consistency of reported symptoms, the presence or absence of behavioral correlates to reported symptomatology, the degree of impairment from the reported symptoms, and the evolution of symptoms.

Collect collateral information. Although we often rely on our patients’ reports of symptoms to establish a diagnosis, this information should not be the sole source. We can obtain a more complete picture if we approach a patient’s family members for their input, including asking about a family history of mental illness or substance use disorders. We can also review prior treatment records and gather observations from clinic or inpatient staff for additional information.

Order laboratory studies. Serum studies and urine toxicology screens provide information that can help form an accurate diagnosis. This information is helpful because certain medical conditions, substance intoxication, and substance withdrawal can mimic psychiatric symptoms.

Continuously re-evaluate your diagnoses. As clinicians, we’d like to provide an accurate diagnosis at the onset of treatment; however, this may not be realistic because the patient’s presentation might change over time. It is paramount that we view diagnoses as evolving, so that we can more readily adjust our approach to treatment, especially when the patient is not benefitting from a well-formulated and comprehensive treatment plan.

Our patients are best served when we take the necessary time to use all resources to conceptualize them as more than a checklist of symptoms.

References

1. Kontos N, Freudenreich O, Querques J. Thoughtful diagnoses: not ‘checklist’ psychiatry. Current Psychiatry. 2007;6(3):112.
2. Frances A. My 12 best tips on psychiatric diagnosis. Psychiatric Times. http://www.psychiatrictimes.com/dsm-5/my-12-best-tips-psychiatric-diagnosis. Published June 17, 2013. Accessed July 19, 2019.

Article PDF
Author and Disclosure Information

Dr. Joshi is Associate Professor of Clinical Psychiatry and Associate Director, Forensic Psychiatry Fellowship, Department of Neuropsychiatry and Behavioral Science, University of South Carolina School of Medicine, Columbia, South Carolina. Dr. Payne is a Forensic Psychiatry Fellow, Prisma Health, Columbia, South Carolina; and is board-certified in addiction psychiatry.

Disclosures
The authors report no financial relationships with any companies whose products are mentioned in this article, or with manufacturers of competing products.

Issue
Current Psychiatry - 18(8)
Publications
Page Number
40-41
Sections
Author and Disclosure Information

Dr. Joshi is Associate Professor of Clinical Psychiatry and Associate Director, Forensic Psychiatry Fellowship, Department of Neuropsychiatry and Behavioral Science, University of South Carolina School of Medicine, Columbia, South Carolina. Dr. Payne is a Forensic Psychiatry Fellow, Prisma Health, Columbia, South Carolina; and is board-certified in addiction psychiatry.

Disclosures
The authors report no financial relationships with any companies whose products are mentioned in this article, or with manufacturers of competing products.

Author and Disclosure Information

Dr. Joshi is Associate Professor of Clinical Psychiatry and Associate Director, Forensic Psychiatry Fellowship, Department of Neuropsychiatry and Behavioral Science, University of South Carolina School of Medicine, Columbia, South Carolina. Dr. Payne is a Forensic Psychiatry Fellow, Prisma Health, Columbia, South Carolina; and is board-certified in addiction psychiatry.

Disclosures
The authors report no financial relationships with any companies whose products are mentioned in this article, or with manufacturers of competing products.

Article PDF
Article PDF

To determine whether a patient meets the criteria for a DSM-5 diagnosis, we rely on objective data, direct observations, and individual biopsychosocial factors as well as our patient’s subjective report of symptoms. However, because the line differentiating normal from abnormal emotional responses can sometimes be blurred, we should be prudent when establishing a diagnosis. Specifically, we need to avoid falling into the trap of “checklist” psychiatry—relegating diagnostic assessments to robotic statements about whether patients meet DSM criteria—because this can lead to making diagnoses too quickly or inaccurately.1 Potential consequences of checklist psychiatry include1,2:

  • becoming so “married” to a particular diagnosis that you don’t consider alternative diagnoses
  • labeling patients with a diagnosis that many clinicians may view as pejorative (eg, antisocial personality disorder), which might affect their ability to receive future treatment
  • developing ineffective treatment plans based on an incorrect diagnosis, including exposing patients to medications that could have serious adverse effects
  • performing suicide or violence risk assessments based on inaccurate diagnoses, thereby over- or underestimating the possible risk for an adverse outcome
  • leading patients to assume the identity of the inaccurate diagnosis and possibly viewing themselves as dysfunctional or impaired.

When you are uncertain whether your patient has a diagnosable condition, it can be useful to use the terms “no diagnosis” or “diagnosis deferred.” However, many insurance companies will not reimburse without an actual diagnosis. Therefore, the following tips may be helpful in establishing an accurate diagnosis while avoiding checklist psychiatry.1,2

Ask patients about the degree and duration of impairment in functioning. Although impairment in functioning is a criterion of almost all DSM-5 diagnoses, not all endorsed symptoms warrant a diagnosis. Mild symptoms often resolve spontaneously over time without the need for diagnostic labels or interventions.

Make longitudinal observations. Inter­viewing patients over a long period of time and on multiple occasions can provide data on the consistency of reported symptoms, the presence or absence of behavioral correlates to reported symptomatology, the degree of impairment from the reported symptoms, and the evolution of symptoms.

Collect collateral information. Although we often rely on our patients’ reports of symptoms to establish a diagnosis, this information should not be the sole source. We can obtain a more complete picture if we approach a patient’s family members for their input, including asking about a family history of mental illness or substance use disorders. We can also review prior treatment records and gather observations from clinic or inpatient staff for additional information.

Order laboratory studies. Serum studies and urine toxicology screens provide information that can help form an accurate diagnosis. This information is helpful because certain medical conditions, substance intoxication, and substance withdrawal can mimic psychiatric symptoms.

Continuously re-evaluate your diagnoses. As clinicians, we’d like to provide an accurate diagnosis at the onset of treatment; however, this may not be realistic because the patient’s presentation might change over time. It is paramount that we view diagnoses as evolving, so that we can more readily adjust our approach to treatment, especially when the patient is not benefitting from a well-formulated and comprehensive treatment plan.

Our patients are best served when we take the necessary time to use all resources to conceptualize them as more than a checklist of symptoms.

To determine whether a patient meets the criteria for a DSM-5 diagnosis, we rely on objective data, direct observations, and individual biopsychosocial factors as well as our patient’s subjective report of symptoms. However, because the line differentiating normal from abnormal emotional responses can sometimes be blurred, we should be prudent when establishing a diagnosis. Specifically, we need to avoid falling into the trap of “checklist” psychiatry—relegating diagnostic assessments to robotic statements about whether patients meet DSM criteria—because this can lead to making diagnoses too quickly or inaccurately.1 Potential consequences of checklist psychiatry include1,2:

  • becoming so “married” to a particular diagnosis that you don’t consider alternative diagnoses
  • labeling patients with a diagnosis that many clinicians may view as pejorative (eg, antisocial personality disorder), which might affect their ability to receive future treatment
  • developing ineffective treatment plans based on an incorrect diagnosis, including exposing patients to medications that could have serious adverse effects
  • performing suicide or violence risk assessments based on inaccurate diagnoses, thereby over- or underestimating the possible risk for an adverse outcome
  • leading patients to assume the identity of the inaccurate diagnosis and possibly viewing themselves as dysfunctional or impaired.

When you are uncertain whether your patient has a diagnosable condition, it can be useful to use the terms “no diagnosis” or “diagnosis deferred.” However, many insurance companies will not reimburse without an actual diagnosis. Therefore, the following tips may be helpful in establishing an accurate diagnosis while avoiding checklist psychiatry.1,2

Ask patients about the degree and duration of impairment in functioning. Although impairment in functioning is a criterion of almost all DSM-5 diagnoses, not all endorsed symptoms warrant a diagnosis. Mild symptoms often resolve spontaneously over time without the need for diagnostic labels or interventions.

Make longitudinal observations. Inter­viewing patients over a long period of time and on multiple occasions can provide data on the consistency of reported symptoms, the presence or absence of behavioral correlates to reported symptomatology, the degree of impairment from the reported symptoms, and the evolution of symptoms.

Collect collateral information. Although we often rely on our patients’ reports of symptoms to establish a diagnosis, this information should not be the sole source. We can obtain a more complete picture if we approach a patient’s family members for their input, including asking about a family history of mental illness or substance use disorders. We can also review prior treatment records and gather observations from clinic or inpatient staff for additional information.

Order laboratory studies. Serum studies and urine toxicology screens provide information that can help form an accurate diagnosis. This information is helpful because certain medical conditions, substance intoxication, and substance withdrawal can mimic psychiatric symptoms.

Continuously re-evaluate your diagnoses. As clinicians, we’d like to provide an accurate diagnosis at the onset of treatment; however, this may not be realistic because the patient’s presentation might change over time. It is paramount that we view diagnoses as evolving, so that we can more readily adjust our approach to treatment, especially when the patient is not benefitting from a well-formulated and comprehensive treatment plan.

Our patients are best served when we take the necessary time to use all resources to conceptualize them as more than a checklist of symptoms.

References

1. Kontos N, Freudenreich O, Querques J. Thoughtful diagnoses: not ‘checklist’ psychiatry. Current Psychiatry. 2007;6(3):112.
2. Frances A. My 12 best tips on psychiatric diagnosis. Psychiatric Times. http://www.psychiatrictimes.com/dsm-5/my-12-best-tips-psychiatric-diagnosis. Published June 17, 2013. Accessed July 19, 2019.

References

1. Kontos N, Freudenreich O, Querques J. Thoughtful diagnoses: not ‘checklist’ psychiatry. Current Psychiatry. 2007;6(3):112.
2. Frances A. My 12 best tips on psychiatric diagnosis. Psychiatric Times. http://www.psychiatrictimes.com/dsm-5/my-12-best-tips-psychiatric-diagnosis. Published June 17, 2013. Accessed July 19, 2019.

Issue
Current Psychiatry - 18(8)
Issue
Current Psychiatry - 18(8)
Page Number
40-41
Page Number
40-41
Publications
Publications
Article Type
Display Headline
How to avoid ‘checklist’ psychiatry
Display Headline
How to avoid ‘checklist’ psychiatry
Sections
Disallow All Ads
Content Gating
No Gating (article Unlocked/Free)
Alternative CME
Disqus Comments
Default
Use ProPublica
Hide sidebar & use full width
render the right sidebar.
Article PDF Media

Child trafficking: How to recognize the signs

Article Type
Changed
Thu, 01/16/2020 - 16:05
Display Headline
Child trafficking: How to recognize the signs

Child trafficking—a modern-day form of slavery that continues to destroy many lives—often is hidden, even from the clinicians who see its victims. Traffickers typically exploit children for labor or commercial sexual work. The signs and symptoms that suggest a child is being trafficked may be less clear than those of the psychiatric illnesses we usually diagnose and treat. In this article, I summarize characteristics that could be helpful to note when you suspect a child is being trafficked, and offer some resources for helping victims.

How to identify possible victims

Children can be trafficked anywhere. The concept of a child being picked up off a street corner is outdated. Trafficking occurs in cities, suburbs, and rural areas. It happens in hotel rooms, at truck stops, on quiet residential streets, and in expensive homes. The internet has made it easier for traffickers to find victims.

Traffickers typically target youth who are emotionally and physically vulnerable. They often seek out teenagers who are undergoing financial hardships, experiencing family conflict, or have survived natural disasters. Many victims are runaways. In 2016, 1 in 6 child runaways reported to the National Center for Missing and Exploited Children were likely victims of trafficking.1 Of those children, 86% were receiving social services support or living in foster homes.

Traffickers are adept at emotional manipulation, which may explain why a child or adolescent might minimize the abuse during a clinical visit. Traffickers shroud the realities of trafficking with notions of love and inclusion. They use several physical and mental schemes to keep children and adolescents in their grip, such as withholding food, sleep, or medical care. Therefore, we should check for signs and symptoms of chronic medical conditions that have gone untreated, malnutrition, or bruises in various stages of healing.

Connecting risk factors for trafficking to dramatic changes in a young patient’s behavior is challenging. These youth often have dropped out of school, lack consistent family support, and spend their nights in search of a warm place to sleep. Their lives are upended. A child who once was more social may be forced into isolation and make excuses for why she no longer spends time with her friends.

In a study of 106 survivors of domestic sex trafficking, approximately 89% of respondents reported depression during depression. Many respondents reported experiencing anxiety (76.4%), nightmares (73.6%), flashbacks (68%), low self-esteem (81.1%), or feelings of shame or guilt (82.1%).2 Almost 88% of respondents said that they saw a doctor or other clinician while being trafficked, but their clinicians were unable to recognize the signs of trafficking. Part of the challenge is that many children and adolescents are not comfortable discussing their situations with clinicians because they may struggle with shame and guilt. Their traffickers also might have convinced them that they are criminals, not victims. These patients also may have an overwhelming fear of their trafficker, being reported to child welfare authorities, being arrested, being deported, or having their traffickers retaliate against their families. Gaining the trust of a patient who is being trafficked is critical, but not easy, because children may be skeptical of a clinician’s promise of confidentiality.

Some signs of trafficking overlap with the psychiatric presentations with which we are more familiar. These patients may abuse drugs or alcohol as means of escape or because their traffickers force them to use substances.2 They may show symptoms of depression or posttraumatic stress disorder (PTSD) and may be disoriented. Other indicators may be more telling, such as if a child or adolescent describes:

  • having no control of their schedules or forms of identification
  • having to work excessively long hours, often to pay off an overwhelming debt
  • having high security measures installed in their place of residence (such as cameras or barred windows).

Continue to: Also, they may be...

 

 

Also, they may be dressed inappropriately for the weather.

We should be concerned when patients’ responses seem coached, if they say they are isolated from their family and community, or if they are submissive or overly timid. In addition, our suspicions should be raised if an accompanying adult guardian insists on sitting in on the appointment or translating for the child. In such instances, we may request that the guardian remain in the waiting area during the appointment so the child will have the opportunity to speak freely.2

How to help a suspected victim

Several local and national organizations help trafficking victims. These organizations provide educational materials and training opportunities for clinicians, as well as direct support for victims. The Homeland Security Blue Campaign advises against confronting a suspected trafficker directly and encourages clinicians to instead report suspected cases to 1-866-347-2423.3

Clinicians can better help children who are trafficked by taking the following 5 steps:

  1. Learn about the risk factors and signs of child trafficking.
  2. Post the National Human Trafficking Hotline (1-888-373-7888) in your waiting room.
  3. Determine if your patient is in danger and needs to be moved to a safe place.
  4. Connect the patient to social service agencies that can provide financial support and housing assistance so he/she doesn’t feel trapped by financial burdens.
  5. Work to rebuild their emotional and physical well-being while treating depression, PTSD, substance abuse, or any other mental illness.
References

1. National Center for Missing and Exploited Childr en. Missing children, state care, and child sex trafficking. http://www.missingkids.com/content/dam/missingkids/pdfs/publications/missingchildrenstatecare.pdf. Accessed June 10, 2019.
2. Lederer LJ, Wetzel CA. The health consequences of sex trafficking and their implications for identifying victims in healthcare facilities. Ann Health Law. 2014;23(1):61-91.
3. Blue Campaign. Identify a victim. US Department of Homeland Security. https://www.dhs.gov/blue-campaign/identify-victim. Accessed June 10, 2019.

Article PDF
Author and Disclosure Information

Dr. Adam is Associate Professor of Clinical Psychiatry, Child and Adolescent Psychiatry, University of Missouri Columbia School of Medicine, Columbia, Missouri.

Disclosure
The author reports no financial relationships with any companies whose products are mentioned in this article, or with manufacturers of competing products.

Issue
Current Psychiatry - 18(8)
Publications
Topics
Page Number
50-51
Sections
Author and Disclosure Information

Dr. Adam is Associate Professor of Clinical Psychiatry, Child and Adolescent Psychiatry, University of Missouri Columbia School of Medicine, Columbia, Missouri.

Disclosure
The author reports no financial relationships with any companies whose products are mentioned in this article, or with manufacturers of competing products.

Author and Disclosure Information

Dr. Adam is Associate Professor of Clinical Psychiatry, Child and Adolescent Psychiatry, University of Missouri Columbia School of Medicine, Columbia, Missouri.

Disclosure
The author reports no financial relationships with any companies whose products are mentioned in this article, or with manufacturers of competing products.

Article PDF
Article PDF

Child trafficking—a modern-day form of slavery that continues to destroy many lives—often is hidden, even from the clinicians who see its victims. Traffickers typically exploit children for labor or commercial sexual work. The signs and symptoms that suggest a child is being trafficked may be less clear than those of the psychiatric illnesses we usually diagnose and treat. In this article, I summarize characteristics that could be helpful to note when you suspect a child is being trafficked, and offer some resources for helping victims.

How to identify possible victims

Children can be trafficked anywhere. The concept of a child being picked up off a street corner is outdated. Trafficking occurs in cities, suburbs, and rural areas. It happens in hotel rooms, at truck stops, on quiet residential streets, and in expensive homes. The internet has made it easier for traffickers to find victims.

Traffickers typically target youth who are emotionally and physically vulnerable. They often seek out teenagers who are undergoing financial hardships, experiencing family conflict, or have survived natural disasters. Many victims are runaways. In 2016, 1 in 6 child runaways reported to the National Center for Missing and Exploited Children were likely victims of trafficking.1 Of those children, 86% were receiving social services support or living in foster homes.

Traffickers are adept at emotional manipulation, which may explain why a child or adolescent might minimize the abuse during a clinical visit. Traffickers shroud the realities of trafficking with notions of love and inclusion. They use several physical and mental schemes to keep children and adolescents in their grip, such as withholding food, sleep, or medical care. Therefore, we should check for signs and symptoms of chronic medical conditions that have gone untreated, malnutrition, or bruises in various stages of healing.

Connecting risk factors for trafficking to dramatic changes in a young patient’s behavior is challenging. These youth often have dropped out of school, lack consistent family support, and spend their nights in search of a warm place to sleep. Their lives are upended. A child who once was more social may be forced into isolation and make excuses for why she no longer spends time with her friends.

In a study of 106 survivors of domestic sex trafficking, approximately 89% of respondents reported depression during depression. Many respondents reported experiencing anxiety (76.4%), nightmares (73.6%), flashbacks (68%), low self-esteem (81.1%), or feelings of shame or guilt (82.1%).2 Almost 88% of respondents said that they saw a doctor or other clinician while being trafficked, but their clinicians were unable to recognize the signs of trafficking. Part of the challenge is that many children and adolescents are not comfortable discussing their situations with clinicians because they may struggle with shame and guilt. Their traffickers also might have convinced them that they are criminals, not victims. These patients also may have an overwhelming fear of their trafficker, being reported to child welfare authorities, being arrested, being deported, or having their traffickers retaliate against their families. Gaining the trust of a patient who is being trafficked is critical, but not easy, because children may be skeptical of a clinician’s promise of confidentiality.

Some signs of trafficking overlap with the psychiatric presentations with which we are more familiar. These patients may abuse drugs or alcohol as means of escape or because their traffickers force them to use substances.2 They may show symptoms of depression or posttraumatic stress disorder (PTSD) and may be disoriented. Other indicators may be more telling, such as if a child or adolescent describes:

  • having no control of their schedules or forms of identification
  • having to work excessively long hours, often to pay off an overwhelming debt
  • having high security measures installed in their place of residence (such as cameras or barred windows).

Continue to: Also, they may be...

 

 

Also, they may be dressed inappropriately for the weather.

We should be concerned when patients’ responses seem coached, if they say they are isolated from their family and community, or if they are submissive or overly timid. In addition, our suspicions should be raised if an accompanying adult guardian insists on sitting in on the appointment or translating for the child. In such instances, we may request that the guardian remain in the waiting area during the appointment so the child will have the opportunity to speak freely.2

How to help a suspected victim

Several local and national organizations help trafficking victims. These organizations provide educational materials and training opportunities for clinicians, as well as direct support for victims. The Homeland Security Blue Campaign advises against confronting a suspected trafficker directly and encourages clinicians to instead report suspected cases to 1-866-347-2423.3

Clinicians can better help children who are trafficked by taking the following 5 steps:

  1. Learn about the risk factors and signs of child trafficking.
  2. Post the National Human Trafficking Hotline (1-888-373-7888) in your waiting room.
  3. Determine if your patient is in danger and needs to be moved to a safe place.
  4. Connect the patient to social service agencies that can provide financial support and housing assistance so he/she doesn’t feel trapped by financial burdens.
  5. Work to rebuild their emotional and physical well-being while treating depression, PTSD, substance abuse, or any other mental illness.

Child trafficking—a modern-day form of slavery that continues to destroy many lives—often is hidden, even from the clinicians who see its victims. Traffickers typically exploit children for labor or commercial sexual work. The signs and symptoms that suggest a child is being trafficked may be less clear than those of the psychiatric illnesses we usually diagnose and treat. In this article, I summarize characteristics that could be helpful to note when you suspect a child is being trafficked, and offer some resources for helping victims.

How to identify possible victims

Children can be trafficked anywhere. The concept of a child being picked up off a street corner is outdated. Trafficking occurs in cities, suburbs, and rural areas. It happens in hotel rooms, at truck stops, on quiet residential streets, and in expensive homes. The internet has made it easier for traffickers to find victims.

Traffickers typically target youth who are emotionally and physically vulnerable. They often seek out teenagers who are undergoing financial hardships, experiencing family conflict, or have survived natural disasters. Many victims are runaways. In 2016, 1 in 6 child runaways reported to the National Center for Missing and Exploited Children were likely victims of trafficking.1 Of those children, 86% were receiving social services support or living in foster homes.

Traffickers are adept at emotional manipulation, which may explain why a child or adolescent might minimize the abuse during a clinical visit. Traffickers shroud the realities of trafficking with notions of love and inclusion. They use several physical and mental schemes to keep children and adolescents in their grip, such as withholding food, sleep, or medical care. Therefore, we should check for signs and symptoms of chronic medical conditions that have gone untreated, malnutrition, or bruises in various stages of healing.

Connecting risk factors for trafficking to dramatic changes in a young patient’s behavior is challenging. These youth often have dropped out of school, lack consistent family support, and spend their nights in search of a warm place to sleep. Their lives are upended. A child who once was more social may be forced into isolation and make excuses for why she no longer spends time with her friends.

In a study of 106 survivors of domestic sex trafficking, approximately 89% of respondents reported depression during depression. Many respondents reported experiencing anxiety (76.4%), nightmares (73.6%), flashbacks (68%), low self-esteem (81.1%), or feelings of shame or guilt (82.1%).2 Almost 88% of respondents said that they saw a doctor or other clinician while being trafficked, but their clinicians were unable to recognize the signs of trafficking. Part of the challenge is that many children and adolescents are not comfortable discussing their situations with clinicians because they may struggle with shame and guilt. Their traffickers also might have convinced them that they are criminals, not victims. These patients also may have an overwhelming fear of their trafficker, being reported to child welfare authorities, being arrested, being deported, or having their traffickers retaliate against their families. Gaining the trust of a patient who is being trafficked is critical, but not easy, because children may be skeptical of a clinician’s promise of confidentiality.

Some signs of trafficking overlap with the psychiatric presentations with which we are more familiar. These patients may abuse drugs or alcohol as means of escape or because their traffickers force them to use substances.2 They may show symptoms of depression or posttraumatic stress disorder (PTSD) and may be disoriented. Other indicators may be more telling, such as if a child or adolescent describes:

  • having no control of their schedules or forms of identification
  • having to work excessively long hours, often to pay off an overwhelming debt
  • having high security measures installed in their place of residence (such as cameras or barred windows).

Continue to: Also, they may be...

 

 

Also, they may be dressed inappropriately for the weather.

We should be concerned when patients’ responses seem coached, if they say they are isolated from their family and community, or if they are submissive or overly timid. In addition, our suspicions should be raised if an accompanying adult guardian insists on sitting in on the appointment or translating for the child. In such instances, we may request that the guardian remain in the waiting area during the appointment so the child will have the opportunity to speak freely.2

How to help a suspected victim

Several local and national organizations help trafficking victims. These organizations provide educational materials and training opportunities for clinicians, as well as direct support for victims. The Homeland Security Blue Campaign advises against confronting a suspected trafficker directly and encourages clinicians to instead report suspected cases to 1-866-347-2423.3

Clinicians can better help children who are trafficked by taking the following 5 steps:

  1. Learn about the risk factors and signs of child trafficking.
  2. Post the National Human Trafficking Hotline (1-888-373-7888) in your waiting room.
  3. Determine if your patient is in danger and needs to be moved to a safe place.
  4. Connect the patient to social service agencies that can provide financial support and housing assistance so he/she doesn’t feel trapped by financial burdens.
  5. Work to rebuild their emotional and physical well-being while treating depression, PTSD, substance abuse, or any other mental illness.
References

1. National Center for Missing and Exploited Childr en. Missing children, state care, and child sex trafficking. http://www.missingkids.com/content/dam/missingkids/pdfs/publications/missingchildrenstatecare.pdf. Accessed June 10, 2019.
2. Lederer LJ, Wetzel CA. The health consequences of sex trafficking and their implications for identifying victims in healthcare facilities. Ann Health Law. 2014;23(1):61-91.
3. Blue Campaign. Identify a victim. US Department of Homeland Security. https://www.dhs.gov/blue-campaign/identify-victim. Accessed June 10, 2019.

References

1. National Center for Missing and Exploited Childr en. Missing children, state care, and child sex trafficking. http://www.missingkids.com/content/dam/missingkids/pdfs/publications/missingchildrenstatecare.pdf. Accessed June 10, 2019.
2. Lederer LJ, Wetzel CA. The health consequences of sex trafficking and their implications for identifying victims in healthcare facilities. Ann Health Law. 2014;23(1):61-91.
3. Blue Campaign. Identify a victim. US Department of Homeland Security. https://www.dhs.gov/blue-campaign/identify-victim. Accessed June 10, 2019.

Issue
Current Psychiatry - 18(8)
Issue
Current Psychiatry - 18(8)
Page Number
50-51
Page Number
50-51
Publications
Publications
Topics
Article Type
Display Headline
Child trafficking: How to recognize the signs
Display Headline
Child trafficking: How to recognize the signs
Sections
Disallow All Ads
Content Gating
No Gating (article Unlocked/Free)
Alternative CME
Disqus Comments
Default
Use ProPublica
Hide sidebar & use full width
render the right sidebar.
Article PDF Media

Rash on both palms

Article Type
Changed
Thu, 08/01/2019 - 14:15
Display Headline
Rash on both palms

Rash on both palms

The FP diagnosed erythema multiforme (EM) in this patient based on the target lesions with central epithelial disruption on his palms. In this case, the EM was due to the herpes simplex outbreak on the patient’s lips (herpes labialis) that had occurred about a week earlier.

EM is a hypersensitivity reaction that is often secondary to infections or medications. Herpes simplex viruses (HSVI and HSV2) are the most common causative agents and have been implicated in ≥ 60% of cases.

The patient did not know that cold sores were due to herpes simplex and most oral HSV is due to HSV1 infection. He acknowledged that he experienced cold sores about every 2 months that were usually related to stress or exposure to intense sunlight. The FP recommended that the patient avoid intense sunlight (midday sun avoidance; wearing sunscreen and hats) and use lip protection with at least an SPF of 15. As the lip lesions were > 90% healed, there was no reason for the FP to prescribe an antiviral agent. The FP did, however, offer a prescription for valacyclovir to be used at the first signs of an oral herpes outbreak to avoid another case of EM (2000 mg by mouth every 12 hours x 2 doses). For symptomatic relief of the EM, the physician prescribed a 15 g tube of 0.1% triamcinolone cream to be applied to the lesions twice daily.

Photo courtesy of the University of Texas Health Sciences Center, Division of Dermatology and text for Photo Rounds Friday courtesy of Richard P. Usatine, MD. This case was adapted from: Milana C, Smith M. Erythema multiforme, Stevens-Johnson syndrome, and toxic epidermal necrolysis. In: Usatine R, Smith M, Mayeaux EJ, et al, eds. Color Atlas and Synopsis of Family Medicine. 3rd ed. New York, NY: McGraw-Hill; 2019:1161-1168.

To learn more about the 3rd edition of the Color Atlas and Synopsis of Family Medicine, see: https://www.amazon.com/Color-Atlas-Synopsis-Family-Medicine/dp/1259862046/

You can get the Color Atlas of Family Medicine app by clicking on this link: usatinemedia.com

Issue
The Journal of Family Practice - 68(6)
Publications
Topics
Sections

Rash on both palms

The FP diagnosed erythema multiforme (EM) in this patient based on the target lesions with central epithelial disruption on his palms. In this case, the EM was due to the herpes simplex outbreak on the patient’s lips (herpes labialis) that had occurred about a week earlier.

EM is a hypersensitivity reaction that is often secondary to infections or medications. Herpes simplex viruses (HSVI and HSV2) are the most common causative agents and have been implicated in ≥ 60% of cases.

The patient did not know that cold sores were due to herpes simplex and most oral HSV is due to HSV1 infection. He acknowledged that he experienced cold sores about every 2 months that were usually related to stress or exposure to intense sunlight. The FP recommended that the patient avoid intense sunlight (midday sun avoidance; wearing sunscreen and hats) and use lip protection with at least an SPF of 15. As the lip lesions were > 90% healed, there was no reason for the FP to prescribe an antiviral agent. The FP did, however, offer a prescription for valacyclovir to be used at the first signs of an oral herpes outbreak to avoid another case of EM (2000 mg by mouth every 12 hours x 2 doses). For symptomatic relief of the EM, the physician prescribed a 15 g tube of 0.1% triamcinolone cream to be applied to the lesions twice daily.

Photo courtesy of the University of Texas Health Sciences Center, Division of Dermatology and text for Photo Rounds Friday courtesy of Richard P. Usatine, MD. This case was adapted from: Milana C, Smith M. Erythema multiforme, Stevens-Johnson syndrome, and toxic epidermal necrolysis. In: Usatine R, Smith M, Mayeaux EJ, et al, eds. Color Atlas and Synopsis of Family Medicine. 3rd ed. New York, NY: McGraw-Hill; 2019:1161-1168.

To learn more about the 3rd edition of the Color Atlas and Synopsis of Family Medicine, see: https://www.amazon.com/Color-Atlas-Synopsis-Family-Medicine/dp/1259862046/

You can get the Color Atlas of Family Medicine app by clicking on this link: usatinemedia.com

Rash on both palms

The FP diagnosed erythema multiforme (EM) in this patient based on the target lesions with central epithelial disruption on his palms. In this case, the EM was due to the herpes simplex outbreak on the patient’s lips (herpes labialis) that had occurred about a week earlier.

EM is a hypersensitivity reaction that is often secondary to infections or medications. Herpes simplex viruses (HSVI and HSV2) are the most common causative agents and have been implicated in ≥ 60% of cases.

The patient did not know that cold sores were due to herpes simplex and most oral HSV is due to HSV1 infection. He acknowledged that he experienced cold sores about every 2 months that were usually related to stress or exposure to intense sunlight. The FP recommended that the patient avoid intense sunlight (midday sun avoidance; wearing sunscreen and hats) and use lip protection with at least an SPF of 15. As the lip lesions were > 90% healed, there was no reason for the FP to prescribe an antiviral agent. The FP did, however, offer a prescription for valacyclovir to be used at the first signs of an oral herpes outbreak to avoid another case of EM (2000 mg by mouth every 12 hours x 2 doses). For symptomatic relief of the EM, the physician prescribed a 15 g tube of 0.1% triamcinolone cream to be applied to the lesions twice daily.

Photo courtesy of the University of Texas Health Sciences Center, Division of Dermatology and text for Photo Rounds Friday courtesy of Richard P. Usatine, MD. This case was adapted from: Milana C, Smith M. Erythema multiforme, Stevens-Johnson syndrome, and toxic epidermal necrolysis. In: Usatine R, Smith M, Mayeaux EJ, et al, eds. Color Atlas and Synopsis of Family Medicine. 3rd ed. New York, NY: McGraw-Hill; 2019:1161-1168.

To learn more about the 3rd edition of the Color Atlas and Synopsis of Family Medicine, see: https://www.amazon.com/Color-Atlas-Synopsis-Family-Medicine/dp/1259862046/

You can get the Color Atlas of Family Medicine app by clicking on this link: usatinemedia.com

Issue
The Journal of Family Practice - 68(6)
Issue
The Journal of Family Practice - 68(6)
Publications
Publications
Topics
Article Type
Display Headline
Rash on both palms
Display Headline
Rash on both palms
Sections
Disallow All Ads
Content Gating
No Gating (article Unlocked/Free)
Alternative CME
Disqus Comments
Default
Gate On Date
Wed, 07/31/2019 - 15:15
Un-Gate On Date
Wed, 07/31/2019 - 15:15
Use ProPublica
CFC Schedule Remove Status
Wed, 07/31/2019 - 15:15
Hide sidebar & use full width
render the right sidebar.

Diagnosis and management of gastric intestinal metaplasia in the United States

Article Type
Changed
Thu, 08/01/2019 - 00:01

Introduction

Despite a global decline in the incidence of gastric cancer over the past 3 decades, it remains the fifth most commonly diagnosed cancer and the third most common cause of cancer deaths worldwide.1 In the United States it is the fourth most commonly diagnosed GI malignancy, after colorectal, pancreas, and liver cancer. The prevalence remains high in Latin America and Asia, which has implications in the United States because of growing Hispanic and Asian populations.2,3 In recent years, a change in the trend of gastric cancer among non-Hispanic whites has been observed, particularly in women younger than 50 years old.4 Gastric intestinal metaplasia has been recognized worldwide as a premalignant precursor to gastric cancer, but currently, there are limited U.S. guidelines, leading to controversy over management of this condition.5

Etiology

Gastric adenocarcinomas are classified into two subcategories based on location (cardia and noncardia) and histology (intestinal and diffuse types).6,7 Atrophic gastritis and gastric intestinal metaplasia (GIM) are considered precursors of intestinal-type noncardia gastric adenocarcinoma. The Correa cascade is a commonly accepted precancer sequence for noncardia gastric adenocarcinoma that describes mucosal changes from inflammation to atrophy to metaplasia to intraepithelial neoplasia and culminating in carcinoma.8,9 It has been observed that GIM may be the histologic change prior to the development of dysplasia and over 50% of patients with high-grade dysplasia will progress to adenocarcinoma.10-12 In the United States, GIM has the highest prevalence in African Americans, Hispanics, and East Asians, with the overall GIM prevalence regardless of ethnicity reported from 3.05% to 19.2%.5,13

Risk factors and subclassification

Replacement of the foveolar and/or glandular epithelium in the oxyntic and antral mucosa by intestinal epithelium results in GIM. It can be focal when limited to one region of the stomach or extensive when two or more regions are involved.14 The main risk factors for GIM development are Helicobacter pylori infection, tobacco, alcohol consumption, high salt intake, and chronic bile reflux.15,16 Additional risks for developing gastric cancer include older age, certain ethnicities, and male sex.17

Dr. Diana Curras-Martin, internal medicine resident at Hackensack Meridian Jersey Shore University Medical Center
Dr. Diana Curras-Martin

CagA strains of H. pylori can promote carcinogenesis by inducing a mitogenic cellular response and downregulating cell adhesion.18,19 Less carcinogenic risk is associated with H. pylori Cag-A negative strains; however, they also have oncogenic potential mediated by expression of babA2 and vacA genes.20 Hence, the combination of multiple virulent factors encoded in babA2, CagA, and vacA genes has been associated with increased risk of GIM, inflammation, and development of gastric cancer.15 The clinical usefulness of genotyping H. pylori strains specifically to survey precancerous gastric lesions remains to be seen because of a lack of sufficient clinical studies. In addition, genotyping H. pylori is not commonly performed as part of clinical practice.

The loss of parietal cells seen in atrophic gastritis due to chronic H. pylori infection has been linked to the development of metaplasia due to possible loss of differentiation-promoting factors. As a result, metaplastic cells emerge that express spasmolytic polypeptide (SP or TFF2); hence, this type of metaplasia is referred to as spasmolytic polypeptide–expressing metaplasia (SPEM). The cellular mechanism that may explain a precursor role of SPEM in the development of GIM remains unknown.14 A second competing theory for the development of GIM is the clonal expansion of stem cells in the gastric isthmus that can lead to dysplasia and cancer development.14

Dr. Susana Gonzalez, assistant professor of medicine, division of gastroenterology and hepatology, Weill Cornell Medicine, New York Presbyterian Hospital-Cornell
Dr. Susana Gonzalez

On the basis of histological similarities with small intestinal or colonic epithelium, GIM can be further classified into complete or incomplete intestinal metaplasia.21 Complete intestinal metaplasia most closely resembles small intestinal epithelium with a brush border and goblet cells. Incomplete intestinal metaplasia resembles the colonic epithelium and lacks a brush border. A second classification further classifies GIM into three subtypes: Type I contains nonsecretory absorptive cells and sialomucin secreting goblet cells; type II has few absorptive cells, columnar cells secreting sialomucin, goblet cells secreting mainly sialomucin but some sulphomucin, and presence of Paneth cells; and type III consists of columnar cells secreting predominantly sulphomucin, goblet cells secreting sialomucin or sulphomucin, and absence of Paneth cells.15,22 In this subclassification, type I GIM is known as complete GIM and types II and III as incomplete GIM.23-25

Multiple studies performed outside of the United States have shown a higher progression risk to gastric adenocarcinoma in incomplete intestinal metaplasia, or type III intestinal metaplasia.26-32 Also, the risk of gastric cancer has been demonstrated to be higher among patients with a greater area of metaplasia and extensive intestinal metaplasia, defined as GIM in both the antrum and corpus.33,34 Hence, the extent of the metaplasia determined with mapping biopsies, regardless of the subtype, should also be incorporated into the risk assessment of the patient. Currently, a major limitation in the United States is a standardized method of pathologic reporting including subclassification of incomplete versus complete intestinal metaplasia.
 

 

 

Which patients to screen

Understanding this sequence of carcinogenesis offers a potential window for screening and surveillance. Subsequently, early detection of precancerous mucosal changes would be more amenable for endoscopic submucosal dissection (ESD).35,36 Currently, U.S. society guidelines do not specifically address the management of GIM. The American Society for Gastrointestinal Endoscopy (ASGE) guidelines for management of premalignant and malignant conditions of the stomach recommend surveillance in individuals with a family history of gastric cancer or of high-risk ethnic background but with no specific optimal surveillance interval.37 Also, H. pylori treatment is recommended if identified, but empiric treatment in GIM was felt to be controversial. The AGA recently sought comments on a proposed new guideline for the management of GIM. This guideline should be released after the comment period and help address management of GIM in the United States. In April of 2019, the European Society of Gastrointestinal Endoscopy (ESGE) updated the management of epithelial precancerous conditions and lesions in the stomach (MAPS II) guideline.38 The MAPS II guideline identifies atrophic gastritis and intestinal metaplasia as precancerous lesions. In patients with moderate to marked atrophy or GIM affecting both antral and body mucosa, ESGE recommends endoscopic surveillance with high-definition chromoendoscopy, mapping, and guided biopsies or at least two biopsies taken separately at the lesser and greater curvature of the antrum and body. H. pylori eradication was recommended if the patient tested positive.

Furthermore, MAPS II proposed replacing atrophic gastritis (AG) in the Operative Link on Gastritis Assessment (OLGA) staging by GIM (OLGIM) as it is considered a more reliable predictor of an individual’s gastric neoplasia risk, based on the interobserver agreement kappa value 0.6 for AG versus 0.9 for GIM.39 Five biopsies (two from the antrum, two from the corpus, and one from the incisura angularis) are needed for the OLGA/OLGIM score system to be considered an accurate predictor of this risk.39 This is supported by the early findings of gastric atrophy and GIM in the incisura angularis.23 In addition, for patients with GIM only in either the antrum or the body, a family history of gastric cancer, incomplete GIM, autoimmune gastritis, or persistent H. pylori infection was felt to increase the risk to warrant surveillance every 3 years. In those patients with atrophy or GIM in both the antrum and body with a first-degree relative with gastric cancer, surveillance was recommended every 1-2 years. Patients with any dysplasia and a visible lesion should have staging and resection. With no visible lesion, a follow-up endoscopy should be performed in 6 months with high-grade dysplasia and with low-grade dysplasia a repeat in 12 months. Patients with mild to moderate atrophy in the antrum and no intestinal metaplasia were not felt to warrant any further surveillance. (See Figure 1.)

Figure 1. Surveillance recommendations according to MAPS II, April 2019
A recent study explored the cost-effectiveness of noncardia gastric cancer screening in the United States stratified by race or ethnicity with a time horizon of 30 years. The study determined that performing endoscopic screening with mapping biopsies in high-risk patients (non-Hispanic black, Hispanic, and Asian individuals) from 50 years of age with continued surveillance only when indicated would be cost effective compared to a no-screening strategy. These patients had sampling performed via an updated Sydney protocol. If GIM was found, the patients would be enrolled into a 3-year surveillance program. Whereas if dysplasia was present, the patients would undergo endoscopic submucosal dissection or surgical resection and continue a postresection surveillance schedule.40,41
 

 

 

How to screen

Previous studies have found a poor correlation between the endoscopic determination of gastric atrophy and the histologic diagnosis.42 Several studies also found that gastric cancer was missed on initial endoscopic examinations. Sensitivity of endoscopy to detect gastric cancer has ranged from 77% to 93%.43,44 In the United States, there is a lack of standardized quality indicators for upper endoscopy exams. The ESGE has suggested several performance measures to ensure a quality endoscopy exam, including accurate photo documentation, sufficient procedure time of at least 7 minutes, adherence to biopsy protocols, and low complication rates.45 In Asia, a systematic screening protocol is used for photo documentation, and simple techniques such as adequate air insufflation and irrigation to remove mucus are routinely used to improve the endoscopy exam.46,47 The mean time of an endoscopy exam has also been found to increase the detection of neoplastic lesions, as slow endoscopists – with a mean exam duration of 8.6 ± 4.2 min during upper endoscopy – detected threefold more neoplastic lesions than did fast endoscopists.48

A standardized biopsy approach is also important when screening patients. The updated Sydney protocol has been suggested for mapping the stomach to screen for atrophy and GIM. This protocol recommends two biopsies from the antrum (at the lesser and greater curvature), two from the body (at the lesser and greater curvature), and one from the incisura.23 This biopsy protocol was also suggested in the recent MAPS II update, with the biopsy of the incisura felt to be an additional biopsy left to the discretion of the endoscopist. Notably, abnormal appearing mucosal areas should be biopsied separately from the mapping biopsies.

High-definition endoscopy with virtual chromoendoscopy is felt to be better than white-light endoscopy alone at detecting precancerous gastric lesions.38 (See Figure 2.)

Figure 2. A. High definition white light endoscopy of patient with diffuse gastric intestinal metaplasia. B. NBI image of patient with diffuse GIM shows ridge and villous appearance. C. High powered H&E of biopsy shows intestinal metaplasia.
Courtesy Diana Curras-Martin, MD, and Susana Gonzalez, MD
Figure 2. A. High definition white light endoscopy of patient with diffuse gastric intestinal metaplasia. B. NBI image of patient with diffuse GIM shows ridge and villous appearance. C. High powered H&E of biopsy shows intestinal metaplasia.

In particular, narrow-band imaging (NBI) has been studied and found to increase the diagnostic yield of GIM and dysplasia compared with white light alone.49 Several studies have shown an increased accuracy for the detection of GIM with magnification NBI.50-52 An unfortunate limitation is the geographic availability of magnification NBI: It is not available in the United States. A multicenter study in Portugal developed a new classification system for the appearance of precancerous lesions with NBI and tested its accuracy in endoscopists with a wide range of NBI experience. An abnormal mucosal pattern that showed light blue crests/regular ridge or a tubulovillous appearance and a regular mucosal pattern was found with GIM. An irregular vascular pattern with a white opaque substance and an absent or irregular mucosal pattern was most often found with dysplasia. Furthermore, the reproducibility of these patterns was high between endoscopists.53 Multiple studies have been performed on additional imaging technologies to enhance the detection of gastric neoplasia; however, these technologies are still investigational and currently not recommended for screening.54-57

Serum pepsinogens have been studied in Europe and Asia as noninvasive indicators of gastric atrophy to determine who should be screened with endoscopy.58 A low serum pepsinogen I level below 70 ng/mL and pepsinogen I/II ratio below 3 has generally been used to detect atrophic gastritis and at-risk populations. However, the studies performed in Europe and Asia used different methods for quantifying pepsinogen levels. Therefore, cutoff values cannot be generalized for all assays and should be validated for the specific tests used.38
 

 

 

Summary

Gastric atrophy and gastric intestinal metaplasia are considered precancerous lesions with an increased risk of development of gastric cancer. H. pylori is a major risk factor for the development of GIM. The extent of GIM as well as the presence of incomplete intestinal metaplasia, or type III intestinal metaplasia has been found to have the highest gastric cancer risk. Currently, in the United States, specific guidelines on endoscopic screening and surveillance for noncardia gastric adenocarcinoma based on histological subtype of GIM, location, and extension are lacking. The ESGE recently updated guidelines that recommend surveillance of patients with extensive atrophy and intestinal metaplasia or with a significant family history. Location and extension of intestinal metaplasia plays a role in increased risk. Screening should include a standardized upper endoscopy approach with high-definition white- light endoscopy and NBI, at least a 7-minute examination, adequate insufflation and cleaning, adequate photo documentation, and a standardized biopsy protocol. Further studies are needed to determine an appropriate surveillance interval and standardized pathology reporting approach as well.

Diana Curras-Martin MD, is an internal medicine resident at Hackensack Meridian Jersey Shore University Medical Center. Susana Gonzalez, MD, is assistant professor of medicine in the division of gastroenterology and hepatology (@WCM_GI), Weill Cornell Medicine, New York Presbyterian Hospital–Cornell. 

 

 

References

1. Bray F et al. CA Cancer J Clin. 2018;68(6):394-424.

2. Global Burden of Disease Cancer Collaboration et al. JAMA Oncol. 2018;4(11):1553-68.

3. Balakrishnan M et al. Curr Gastroenterol Rep. 2017;19(8):36.

4. Anderson WF et al. J Natl Cancer Inst. 2018;110(6):608-15.

5. Trieu JA et al. Dig Dis Sci. 2019;64(5):1079-88.

6. Lauren P. Acta Pathol Microbiol Scand. 1965;64:31-49.

7. Correa P, Schneider BG. Cancer Epidemiol Biomarkers Prev. 2005;14(8):1865-8.

8. Correa P. Cancer Res. 1992;52(24):6735-40.

9. Correa P, Piazuelo MB. J Dig Dis. 2012;13(1):2-9.

10. Correa P et al. J Natl Cancer Inst. 1970;44(2):297-306.

11. Correa P. Semin Oncol. 1985;12(1):2-10.

12. Rugge M et al. Hum Pathol. 1991;22(10):1002-8.

13. Simko V et al. Bratisl Lek Listy. 2015;116(1):3-8.

14. Giroux V, Rustgi AK. Nat Rev Cancer. 2017;17(10):594-604.

15. Jencks DS et al. Gastroenterol Hepatol (N Y). 2018;14(2):92-101.

16. Amieva M, Peek RM Jr. Gastroenterology. 2016;150(1):64-78.

17. Karimi P et al. Cancer Epidemiol Biomarkers Prev. 2014;23(5):700-13.

18. Hatakeyama M. Proc Jpn Acad Ser B Phys Biol Sci. 2017;93(4):196-219.

19. Tsutsumi R et al. Mol Cell Biol. 2006;26(1):261-76.

20. Kikuchi S et al. Am J Gastroenterol. 1999;94(12):3455-9.

21. Jass JR, Filipe MI. Histopathology. 1980;4(3):271-9.

22. Jass JR, Filipe MI. Histochem J. 1981;13(6):931-9.

23. Dixon MF et al. Am J Surg Pathol. 1996;20(10):1161-81.

24. Kang KP et al. J Gastroenterol Hepatol. 2009;24(1):140-8.

25. Gonzalez CA et al. Int J Cancer. 2010;127(11):2654-60.

26. Filipe MI et al. Gut. 1985;26(12):1319-26.

27. Filipe MI et al. Int J Cancer. 1994;57(3):324-9.

28. Gonzalez CA et al. J Gastroenterol Hepatol. 2016;31(5):953-8.

29. Cassaro M et al. Am J Gastroenterol. 2000;95(6):1431-8.

30. Shao L et al. Int J Cancer. Apr 29. 2018.

31. Stemmermann GN. Cancer. 1994;74(2):556-64.

32. Gonzalez CA et al. Int J Cancer. 2013;133(5):1023-32.

33. Reddy KM et al. Clin Gastroenterol Hepatol. 2016;14(10):1420-5.

34. Tava F et al. Hum Pathol. 2006;37(11):1489-97.

35. Fernandez-Esparrach G et al. Rev Esp Enferm Dig. 2014;106(2):120-32.

36. Ono H et al. Dig Endosc. 2016;28(1):3-15.

37. Evans JA, DeWitt JM. Gastrointest Endosc. 2016;83(1):274.

38. Pimentel-Nunes P et al. Endoscopy. 2019;51(4):365-88.

39. Capelle LG et al. Gastrointest Endosc. 2010;71(7):1150-8.

40. Saumoy M et al. Gastroenterology. 2018;155(3):648-60.

41. Gupta N et al. Gastrointest Endosc. 2011;74(3):610-24 e612.

42. Eshmuratov A et al. Dig Dis Sci. 2010;55(5):1364-75.

43. Nam JH et al. Cancer. 2012;118(20):4953-60.

44. Amin A et al. J R Coll Surg Edinb. 2002;47(5):681-4.

45. Bisschops R et al. United European Gastroenterol J. 2016;4(5):629-56.

46. Uedo N et al. Gastroenterol Clin North Am. 2013;42(2):317-35.

47. Yao K. Ann Gastroenterol. 2013;26(1):11-22.

48. Teh JL et al. Clin Gastroenterol Hepatol. 2015;13(3):480-7 e482.

49. Capelle LG et al. Dig Dis Sci. 2010;55(12):3442-8.

50. Bansal A et al. Gastrointest Endosc. 2008;67(2):210-6.

51. Tahara T et al. Gastrointest Endosc. 2009;70(2):246-53.

52. Uedo N et al. Endoscopy. 2006;38(8):819-24.

53. Pimentel-Nunes P et al. Endoscopy. 2012;44(3):236-46.

54. Kato M et al. Gastrointest Endosc. 2009;70(5):899-906.

55. Nishimura J et al. Gastroenterol Res Pract. 2014;2014:819395.

56. Dohi O et al. Gastrointest Endosc. 2019;89(1):47-57.

57. Osawa H et al. World J Gastrointest Endosc. 2012;4(8):356-61.

58. Pasechnikov V et al. World J Gastroenterol. 2014;20(38):13842-62.

 

Publications
Topics
Sections

Introduction

Despite a global decline in the incidence of gastric cancer over the past 3 decades, it remains the fifth most commonly diagnosed cancer and the third most common cause of cancer deaths worldwide.1 In the United States it is the fourth most commonly diagnosed GI malignancy, after colorectal, pancreas, and liver cancer. The prevalence remains high in Latin America and Asia, which has implications in the United States because of growing Hispanic and Asian populations.2,3 In recent years, a change in the trend of gastric cancer among non-Hispanic whites has been observed, particularly in women younger than 50 years old.4 Gastric intestinal metaplasia has been recognized worldwide as a premalignant precursor to gastric cancer, but currently, there are limited U.S. guidelines, leading to controversy over management of this condition.5

Etiology

Gastric adenocarcinomas are classified into two subcategories based on location (cardia and noncardia) and histology (intestinal and diffuse types).6,7 Atrophic gastritis and gastric intestinal metaplasia (GIM) are considered precursors of intestinal-type noncardia gastric adenocarcinoma. The Correa cascade is a commonly accepted precancer sequence for noncardia gastric adenocarcinoma that describes mucosal changes from inflammation to atrophy to metaplasia to intraepithelial neoplasia and culminating in carcinoma.8,9 It has been observed that GIM may be the histologic change prior to the development of dysplasia and over 50% of patients with high-grade dysplasia will progress to adenocarcinoma.10-12 In the United States, GIM has the highest prevalence in African Americans, Hispanics, and East Asians, with the overall GIM prevalence regardless of ethnicity reported from 3.05% to 19.2%.5,13

Risk factors and subclassification

Replacement of the foveolar and/or glandular epithelium in the oxyntic and antral mucosa by intestinal epithelium results in GIM. It can be focal when limited to one region of the stomach or extensive when two or more regions are involved.14 The main risk factors for GIM development are Helicobacter pylori infection, tobacco, alcohol consumption, high salt intake, and chronic bile reflux.15,16 Additional risks for developing gastric cancer include older age, certain ethnicities, and male sex.17

Dr. Diana Curras-Martin, internal medicine resident at Hackensack Meridian Jersey Shore University Medical Center
Dr. Diana Curras-Martin

CagA strains of H. pylori can promote carcinogenesis by inducing a mitogenic cellular response and downregulating cell adhesion.18,19 Less carcinogenic risk is associated with H. pylori Cag-A negative strains; however, they also have oncogenic potential mediated by expression of babA2 and vacA genes.20 Hence, the combination of multiple virulent factors encoded in babA2, CagA, and vacA genes has been associated with increased risk of GIM, inflammation, and development of gastric cancer.15 The clinical usefulness of genotyping H. pylori strains specifically to survey precancerous gastric lesions remains to be seen because of a lack of sufficient clinical studies. In addition, genotyping H. pylori is not commonly performed as part of clinical practice.

The loss of parietal cells seen in atrophic gastritis due to chronic H. pylori infection has been linked to the development of metaplasia due to possible loss of differentiation-promoting factors. As a result, metaplastic cells emerge that express spasmolytic polypeptide (SP or TFF2); hence, this type of metaplasia is referred to as spasmolytic polypeptide–expressing metaplasia (SPEM). The cellular mechanism that may explain a precursor role of SPEM in the development of GIM remains unknown.14 A second competing theory for the development of GIM is the clonal expansion of stem cells in the gastric isthmus that can lead to dysplasia and cancer development.14

Dr. Susana Gonzalez, assistant professor of medicine, division of gastroenterology and hepatology, Weill Cornell Medicine, New York Presbyterian Hospital-Cornell
Dr. Susana Gonzalez

On the basis of histological similarities with small intestinal or colonic epithelium, GIM can be further classified into complete or incomplete intestinal metaplasia.21 Complete intestinal metaplasia most closely resembles small intestinal epithelium with a brush border and goblet cells. Incomplete intestinal metaplasia resembles the colonic epithelium and lacks a brush border. A second classification further classifies GIM into three subtypes: Type I contains nonsecretory absorptive cells and sialomucin secreting goblet cells; type II has few absorptive cells, columnar cells secreting sialomucin, goblet cells secreting mainly sialomucin but some sulphomucin, and presence of Paneth cells; and type III consists of columnar cells secreting predominantly sulphomucin, goblet cells secreting sialomucin or sulphomucin, and absence of Paneth cells.15,22 In this subclassification, type I GIM is known as complete GIM and types II and III as incomplete GIM.23-25

Multiple studies performed outside of the United States have shown a higher progression risk to gastric adenocarcinoma in incomplete intestinal metaplasia, or type III intestinal metaplasia.26-32 Also, the risk of gastric cancer has been demonstrated to be higher among patients with a greater area of metaplasia and extensive intestinal metaplasia, defined as GIM in both the antrum and corpus.33,34 Hence, the extent of the metaplasia determined with mapping biopsies, regardless of the subtype, should also be incorporated into the risk assessment of the patient. Currently, a major limitation in the United States is a standardized method of pathologic reporting including subclassification of incomplete versus complete intestinal metaplasia.
 

 

 

Which patients to screen

Understanding this sequence of carcinogenesis offers a potential window for screening and surveillance. Subsequently, early detection of precancerous mucosal changes would be more amenable for endoscopic submucosal dissection (ESD).35,36 Currently, U.S. society guidelines do not specifically address the management of GIM. The American Society for Gastrointestinal Endoscopy (ASGE) guidelines for management of premalignant and malignant conditions of the stomach recommend surveillance in individuals with a family history of gastric cancer or of high-risk ethnic background but with no specific optimal surveillance interval.37 Also, H. pylori treatment is recommended if identified, but empiric treatment in GIM was felt to be controversial. The AGA recently sought comments on a proposed new guideline for the management of GIM. This guideline should be released after the comment period and help address management of GIM in the United States. In April of 2019, the European Society of Gastrointestinal Endoscopy (ESGE) updated the management of epithelial precancerous conditions and lesions in the stomach (MAPS II) guideline.38 The MAPS II guideline identifies atrophic gastritis and intestinal metaplasia as precancerous lesions. In patients with moderate to marked atrophy or GIM affecting both antral and body mucosa, ESGE recommends endoscopic surveillance with high-definition chromoendoscopy, mapping, and guided biopsies or at least two biopsies taken separately at the lesser and greater curvature of the antrum and body. H. pylori eradication was recommended if the patient tested positive.

Furthermore, MAPS II proposed replacing atrophic gastritis (AG) in the Operative Link on Gastritis Assessment (OLGA) staging by GIM (OLGIM) as it is considered a more reliable predictor of an individual’s gastric neoplasia risk, based on the interobserver agreement kappa value 0.6 for AG versus 0.9 for GIM.39 Five biopsies (two from the antrum, two from the corpus, and one from the incisura angularis) are needed for the OLGA/OLGIM score system to be considered an accurate predictor of this risk.39 This is supported by the early findings of gastric atrophy and GIM in the incisura angularis.23 In addition, for patients with GIM only in either the antrum or the body, a family history of gastric cancer, incomplete GIM, autoimmune gastritis, or persistent H. pylori infection was felt to increase the risk to warrant surveillance every 3 years. In those patients with atrophy or GIM in both the antrum and body with a first-degree relative with gastric cancer, surveillance was recommended every 1-2 years. Patients with any dysplasia and a visible lesion should have staging and resection. With no visible lesion, a follow-up endoscopy should be performed in 6 months with high-grade dysplasia and with low-grade dysplasia a repeat in 12 months. Patients with mild to moderate atrophy in the antrum and no intestinal metaplasia were not felt to warrant any further surveillance. (See Figure 1.)

Figure 1. Surveillance recommendations according to MAPS II, April 2019
A recent study explored the cost-effectiveness of noncardia gastric cancer screening in the United States stratified by race or ethnicity with a time horizon of 30 years. The study determined that performing endoscopic screening with mapping biopsies in high-risk patients (non-Hispanic black, Hispanic, and Asian individuals) from 50 years of age with continued surveillance only when indicated would be cost effective compared to a no-screening strategy. These patients had sampling performed via an updated Sydney protocol. If GIM was found, the patients would be enrolled into a 3-year surveillance program. Whereas if dysplasia was present, the patients would undergo endoscopic submucosal dissection or surgical resection and continue a postresection surveillance schedule.40,41
 

 

 

How to screen

Previous studies have found a poor correlation between the endoscopic determination of gastric atrophy and the histologic diagnosis.42 Several studies also found that gastric cancer was missed on initial endoscopic examinations. Sensitivity of endoscopy to detect gastric cancer has ranged from 77% to 93%.43,44 In the United States, there is a lack of standardized quality indicators for upper endoscopy exams. The ESGE has suggested several performance measures to ensure a quality endoscopy exam, including accurate photo documentation, sufficient procedure time of at least 7 minutes, adherence to biopsy protocols, and low complication rates.45 In Asia, a systematic screening protocol is used for photo documentation, and simple techniques such as adequate air insufflation and irrigation to remove mucus are routinely used to improve the endoscopy exam.46,47 The mean time of an endoscopy exam has also been found to increase the detection of neoplastic lesions, as slow endoscopists – with a mean exam duration of 8.6 ± 4.2 min during upper endoscopy – detected threefold more neoplastic lesions than did fast endoscopists.48

A standardized biopsy approach is also important when screening patients. The updated Sydney protocol has been suggested for mapping the stomach to screen for atrophy and GIM. This protocol recommends two biopsies from the antrum (at the lesser and greater curvature), two from the body (at the lesser and greater curvature), and one from the incisura.23 This biopsy protocol was also suggested in the recent MAPS II update, with the biopsy of the incisura felt to be an additional biopsy left to the discretion of the endoscopist. Notably, abnormal appearing mucosal areas should be biopsied separately from the mapping biopsies.

High-definition endoscopy with virtual chromoendoscopy is felt to be better than white-light endoscopy alone at detecting precancerous gastric lesions.38 (See Figure 2.)

Figure 2. A. High definition white light endoscopy of patient with diffuse gastric intestinal metaplasia. B. NBI image of patient with diffuse GIM shows ridge and villous appearance. C. High powered H&E of biopsy shows intestinal metaplasia.
Courtesy Diana Curras-Martin, MD, and Susana Gonzalez, MD
Figure 2. A. High definition white light endoscopy of patient with diffuse gastric intestinal metaplasia. B. NBI image of patient with diffuse GIM shows ridge and villous appearance. C. High powered H&E of biopsy shows intestinal metaplasia.

In particular, narrow-band imaging (NBI) has been studied and found to increase the diagnostic yield of GIM and dysplasia compared with white light alone.49 Several studies have shown an increased accuracy for the detection of GIM with magnification NBI.50-52 An unfortunate limitation is the geographic availability of magnification NBI: It is not available in the United States. A multicenter study in Portugal developed a new classification system for the appearance of precancerous lesions with NBI and tested its accuracy in endoscopists with a wide range of NBI experience. An abnormal mucosal pattern that showed light blue crests/regular ridge or a tubulovillous appearance and a regular mucosal pattern was found with GIM. An irregular vascular pattern with a white opaque substance and an absent or irregular mucosal pattern was most often found with dysplasia. Furthermore, the reproducibility of these patterns was high between endoscopists.53 Multiple studies have been performed on additional imaging technologies to enhance the detection of gastric neoplasia; however, these technologies are still investigational and currently not recommended for screening.54-57

Serum pepsinogens have been studied in Europe and Asia as noninvasive indicators of gastric atrophy to determine who should be screened with endoscopy.58 A low serum pepsinogen I level below 70 ng/mL and pepsinogen I/II ratio below 3 has generally been used to detect atrophic gastritis and at-risk populations. However, the studies performed in Europe and Asia used different methods for quantifying pepsinogen levels. Therefore, cutoff values cannot be generalized for all assays and should be validated for the specific tests used.38
 

 

 

Summary

Gastric atrophy and gastric intestinal metaplasia are considered precancerous lesions with an increased risk of development of gastric cancer. H. pylori is a major risk factor for the development of GIM. The extent of GIM as well as the presence of incomplete intestinal metaplasia, or type III intestinal metaplasia has been found to have the highest gastric cancer risk. Currently, in the United States, specific guidelines on endoscopic screening and surveillance for noncardia gastric adenocarcinoma based on histological subtype of GIM, location, and extension are lacking. The ESGE recently updated guidelines that recommend surveillance of patients with extensive atrophy and intestinal metaplasia or with a significant family history. Location and extension of intestinal metaplasia plays a role in increased risk. Screening should include a standardized upper endoscopy approach with high-definition white- light endoscopy and NBI, at least a 7-minute examination, adequate insufflation and cleaning, adequate photo documentation, and a standardized biopsy protocol. Further studies are needed to determine an appropriate surveillance interval and standardized pathology reporting approach as well.

Diana Curras-Martin MD, is an internal medicine resident at Hackensack Meridian Jersey Shore University Medical Center. Susana Gonzalez, MD, is assistant professor of medicine in the division of gastroenterology and hepatology (@WCM_GI), Weill Cornell Medicine, New York Presbyterian Hospital–Cornell. 

 

 

References

1. Bray F et al. CA Cancer J Clin. 2018;68(6):394-424.

2. Global Burden of Disease Cancer Collaboration et al. JAMA Oncol. 2018;4(11):1553-68.

3. Balakrishnan M et al. Curr Gastroenterol Rep. 2017;19(8):36.

4. Anderson WF et al. J Natl Cancer Inst. 2018;110(6):608-15.

5. Trieu JA et al. Dig Dis Sci. 2019;64(5):1079-88.

6. Lauren P. Acta Pathol Microbiol Scand. 1965;64:31-49.

7. Correa P, Schneider BG. Cancer Epidemiol Biomarkers Prev. 2005;14(8):1865-8.

8. Correa P. Cancer Res. 1992;52(24):6735-40.

9. Correa P, Piazuelo MB. J Dig Dis. 2012;13(1):2-9.

10. Correa P et al. J Natl Cancer Inst. 1970;44(2):297-306.

11. Correa P. Semin Oncol. 1985;12(1):2-10.

12. Rugge M et al. Hum Pathol. 1991;22(10):1002-8.

13. Simko V et al. Bratisl Lek Listy. 2015;116(1):3-8.

14. Giroux V, Rustgi AK. Nat Rev Cancer. 2017;17(10):594-604.

15. Jencks DS et al. Gastroenterol Hepatol (N Y). 2018;14(2):92-101.

16. Amieva M, Peek RM Jr. Gastroenterology. 2016;150(1):64-78.

17. Karimi P et al. Cancer Epidemiol Biomarkers Prev. 2014;23(5):700-13.

18. Hatakeyama M. Proc Jpn Acad Ser B Phys Biol Sci. 2017;93(4):196-219.

19. Tsutsumi R et al. Mol Cell Biol. 2006;26(1):261-76.

20. Kikuchi S et al. Am J Gastroenterol. 1999;94(12):3455-9.

21. Jass JR, Filipe MI. Histopathology. 1980;4(3):271-9.

22. Jass JR, Filipe MI. Histochem J. 1981;13(6):931-9.

23. Dixon MF et al. Am J Surg Pathol. 1996;20(10):1161-81.

24. Kang KP et al. J Gastroenterol Hepatol. 2009;24(1):140-8.

25. Gonzalez CA et al. Int J Cancer. 2010;127(11):2654-60.

26. Filipe MI et al. Gut. 1985;26(12):1319-26.

27. Filipe MI et al. Int J Cancer. 1994;57(3):324-9.

28. Gonzalez CA et al. J Gastroenterol Hepatol. 2016;31(5):953-8.

29. Cassaro M et al. Am J Gastroenterol. 2000;95(6):1431-8.

30. Shao L et al. Int J Cancer. Apr 29. 2018.

31. Stemmermann GN. Cancer. 1994;74(2):556-64.

32. Gonzalez CA et al. Int J Cancer. 2013;133(5):1023-32.

33. Reddy KM et al. Clin Gastroenterol Hepatol. 2016;14(10):1420-5.

34. Tava F et al. Hum Pathol. 2006;37(11):1489-97.

35. Fernandez-Esparrach G et al. Rev Esp Enferm Dig. 2014;106(2):120-32.

36. Ono H et al. Dig Endosc. 2016;28(1):3-15.

37. Evans JA, DeWitt JM. Gastrointest Endosc. 2016;83(1):274.

38. Pimentel-Nunes P et al. Endoscopy. 2019;51(4):365-88.

39. Capelle LG et al. Gastrointest Endosc. 2010;71(7):1150-8.

40. Saumoy M et al. Gastroenterology. 2018;155(3):648-60.

41. Gupta N et al. Gastrointest Endosc. 2011;74(3):610-24 e612.

42. Eshmuratov A et al. Dig Dis Sci. 2010;55(5):1364-75.

43. Nam JH et al. Cancer. 2012;118(20):4953-60.

44. Amin A et al. J R Coll Surg Edinb. 2002;47(5):681-4.

45. Bisschops R et al. United European Gastroenterol J. 2016;4(5):629-56.

46. Uedo N et al. Gastroenterol Clin North Am. 2013;42(2):317-35.

47. Yao K. Ann Gastroenterol. 2013;26(1):11-22.

48. Teh JL et al. Clin Gastroenterol Hepatol. 2015;13(3):480-7 e482.

49. Capelle LG et al. Dig Dis Sci. 2010;55(12):3442-8.

50. Bansal A et al. Gastrointest Endosc. 2008;67(2):210-6.

51. Tahara T et al. Gastrointest Endosc. 2009;70(2):246-53.

52. Uedo N et al. Endoscopy. 2006;38(8):819-24.

53. Pimentel-Nunes P et al. Endoscopy. 2012;44(3):236-46.

54. Kato M et al. Gastrointest Endosc. 2009;70(5):899-906.

55. Nishimura J et al. Gastroenterol Res Pract. 2014;2014:819395.

56. Dohi O et al. Gastrointest Endosc. 2019;89(1):47-57.

57. Osawa H et al. World J Gastrointest Endosc. 2012;4(8):356-61.

58. Pasechnikov V et al. World J Gastroenterol. 2014;20(38):13842-62.

 

Introduction

Despite a global decline in the incidence of gastric cancer over the past 3 decades, it remains the fifth most commonly diagnosed cancer and the third most common cause of cancer deaths worldwide.1 In the United States it is the fourth most commonly diagnosed GI malignancy, after colorectal, pancreas, and liver cancer. The prevalence remains high in Latin America and Asia, which has implications in the United States because of growing Hispanic and Asian populations.2,3 In recent years, a change in the trend of gastric cancer among non-Hispanic whites has been observed, particularly in women younger than 50 years old.4 Gastric intestinal metaplasia has been recognized worldwide as a premalignant precursor to gastric cancer, but currently, there are limited U.S. guidelines, leading to controversy over management of this condition.5

Etiology

Gastric adenocarcinomas are classified into two subcategories based on location (cardia and noncardia) and histology (intestinal and diffuse types).6,7 Atrophic gastritis and gastric intestinal metaplasia (GIM) are considered precursors of intestinal-type noncardia gastric adenocarcinoma. The Correa cascade is a commonly accepted precancer sequence for noncardia gastric adenocarcinoma that describes mucosal changes from inflammation to atrophy to metaplasia to intraepithelial neoplasia and culminating in carcinoma.8,9 It has been observed that GIM may be the histologic change prior to the development of dysplasia and over 50% of patients with high-grade dysplasia will progress to adenocarcinoma.10-12 In the United States, GIM has the highest prevalence in African Americans, Hispanics, and East Asians, with the overall GIM prevalence regardless of ethnicity reported from 3.05% to 19.2%.5,13

Risk factors and subclassification

Replacement of the foveolar and/or glandular epithelium in the oxyntic and antral mucosa by intestinal epithelium results in GIM. It can be focal when limited to one region of the stomach or extensive when two or more regions are involved.14 The main risk factors for GIM development are Helicobacter pylori infection, tobacco, alcohol consumption, high salt intake, and chronic bile reflux.15,16 Additional risks for developing gastric cancer include older age, certain ethnicities, and male sex.17

Dr. Diana Curras-Martin, internal medicine resident at Hackensack Meridian Jersey Shore University Medical Center
Dr. Diana Curras-Martin

CagA strains of H. pylori can promote carcinogenesis by inducing a mitogenic cellular response and downregulating cell adhesion.18,19 Less carcinogenic risk is associated with H. pylori Cag-A negative strains; however, they also have oncogenic potential mediated by expression of babA2 and vacA genes.20 Hence, the combination of multiple virulent factors encoded in babA2, CagA, and vacA genes has been associated with increased risk of GIM, inflammation, and development of gastric cancer.15 The clinical usefulness of genotyping H. pylori strains specifically to survey precancerous gastric lesions remains to be seen because of a lack of sufficient clinical studies. In addition, genotyping H. pylori is not commonly performed as part of clinical practice.

The loss of parietal cells seen in atrophic gastritis due to chronic H. pylori infection has been linked to the development of metaplasia due to possible loss of differentiation-promoting factors. As a result, metaplastic cells emerge that express spasmolytic polypeptide (SP or TFF2); hence, this type of metaplasia is referred to as spasmolytic polypeptide–expressing metaplasia (SPEM). The cellular mechanism that may explain a precursor role of SPEM in the development of GIM remains unknown.14 A second competing theory for the development of GIM is the clonal expansion of stem cells in the gastric isthmus that can lead to dysplasia and cancer development.14

Dr. Susana Gonzalez, assistant professor of medicine, division of gastroenterology and hepatology, Weill Cornell Medicine, New York Presbyterian Hospital-Cornell
Dr. Susana Gonzalez

On the basis of histological similarities with small intestinal or colonic epithelium, GIM can be further classified into complete or incomplete intestinal metaplasia.21 Complete intestinal metaplasia most closely resembles small intestinal epithelium with a brush border and goblet cells. Incomplete intestinal metaplasia resembles the colonic epithelium and lacks a brush border. A second classification further classifies GIM into three subtypes: Type I contains nonsecretory absorptive cells and sialomucin secreting goblet cells; type II has few absorptive cells, columnar cells secreting sialomucin, goblet cells secreting mainly sialomucin but some sulphomucin, and presence of Paneth cells; and type III consists of columnar cells secreting predominantly sulphomucin, goblet cells secreting sialomucin or sulphomucin, and absence of Paneth cells.15,22 In this subclassification, type I GIM is known as complete GIM and types II and III as incomplete GIM.23-25

Multiple studies performed outside of the United States have shown a higher progression risk to gastric adenocarcinoma in incomplete intestinal metaplasia, or type III intestinal metaplasia.26-32 Also, the risk of gastric cancer has been demonstrated to be higher among patients with a greater area of metaplasia and extensive intestinal metaplasia, defined as GIM in both the antrum and corpus.33,34 Hence, the extent of the metaplasia determined with mapping biopsies, regardless of the subtype, should also be incorporated into the risk assessment of the patient. Currently, a major limitation in the United States is a standardized method of pathologic reporting including subclassification of incomplete versus complete intestinal metaplasia.
 

 

 

Which patients to screen

Understanding this sequence of carcinogenesis offers a potential window for screening and surveillance. Subsequently, early detection of precancerous mucosal changes would be more amenable for endoscopic submucosal dissection (ESD).35,36 Currently, U.S. society guidelines do not specifically address the management of GIM. The American Society for Gastrointestinal Endoscopy (ASGE) guidelines for management of premalignant and malignant conditions of the stomach recommend surveillance in individuals with a family history of gastric cancer or of high-risk ethnic background but with no specific optimal surveillance interval.37 Also, H. pylori treatment is recommended if identified, but empiric treatment in GIM was felt to be controversial. The AGA recently sought comments on a proposed new guideline for the management of GIM. This guideline should be released after the comment period and help address management of GIM in the United States. In April of 2019, the European Society of Gastrointestinal Endoscopy (ESGE) updated the management of epithelial precancerous conditions and lesions in the stomach (MAPS II) guideline.38 The MAPS II guideline identifies atrophic gastritis and intestinal metaplasia as precancerous lesions. In patients with moderate to marked atrophy or GIM affecting both antral and body mucosa, ESGE recommends endoscopic surveillance with high-definition chromoendoscopy, mapping, and guided biopsies or at least two biopsies taken separately at the lesser and greater curvature of the antrum and body. H. pylori eradication was recommended if the patient tested positive.

Furthermore, MAPS II proposed replacing atrophic gastritis (AG) in the Operative Link on Gastritis Assessment (OLGA) staging by GIM (OLGIM) as it is considered a more reliable predictor of an individual’s gastric neoplasia risk, based on the interobserver agreement kappa value 0.6 for AG versus 0.9 for GIM.39 Five biopsies (two from the antrum, two from the corpus, and one from the incisura angularis) are needed for the OLGA/OLGIM score system to be considered an accurate predictor of this risk.39 This is supported by the early findings of gastric atrophy and GIM in the incisura angularis.23 In addition, for patients with GIM only in either the antrum or the body, a family history of gastric cancer, incomplete GIM, autoimmune gastritis, or persistent H. pylori infection was felt to increase the risk to warrant surveillance every 3 years. In those patients with atrophy or GIM in both the antrum and body with a first-degree relative with gastric cancer, surveillance was recommended every 1-2 years. Patients with any dysplasia and a visible lesion should have staging and resection. With no visible lesion, a follow-up endoscopy should be performed in 6 months with high-grade dysplasia and with low-grade dysplasia a repeat in 12 months. Patients with mild to moderate atrophy in the antrum and no intestinal metaplasia were not felt to warrant any further surveillance. (See Figure 1.)

Figure 1. Surveillance recommendations according to MAPS II, April 2019
A recent study explored the cost-effectiveness of noncardia gastric cancer screening in the United States stratified by race or ethnicity with a time horizon of 30 years. The study determined that performing endoscopic screening with mapping biopsies in high-risk patients (non-Hispanic black, Hispanic, and Asian individuals) from 50 years of age with continued surveillance only when indicated would be cost effective compared to a no-screening strategy. These patients had sampling performed via an updated Sydney protocol. If GIM was found, the patients would be enrolled into a 3-year surveillance program. Whereas if dysplasia was present, the patients would undergo endoscopic submucosal dissection or surgical resection and continue a postresection surveillance schedule.40,41
 

 

 

How to screen

Previous studies have found a poor correlation between the endoscopic determination of gastric atrophy and the histologic diagnosis.42 Several studies also found that gastric cancer was missed on initial endoscopic examinations. Sensitivity of endoscopy to detect gastric cancer has ranged from 77% to 93%.43,44 In the United States, there is a lack of standardized quality indicators for upper endoscopy exams. The ESGE has suggested several performance measures to ensure a quality endoscopy exam, including accurate photo documentation, sufficient procedure time of at least 7 minutes, adherence to biopsy protocols, and low complication rates.45 In Asia, a systematic screening protocol is used for photo documentation, and simple techniques such as adequate air insufflation and irrigation to remove mucus are routinely used to improve the endoscopy exam.46,47 The mean time of an endoscopy exam has also been found to increase the detection of neoplastic lesions, as slow endoscopists – with a mean exam duration of 8.6 ± 4.2 min during upper endoscopy – detected threefold more neoplastic lesions than did fast endoscopists.48

A standardized biopsy approach is also important when screening patients. The updated Sydney protocol has been suggested for mapping the stomach to screen for atrophy and GIM. This protocol recommends two biopsies from the antrum (at the lesser and greater curvature), two from the body (at the lesser and greater curvature), and one from the incisura.23 This biopsy protocol was also suggested in the recent MAPS II update, with the biopsy of the incisura felt to be an additional biopsy left to the discretion of the endoscopist. Notably, abnormal appearing mucosal areas should be biopsied separately from the mapping biopsies.

High-definition endoscopy with virtual chromoendoscopy is felt to be better than white-light endoscopy alone at detecting precancerous gastric lesions.38 (See Figure 2.)

Figure 2. A. High definition white light endoscopy of patient with diffuse gastric intestinal metaplasia. B. NBI image of patient with diffuse GIM shows ridge and villous appearance. C. High powered H&E of biopsy shows intestinal metaplasia.
Courtesy Diana Curras-Martin, MD, and Susana Gonzalez, MD
Figure 2. A. High definition white light endoscopy of patient with diffuse gastric intestinal metaplasia. B. NBI image of patient with diffuse GIM shows ridge and villous appearance. C. High powered H&E of biopsy shows intestinal metaplasia.

In particular, narrow-band imaging (NBI) has been studied and found to increase the diagnostic yield of GIM and dysplasia compared with white light alone.49 Several studies have shown an increased accuracy for the detection of GIM with magnification NBI.50-52 An unfortunate limitation is the geographic availability of magnification NBI: It is not available in the United States. A multicenter study in Portugal developed a new classification system for the appearance of precancerous lesions with NBI and tested its accuracy in endoscopists with a wide range of NBI experience. An abnormal mucosal pattern that showed light blue crests/regular ridge or a tubulovillous appearance and a regular mucosal pattern was found with GIM. An irregular vascular pattern with a white opaque substance and an absent or irregular mucosal pattern was most often found with dysplasia. Furthermore, the reproducibility of these patterns was high between endoscopists.53 Multiple studies have been performed on additional imaging technologies to enhance the detection of gastric neoplasia; however, these technologies are still investigational and currently not recommended for screening.54-57

Serum pepsinogens have been studied in Europe and Asia as noninvasive indicators of gastric atrophy to determine who should be screened with endoscopy.58 A low serum pepsinogen I level below 70 ng/mL and pepsinogen I/II ratio below 3 has generally been used to detect atrophic gastritis and at-risk populations. However, the studies performed in Europe and Asia used different methods for quantifying pepsinogen levels. Therefore, cutoff values cannot be generalized for all assays and should be validated for the specific tests used.38
 

 

 

Summary

Gastric atrophy and gastric intestinal metaplasia are considered precancerous lesions with an increased risk of development of gastric cancer. H. pylori is a major risk factor for the development of GIM. The extent of GIM as well as the presence of incomplete intestinal metaplasia, or type III intestinal metaplasia has been found to have the highest gastric cancer risk. Currently, in the United States, specific guidelines on endoscopic screening and surveillance for noncardia gastric adenocarcinoma based on histological subtype of GIM, location, and extension are lacking. The ESGE recently updated guidelines that recommend surveillance of patients with extensive atrophy and intestinal metaplasia or with a significant family history. Location and extension of intestinal metaplasia plays a role in increased risk. Screening should include a standardized upper endoscopy approach with high-definition white- light endoscopy and NBI, at least a 7-minute examination, adequate insufflation and cleaning, adequate photo documentation, and a standardized biopsy protocol. Further studies are needed to determine an appropriate surveillance interval and standardized pathology reporting approach as well.

Diana Curras-Martin MD, is an internal medicine resident at Hackensack Meridian Jersey Shore University Medical Center. Susana Gonzalez, MD, is assistant professor of medicine in the division of gastroenterology and hepatology (@WCM_GI), Weill Cornell Medicine, New York Presbyterian Hospital–Cornell. 

 

 

References

1. Bray F et al. CA Cancer J Clin. 2018;68(6):394-424.

2. Global Burden of Disease Cancer Collaboration et al. JAMA Oncol. 2018;4(11):1553-68.

3. Balakrishnan M et al. Curr Gastroenterol Rep. 2017;19(8):36.

4. Anderson WF et al. J Natl Cancer Inst. 2018;110(6):608-15.

5. Trieu JA et al. Dig Dis Sci. 2019;64(5):1079-88.

6. Lauren P. Acta Pathol Microbiol Scand. 1965;64:31-49.

7. Correa P, Schneider BG. Cancer Epidemiol Biomarkers Prev. 2005;14(8):1865-8.

8. Correa P. Cancer Res. 1992;52(24):6735-40.

9. Correa P, Piazuelo MB. J Dig Dis. 2012;13(1):2-9.

10. Correa P et al. J Natl Cancer Inst. 1970;44(2):297-306.

11. Correa P. Semin Oncol. 1985;12(1):2-10.

12. Rugge M et al. Hum Pathol. 1991;22(10):1002-8.

13. Simko V et al. Bratisl Lek Listy. 2015;116(1):3-8.

14. Giroux V, Rustgi AK. Nat Rev Cancer. 2017;17(10):594-604.

15. Jencks DS et al. Gastroenterol Hepatol (N Y). 2018;14(2):92-101.

16. Amieva M, Peek RM Jr. Gastroenterology. 2016;150(1):64-78.

17. Karimi P et al. Cancer Epidemiol Biomarkers Prev. 2014;23(5):700-13.

18. Hatakeyama M. Proc Jpn Acad Ser B Phys Biol Sci. 2017;93(4):196-219.

19. Tsutsumi R et al. Mol Cell Biol. 2006;26(1):261-76.

20. Kikuchi S et al. Am J Gastroenterol. 1999;94(12):3455-9.

21. Jass JR, Filipe MI. Histopathology. 1980;4(3):271-9.

22. Jass JR, Filipe MI. Histochem J. 1981;13(6):931-9.

23. Dixon MF et al. Am J Surg Pathol. 1996;20(10):1161-81.

24. Kang KP et al. J Gastroenterol Hepatol. 2009;24(1):140-8.

25. Gonzalez CA et al. Int J Cancer. 2010;127(11):2654-60.

26. Filipe MI et al. Gut. 1985;26(12):1319-26.

27. Filipe MI et al. Int J Cancer. 1994;57(3):324-9.

28. Gonzalez CA et al. J Gastroenterol Hepatol. 2016;31(5):953-8.

29. Cassaro M et al. Am J Gastroenterol. 2000;95(6):1431-8.

30. Shao L et al. Int J Cancer. Apr 29. 2018.

31. Stemmermann GN. Cancer. 1994;74(2):556-64.

32. Gonzalez CA et al. Int J Cancer. 2013;133(5):1023-32.

33. Reddy KM et al. Clin Gastroenterol Hepatol. 2016;14(10):1420-5.

34. Tava F et al. Hum Pathol. 2006;37(11):1489-97.

35. Fernandez-Esparrach G et al. Rev Esp Enferm Dig. 2014;106(2):120-32.

36. Ono H et al. Dig Endosc. 2016;28(1):3-15.

37. Evans JA, DeWitt JM. Gastrointest Endosc. 2016;83(1):274.

38. Pimentel-Nunes P et al. Endoscopy. 2019;51(4):365-88.

39. Capelle LG et al. Gastrointest Endosc. 2010;71(7):1150-8.

40. Saumoy M et al. Gastroenterology. 2018;155(3):648-60.

41. Gupta N et al. Gastrointest Endosc. 2011;74(3):610-24 e612.

42. Eshmuratov A et al. Dig Dis Sci. 2010;55(5):1364-75.

43. Nam JH et al. Cancer. 2012;118(20):4953-60.

44. Amin A et al. J R Coll Surg Edinb. 2002;47(5):681-4.

45. Bisschops R et al. United European Gastroenterol J. 2016;4(5):629-56.

46. Uedo N et al. Gastroenterol Clin North Am. 2013;42(2):317-35.

47. Yao K. Ann Gastroenterol. 2013;26(1):11-22.

48. Teh JL et al. Clin Gastroenterol Hepatol. 2015;13(3):480-7 e482.

49. Capelle LG et al. Dig Dis Sci. 2010;55(12):3442-8.

50. Bansal A et al. Gastrointest Endosc. 2008;67(2):210-6.

51. Tahara T et al. Gastrointest Endosc. 2009;70(2):246-53.

52. Uedo N et al. Endoscopy. 2006;38(8):819-24.

53. Pimentel-Nunes P et al. Endoscopy. 2012;44(3):236-46.

54. Kato M et al. Gastrointest Endosc. 2009;70(5):899-906.

55. Nishimura J et al. Gastroenterol Res Pract. 2014;2014:819395.

56. Dohi O et al. Gastrointest Endosc. 2019;89(1):47-57.

57. Osawa H et al. World J Gastrointest Endosc. 2012;4(8):356-61.

58. Pasechnikov V et al. World J Gastroenterol. 2014;20(38):13842-62.

 

Publications
Publications
Topics
Article Type
Sections
Disallow All Ads
Content Gating
No Gating (article Unlocked/Free)
Alternative CME
Disqus Comments
Default
Use ProPublica
Hide sidebar & use full width
render the right sidebar.

We owe a lot to scientists like Henry Lynch

Article Type
Changed
Wed, 08/14/2019 - 10:04

It is with great sadness that we note the passing (June 2, 2019; age 91) of Dr. Henry Lynch. Dr. Lynch almost singlehandedly brought attention to the genetic syndrome that bears his name. In 1913 Aldred Warthin (pathology chair at the University of Michigan) first described family “G”, the family of his seamstress who had told him that her family all dies of cancer. She herself succumbed to endometrial cancer. A plaque commemorating Dr. Warthin hangs down the hallway from my office at Michigan. His report fell into obscurity until the 1960s when Dr. Lynch arranged a reunion of family G in Ann Arbor, leading to a detailed update of the family in 1971. He recognized the autosomal dominance of the pedigree pattern.

Dr. John I. Allen

In 1973, C. Richard Boland, MD, AGAF (past AGA President), wrote a medical school thesis entitled “A Familial Cancer Syndrome” and subsequently published two papers in which he first used the term “Lynch syndrome (I and II). Dr. Boland (whose family also carried a Lynch syndrome variant) spent his career adding to our molecular and clinical knowledge about nonpolyposis colon cancer syndromes. In the 1990s Vogelstein and others first described the molecular pathways that lead to colon cancer – and the rest is history.

I was a young faculty gastroenterologist at the Minneapolis VA Medical Center when one day my phone rang; it was Henry Lynch. He wanted to alert me that one of his patients was coming to me for surveillance colonoscopy. He explained the importance of what I was to do and how I should follow this man. I was overwhelmed by his attention to his patient (one of thousands) and his kindness to me. I had the privilege of traveling with him as visiting professors on a trip to South America. He was one of the kindest, most intelligent, and gracious persons I had ever met. I never forgot that experience.

We owe a lot to scientists, clinicians, and thought leaders like Henry Lynch who provide us the scientific basis of the care we give our patients.
 

John I. Allen, MD, MBA, AGAF
Editor in Chief

Publications
Topics
Sections

It is with great sadness that we note the passing (June 2, 2019; age 91) of Dr. Henry Lynch. Dr. Lynch almost singlehandedly brought attention to the genetic syndrome that bears his name. In 1913 Aldred Warthin (pathology chair at the University of Michigan) first described family “G”, the family of his seamstress who had told him that her family all dies of cancer. She herself succumbed to endometrial cancer. A plaque commemorating Dr. Warthin hangs down the hallway from my office at Michigan. His report fell into obscurity until the 1960s when Dr. Lynch arranged a reunion of family G in Ann Arbor, leading to a detailed update of the family in 1971. He recognized the autosomal dominance of the pedigree pattern.

Dr. John I. Allen

In 1973, C. Richard Boland, MD, AGAF (past AGA President), wrote a medical school thesis entitled “A Familial Cancer Syndrome” and subsequently published two papers in which he first used the term “Lynch syndrome (I and II). Dr. Boland (whose family also carried a Lynch syndrome variant) spent his career adding to our molecular and clinical knowledge about nonpolyposis colon cancer syndromes. In the 1990s Vogelstein and others first described the molecular pathways that lead to colon cancer – and the rest is history.

I was a young faculty gastroenterologist at the Minneapolis VA Medical Center when one day my phone rang; it was Henry Lynch. He wanted to alert me that one of his patients was coming to me for surveillance colonoscopy. He explained the importance of what I was to do and how I should follow this man. I was overwhelmed by his attention to his patient (one of thousands) and his kindness to me. I had the privilege of traveling with him as visiting professors on a trip to South America. He was one of the kindest, most intelligent, and gracious persons I had ever met. I never forgot that experience.

We owe a lot to scientists, clinicians, and thought leaders like Henry Lynch who provide us the scientific basis of the care we give our patients.
 

John I. Allen, MD, MBA, AGAF
Editor in Chief

It is with great sadness that we note the passing (June 2, 2019; age 91) of Dr. Henry Lynch. Dr. Lynch almost singlehandedly brought attention to the genetic syndrome that bears his name. In 1913 Aldred Warthin (pathology chair at the University of Michigan) first described family “G”, the family of his seamstress who had told him that her family all dies of cancer. She herself succumbed to endometrial cancer. A plaque commemorating Dr. Warthin hangs down the hallway from my office at Michigan. His report fell into obscurity until the 1960s when Dr. Lynch arranged a reunion of family G in Ann Arbor, leading to a detailed update of the family in 1971. He recognized the autosomal dominance of the pedigree pattern.

Dr. John I. Allen

In 1973, C. Richard Boland, MD, AGAF (past AGA President), wrote a medical school thesis entitled “A Familial Cancer Syndrome” and subsequently published two papers in which he first used the term “Lynch syndrome (I and II). Dr. Boland (whose family also carried a Lynch syndrome variant) spent his career adding to our molecular and clinical knowledge about nonpolyposis colon cancer syndromes. In the 1990s Vogelstein and others first described the molecular pathways that lead to colon cancer – and the rest is history.

I was a young faculty gastroenterologist at the Minneapolis VA Medical Center when one day my phone rang; it was Henry Lynch. He wanted to alert me that one of his patients was coming to me for surveillance colonoscopy. He explained the importance of what I was to do and how I should follow this man. I was overwhelmed by his attention to his patient (one of thousands) and his kindness to me. I had the privilege of traveling with him as visiting professors on a trip to South America. He was one of the kindest, most intelligent, and gracious persons I had ever met. I never forgot that experience.

We owe a lot to scientists, clinicians, and thought leaders like Henry Lynch who provide us the scientific basis of the care we give our patients.
 

John I. Allen, MD, MBA, AGAF
Editor in Chief

Publications
Publications
Topics
Article Type
Sections
Disallow All Ads
Content Gating
No Gating (article Unlocked/Free)
Alternative CME
Disqus Comments
Default
Use ProPublica
Hide sidebar & use full width
render the right sidebar.

“Cupping” With Pain

Article Type
Changed
Thu, 08/01/2019 - 00:01
Display Headline
“Cupping” With Pain

A 30-year-old woman with a history of chronic overexposure to UV light presents to dermatology for a routine skin exam. The patient has a history of poor toleration to UV light, especially as a child, but participated in regular tanning as a teen. However, she stopped tanning when her sister developed a melanoma.

Additionally, the patient has been experiencing upper back pain, for which she has seen a variety of providers. Most recently, she consulted a naturopath, who recommended cupping therapy. Although the patient believes the therapy is alleviating her pain, she is distressed by the subsequent formation of large blemishes on her back and asks about possible treatment.

“Cupping” With Pain

EXAMINATION
There are 10 large round patches, each measuring 7 cm in diameter, on the patient’s back. These patches consist of multiple petechiae and brown hyperpigmentation. On palpation, there is no surface disturbance or tenderness. The discoloration is nonblanchable. The size, shape, and configuration of the lesions is consistent with the patient's description of the cupping procedures she has undergone on several occasions.

Notably, the patient's skin is categorized as type II on the Fitzpatrick scale, with advanced dermatoheliosis.

What’s the diagnosis?

 

 

DISCUSSION
"Cupping," as medical therapy, was first described in ancient texts 3000 to 4000 years ago. The application of cups to the patient’s skin was intended to draw out substances (eg, toxins and fluids) inside the body that were believed to cause a variety of ailments. Though its use has long since been discarded in mainstream medicine, it is still used routinely in both Chinese and alternative medicine.

Cupping has been evaluated by numerous medical individuals and organizations, who uniformly dismiss any benefit it might offer, even as a placebo. From a pathophysiologic standpoint, cupping causes localized dilation of blood and lymph vessels, thus creating telangiectasia that, as they resolve, leave behind postinflammatory hyperpigmentation and edema. (Excessive production of telangiectasia might indicate pathologic capillary fragility, possibly secondary to Rumpel-Leede phenomenon.)

The patient's skin type can affect the rate of resolution (longer for those with darker skin, shorter for those with fair skin); there is little we can do to speed up this process. Although the case patient was disappointed with the lack of available treatment for her blemishes, she was insistent about continuing the cupping therapy.

Interestingly, there is a differential diagnosis for such lesions; it includes injury from tennis balls, racquetballs, paintballs, or even baseballs—though the associated lesions are usually solitary.

TAKE-HOME LEARNING POINTS

  • Cupping, as medical therapy, has been around for thousands of years and is still routinely used in both Chinese and alternative medicine.
  • The intention of its use is to draw out noxious substances that purportedly cause the patient's complaint—however, according to numerous medical authorities, the practice is totally ineffective.
  • The suction effect of cupping induces edema and telangiectasia, which in turn results in postinflammatory hyperpigmentation that clears slowly.
  • Similar lesions can result from being struck by paintballs, racquetballs, tennis balls, and baseballs.
Publications
Topics
Sections

A 30-year-old woman with a history of chronic overexposure to UV light presents to dermatology for a routine skin exam. The patient has a history of poor toleration to UV light, especially as a child, but participated in regular tanning as a teen. However, she stopped tanning when her sister developed a melanoma.

Additionally, the patient has been experiencing upper back pain, for which she has seen a variety of providers. Most recently, she consulted a naturopath, who recommended cupping therapy. Although the patient believes the therapy is alleviating her pain, she is distressed by the subsequent formation of large blemishes on her back and asks about possible treatment.

“Cupping” With Pain

EXAMINATION
There are 10 large round patches, each measuring 7 cm in diameter, on the patient’s back. These patches consist of multiple petechiae and brown hyperpigmentation. On palpation, there is no surface disturbance or tenderness. The discoloration is nonblanchable. The size, shape, and configuration of the lesions is consistent with the patient's description of the cupping procedures she has undergone on several occasions.

Notably, the patient's skin is categorized as type II on the Fitzpatrick scale, with advanced dermatoheliosis.

What’s the diagnosis?

 

 

DISCUSSION
"Cupping," as medical therapy, was first described in ancient texts 3000 to 4000 years ago. The application of cups to the patient’s skin was intended to draw out substances (eg, toxins and fluids) inside the body that were believed to cause a variety of ailments. Though its use has long since been discarded in mainstream medicine, it is still used routinely in both Chinese and alternative medicine.

Cupping has been evaluated by numerous medical individuals and organizations, who uniformly dismiss any benefit it might offer, even as a placebo. From a pathophysiologic standpoint, cupping causes localized dilation of blood and lymph vessels, thus creating telangiectasia that, as they resolve, leave behind postinflammatory hyperpigmentation and edema. (Excessive production of telangiectasia might indicate pathologic capillary fragility, possibly secondary to Rumpel-Leede phenomenon.)

The patient's skin type can affect the rate of resolution (longer for those with darker skin, shorter for those with fair skin); there is little we can do to speed up this process. Although the case patient was disappointed with the lack of available treatment for her blemishes, she was insistent about continuing the cupping therapy.

Interestingly, there is a differential diagnosis for such lesions; it includes injury from tennis balls, racquetballs, paintballs, or even baseballs—though the associated lesions are usually solitary.

TAKE-HOME LEARNING POINTS

  • Cupping, as medical therapy, has been around for thousands of years and is still routinely used in both Chinese and alternative medicine.
  • The intention of its use is to draw out noxious substances that purportedly cause the patient's complaint—however, according to numerous medical authorities, the practice is totally ineffective.
  • The suction effect of cupping induces edema and telangiectasia, which in turn results in postinflammatory hyperpigmentation that clears slowly.
  • Similar lesions can result from being struck by paintballs, racquetballs, tennis balls, and baseballs.

A 30-year-old woman with a history of chronic overexposure to UV light presents to dermatology for a routine skin exam. The patient has a history of poor toleration to UV light, especially as a child, but participated in regular tanning as a teen. However, she stopped tanning when her sister developed a melanoma.

Additionally, the patient has been experiencing upper back pain, for which she has seen a variety of providers. Most recently, she consulted a naturopath, who recommended cupping therapy. Although the patient believes the therapy is alleviating her pain, she is distressed by the subsequent formation of large blemishes on her back and asks about possible treatment.

“Cupping” With Pain

EXAMINATION
There are 10 large round patches, each measuring 7 cm in diameter, on the patient’s back. These patches consist of multiple petechiae and brown hyperpigmentation. On palpation, there is no surface disturbance or tenderness. The discoloration is nonblanchable. The size, shape, and configuration of the lesions is consistent with the patient's description of the cupping procedures she has undergone on several occasions.

Notably, the patient's skin is categorized as type II on the Fitzpatrick scale, with advanced dermatoheliosis.

What’s the diagnosis?

 

 

DISCUSSION
"Cupping," as medical therapy, was first described in ancient texts 3000 to 4000 years ago. The application of cups to the patient’s skin was intended to draw out substances (eg, toxins and fluids) inside the body that were believed to cause a variety of ailments. Though its use has long since been discarded in mainstream medicine, it is still used routinely in both Chinese and alternative medicine.

Cupping has been evaluated by numerous medical individuals and organizations, who uniformly dismiss any benefit it might offer, even as a placebo. From a pathophysiologic standpoint, cupping causes localized dilation of blood and lymph vessels, thus creating telangiectasia that, as they resolve, leave behind postinflammatory hyperpigmentation and edema. (Excessive production of telangiectasia might indicate pathologic capillary fragility, possibly secondary to Rumpel-Leede phenomenon.)

The patient's skin type can affect the rate of resolution (longer for those with darker skin, shorter for those with fair skin); there is little we can do to speed up this process. Although the case patient was disappointed with the lack of available treatment for her blemishes, she was insistent about continuing the cupping therapy.

Interestingly, there is a differential diagnosis for such lesions; it includes injury from tennis balls, racquetballs, paintballs, or even baseballs—though the associated lesions are usually solitary.

TAKE-HOME LEARNING POINTS

  • Cupping, as medical therapy, has been around for thousands of years and is still routinely used in both Chinese and alternative medicine.
  • The intention of its use is to draw out noxious substances that purportedly cause the patient's complaint—however, according to numerous medical authorities, the practice is totally ineffective.
  • The suction effect of cupping induces edema and telangiectasia, which in turn results in postinflammatory hyperpigmentation that clears slowly.
  • Similar lesions can result from being struck by paintballs, racquetballs, tennis balls, and baseballs.
Publications
Publications
Topics
Article Type
Display Headline
“Cupping” With Pain
Display Headline
“Cupping” With Pain
Sections
Disallow All Ads
Content Gating
No Gating (article Unlocked/Free)
Alternative CME
Disqus Comments
Default
Gate On Date
Mon, 07/29/2019 - 11:30
Un-Gate On Date
Mon, 07/29/2019 - 11:30
Use ProPublica
CFC Schedule Remove Status
Mon, 07/29/2019 - 11:30
Hide sidebar & use full width
render the right sidebar.